You are on page 1of 250

1000 Practice Questions for SBI Clerk Prelims 2023

Part - 1
Direction (1-4): Study the following information carefully to Q4. Four of the following five are alike in a certain way and
answer the given questions: hence form a group. Who among the following does not belong
to that group?
A certain number of persons sit in a linear row and all of them (a) B
face towards north direction. Six persons sit between R and M. (b) E
A sits third to the left of M. C sits three places away from R. (c) C
(d) T
One person sits between E and C. E sits to the left of R but not
(e) R
immediate left. The number of persons sit between E and R is
same as the number of persons sit between B and E. B is the Direction (5-8): Study the following information carefully
and answer the given questions.
only neighbour of T. The number of persons sit to the left of T
is two less than the number of persons sit to the right of M. K 798XE&MS46R©5U#NQ2€H%61PK3A@
sits at the extreme end. Q5. If all the symbols are dropped from the series, which
element will be 10th from the right end of the new
Q1. How many persons sit in the linear row? arrangement?
(a) 22 (a) 4
(b) 21 (b) 6
(c) 20 (c) N
(d) 23 (d) Z
(e) None of these
(e) 19
Q6. How many such symbols are there in the given series
Q2. Who among the following sits fifth to the left of K? which are immediately preceded by a consonant and
(a) E immediately followed by a number?
(b) C (a) None
(c) R (b) One
(c) Two
(d) A
(d) Three
(e) None of these (e) Four
Q3. What is the position of E with respect to T? Q7. How many such numbers are there in the given series
(a) Sixth to the left which are immediately preceded by a number and
(b) Fifth to the right immediately followed by a consonant?
(a) None
(c) Sixth to the right
(b) One
(d) Fourth to the left (c) Two
(e) Immediate left (d) Three
(e) Four
Q8. Which among the following letter will be 3rd to the left of
11th letter from the left end?
(a) K
(b) Q
(c) M
(d) A
(e) None of these
Direction (9-13): Study the following information carefully
to answer the given questions:
Ten persons sit in a parallel row such that five persons sit in
each row. In row 1- P, Q, R, S and T sit and face south while in
row 2- A, B, C, D and E sit and face north. The persons sit in
row 1 face the persons sit in row 2 and vice versa.
1 adda247.com/teachers | www.sscadda.com | www.bankersadda.com | www.adda247.com
A faces the one who sits third to the right of T. Two persons sit Q15. What is the shortest distance between point U and point
between A and B. The one who sits second to the left of B faces T?
the one who sits immediate left of P. More than two persons (a) 13m
sit between T and R. D sits second to the left of C who doesn’t (b) 5√13m
face Q. (c) 89m
Q9. Who among the following sits third to the right of Q? (d) √89m
(a) S (e) Can’t be determined
(b) P
Q16. If 1 is added to each even digit and 2 is subtracted from
(c) R
(d) Either R or S each odd digit in the number “32547685”, then how many
(e) Either P or S digits will appear more than once in the new number thus
formed?
Q10. Which among the following statement(s) is/are true? (a) One
(a) More than one person sits to the right of B (b) Three
(b) C sits exactly in the middle of A and B
(c) Two
(c) P sits adjacent to Q
(d) None
(d) No one sits between B and E
(e) None of these
(e) All are true
Q11. Four of the following five are alike in a certain way and Direction (17-18): In this question, relationship between
hence form a group. Who among the following does not belong different elements is shown in the statements. The statements
to that group? are followed by conclusions. Study the conclusions based on
(a) P the given statement and select the appropriate answer.
(b) Q
Q17. Statements: M > B ≥ V; C ≤ B = Z; K ≥ J < Z
(c) A
Conclusions: I: V≤Z II: J<C
(d) E
(e) T (a) If only conclusion I is true
(b) If only conclusion II is true
Q12. Who among the following faces R? (c) If either conclusion I or II is true
(a) D (d) If neither conclusion I nor II is true
(b) A (e) If both conclusions I and II are true
(c) C
(d) E Q18. Statements: Q ≥ E ≤ T ; Y = I < T ; D > I ≥ F
(e) None of these Conclusions: I: I<Q II: D>Y
Q13. How many persons sit between Q and the one who faces (a) If only conclusion I is true
A? (b) If only conclusion II is true
(a) None (c) If either conclusion I or II is true
(b) Two (d) If neither conclusion I nor II is true
(c) Three (e) If both conclusions I and II are true
(d) One
Directions (19-23): Study the following information
(e) None of these
carefully and answer the questions given below:
Direction (14-15): Study the following information carefully
to answer the given questions: Seven persons P, Q, R, S, T, U and V live on separate floors of 7-
floor building but not necessarily in the same order such as
A person starts walking from point X in east direction after ground floor is numbered as 1st floor, just above floor is
walking 10m he stops at point T. From point T, he walks 8m in numbered as 2nd floor and so on until the topmost floor is
north and reaches at point M. Now, from point M he walks 5m
numbered as 7th floor.
in west direction and reaches at point U. From there he walks
P lives on an odd numbered floor but above the 4th floor. Three
12m in east direction and reaches at point K which is 13m in
persons live between P and U. No one lives between R and U.
north of point R. From point R he walks 9m in east direction
and reaches at point S. T lives two floors above V who lives just above S’s floor.

Q14. In which direction is point K with respect to point S? Q19. How many floors gap between P and Q?
(a) North -east (a) One
(b) South (b) None
(c) North-west (c) Two
(d) South -west (d) Three
(e) West (e) More than three

2 adda247.com/teachers | www.sscadda.com | www.bankersadda.com | www.adda247.com


Q20. Four of the following five are alike in a certain way and (a) Either conclusion I or II follows
hence they form a group. Who among the following does not (b) Only conclusion II follows
belong to that group? (c) Only conclusion I follows
(a) Q (d) Both conclusions I and II follow
(b) U (e) Neither conclusion I nor II follows
(c) T
Q26. Statements: Only a few black is blue
(d) P
Only a few blue is yellow
(e) S
No white is yellow
Q21. Which among the following statement is true? Conclusions: I. No blue is white
I. V lives below U’s floor II. All white is blue
II. No one lives below Q (a) Either conclusion I or II follows
III. Two floors gap between U and V (b) Only conclusion II follows
(a) Only III (c) Only conclusion I follows
(b) Both II and III (d) Both conclusions I and II follow
(c) All I, II and III (e) Neither conclusion I nor II follows
(d) Only II
Directions (27-31): Study the following information
(e) None is true
carefully and answer the questions given below:
Q22. How many persons live below V?
Seven persons A, B, C, D, E, F and G are sitting around a circular
(a) Three
table and all are facing towards the centre but not necessarily
(b) Two
in the same order.
(c) Four
F sits two places away from A. C sits 2nd to the left of B who
(d) Five
does not sit near A and F. G sits 2nd to the right of D who does
(e) None of these
not sit near C. More than one person sits between A and E
Q23. Which among the following pair of persons live on an (From both sides).
even numbered floor?
Q27. Who among the following person is an immediate
(a) Q, V
(b) R, Q neighbour of B?
(c) T, V (a) G
(d) R, P (b) C
(e) None of these (c) E
(d) F
Directions (24-26): In the question below some statements (e) A
are given followed by some conclusions. You have to take the
given statements to be true even if they seem to be at variance Q28. How many persons sit between G and E, when counted
with commonly known facts. Read all the conclusions and from right of E?
then decide which of the given conclusions logically follow (a) One
from the given statements disregarding commonly known (b) Two
facts. (c) None
(d) Four
Q24. Statements: Only a few book is copy (e) Three
Some copy is pen
No ink is pen Q29. Who among the following person sits 4th to the right of
Conclusions: I. Some book is not ink G?
II. All copy can never be ink (a) E
(a) Either conclusion I or II follows (b) B
(b) Only conclusion II follows (c) A
(c) Only conclusion I follows (d) C
(d) Both conclusions I and II follow (e) None of these
(e) Neither conclusion I nor II follows Q30. What is the position of F with respect to D?
Q25. Statements: Some bank is not money (a) 3rd to the left
Only a few interest is money (b) 2nd to the left
Some loan is interest (c) 3rd to the right
Conclusions: I. Some interest is not money (d) 2nd to the right
II. Some loan can be bank (e) Immediate left

3 adda247.com/teachers | www.sscadda.com | www.bankersadda.com | www.adda247.com


Q31. Four of the following five are alike in a certain way and Q37. P invested Rs.21000 in a business and after few months
hence they form a group. Which one of the following does not Q joined him with Rs.28000. At the end of a year profit share
belong to that group? of P and Q is equal. Find after how many months B joined the
(a) D, F business.
(b) A, C (a) 4
(c) G, B
(b) 2
(d) E, C
(c) 3
(e) F, B
(d) 5
Direction (32-33): Study the following information carefully (e) 6
to answer the given questions:
Q38. Marked price of an article is Rs.800 and shopkeeper
A family has three generation. F is son in law of D who has only allow a discount of 10% on it. If shopkeeper made a profit of
one sibling. H is brother-in-law of T and spouse of D. H has no
20% on selling the article, then find the cost price of the
siblings. S is niece of E who is brother of G. G is daughter of T’s
sibling. article.
(a) 720 Rs.
Q32. How T is related to E? (b) 640 Rs.
(a) Sister (c) 700 Rs.
(b) Aunt
(d) 600 Rs.
(c) Uncle
(e) 400 Rs.
(d) Brother
(e) Can’t be determined Q39. In a school total 360 students passed the exam, while
Q33. Who among the following is daughter of F? 60% of total boys passed the exam and 40% of total girls
2
(a) D passed the exam. If 66 3 % of total passed boys is 160, then find
(b) G total students in the school.
(c) S
(a) 960
(d) Either D or S
(e) Either G or S (b) 720
(c) 800
Q34. In the word ‘DRAWINGS’, how many pairs of the letters (d) 600
have the same number of letters between them (both forward (e) 700
and backward direction) in the word as in the alphabetical
series? Q40. Sum of squares of three consecutive numbers is 1085.
(a) Four Find the largest number.
(b) Two (a) 19
(c) One (b) 20
(d) Three (c) 18
(e) More than four (d) 21
Q35. If we form a four-letter meaningful word by using the (e) 22
first, second, fifth and eighth letter from the left end of the
word ‘NECESSITY’, then which of the following will be the Q41. Three students Ria, Ram and Pooja got total 74 marks in
fourth letter of the meaningful word thus formed. If more than an exam. The ratio of marks obtained by Ria to Pooja is 4 : 5
one word is formed mark Z as your answer. If no meaningful and the ratio of marks obtained by Pooja to Ram is 3 : 2. Find
word is formed, mark X as your answer? the total marks score by Ram.
(a) N (a) 30
(b) E (b) 20
(c) S (c) 24
(d) X (d) 28
(e) Z (e) 40
Q36. A can do a work in 24 days and B can do the same work
Q42. The speed of a boat in still water is 12 km /hr. If time
in 18 days. If A starts work alone and after four days B joined
him, then find in how many days total work will be completed? taken by the boat to cover 20 km in downstream is equal to
4 time taken by the boat to cover 10 km in upstream, then find
(a) 10 7 days
4
speed (in km/hr.) of stream.
(b) 12 days (a) 2
7
4
(c) 14 days (b) 4
7
3 (c) 6
(d) 12 𝑑𝑎𝑦𝑠
7
2
(d) 3
(e) 12 7 𝑑𝑎𝑦𝑠 (e) 5

4 adda247.com/teachers | www.sscadda.com | www.bankersadda.com | www.adda247.com


Q43. The present age of A is 14 years more than B. 12 years 1
Q48. If 𝑡ℎ of total patients admitted in B in March are female
4
ago, the ratio of age of A to that of B was 2 : 1, then find the
and 60% of total patients admitted in A in April are female,
present age (in years) of A.
then find the difference between total male patients admitted
(a) 40
in A in April and in B in March together and total patients
(b) 30
admitted in C in May.
(c) 32
(d) 44 (a) 1369
(e) 48 (b) 1459
(c) 1469
Q44. A shopkeeper marked price of an article at Rs. 2450 and (d) 1419
he allows two successive discounts of 20% & x% on it. If (e) 1449
selling price of the article is Rs. 1470, then find the value of x?
(a) 12.5 Q49. Out of total patients admitted in C in March, 956 are
(b) 25 female. If total male patients admitted in C in June are 20%
(c) 15 more than total male patients admitted in C in March and total
(d) 10 patients admitted in June in C are 872 more than total patients
(e) 24 admitted in B in May, then find total female patients admitted
Q45. A and B invested same amount in two different schemes in C in June.
on simple interest for five years. A invested at the rate of 12% (a) 1700
p.a. and B invested at the rate of 18% p.a. If difference between (b) 900
interest received by A and B is Rs. 1440, then find the total (c) 1300
amount invested by A and B together. (d) 1100
(a) Rs. 5400 (e) 1500
(b) Rs. 9000
Q50. Find ratio of total patients admitted in A in April to total
(c) Rs. 4800
number of patients admitted in all three hospitals in May?
(d) Rs. 9600
(e) Rs. 4500 (a) 14 : 37
(b) 28 : 73
Q46. The average of 25 number is 90 and when one new (c) 28 : 75
number added in these 25 numbers, then average increased (d) 28 : 77
by 5. Find the new number. (e) 75 : 28
(a) 220
(b) 240 Q51. Total patients admitted in A in May is what percent
(c) 280 (Approximate) of total patients admitted in B & C in April?
(d) 300 (a) 55%
(e) 320 (b) 65%
Direction (47–51): The table given below shows total (c) 70%
number of patients admitted in three (A, B and C) different (d) 50%
hospitals in three different months. Read the data carefully (e) 60%
and answer the questions.
Hospitals March April May
A 1505 2800 2864
B 3676 1976 2228
C 2456 2804 2408

Q47. Total patients admitted in A in March is what percent


(approximate) of total patients admitted in all three hospitals
in May.
(a) 25%
(b) 20%
(c) 15%
(d) 30%
(e) 5%

5 adda247.com/teachers | www.sscadda.com | www.bankersadda.com | www.adda247.com


Direction (52–56): Find the wrong number in the following Q60. 240 × 2.5 + 65 ×
2
= ?2 − √225
13
number series:
(a) 22
Q52. 27, 48, 80, 134, 221, 355, 538 (b) 24
(a) 27 (c) 23
(b) 355 (d) 27
(c) 80 (e) 25
(d) 134
(e) 538 Q61. 4080 ÷ (? + 17) = 102 + 2
(a) 23
Q53. 215, 200, 250, 175, 275, 150, 300
(a) 150 (b) 42
(b) 300 (c) 51
(c) 250 (d) 15
(d) 200 (e) 16
(e) 215
Q62. 31.5 ÷ 3.5 × 12 – 8 = ?2
Q54. 550, 4400, 110, 440, 24, 44, 4.4 (a) 10
(a) 24 (b) 25
(b) 550 (c) 75
(c) 4400 (d) 100
(d) 4.4
(e) 50
(e) 44
Q63. 945 ÷ 35 × 20 =?
Q55. 146, 145, 149, 122, 138, 15, 49
(a) 138 (a) 540
(b) 122 (b) 550
(c) 15 (c) 560
(d) 146 (d) 570
(e) 149 (e) 580
Q56. 1.7, 3.0, 5.6, 9.5, 14.7, 21.8, 29 Q64. 60% of 700 – 450 =? – 85% of 120
(a) 3.0 (a) 84
(b) 9.5 (b) 48
(c) 21.8 (c) 64
(d) 14.7
(d) 36
(e) 29
(e) 72
Direction (57–70): What will come in the place of question 5
9?) mark in following questions. Q65. 30÷ 12 + √144 × 20 = ?
4
√256 (a) 118
Q57. 25 % of 256 + × 8 = ? +10 (b) 156
4
(a) 64 (c) 208
(b) 62 (d) 256
(c) 60 (e) 312
(d) 56
(e) 82 4 1 2 16
Q66. 2 7 + 4 3 − 3 3 + 21 =?
Q58. 274 – 141 = ? + 98 (a) 4
(a) 39 (b) 5
(b) 31 (c) 3
(c) 37 (d) 7
(d) 33 (e) 6
(e) 35
Q67. (0.9)2 × 10 + 2.9 = ? + 132 ÷ 12
Q59. 36% of 250 + 26 ÷ 2 × ? = 207
(a) 4
(a) 8
(b) 5 (b) 3
(c) 6 (c) 2
(d) 9 (d) 1
(e) 7 (e) 0

6 adda247.com/teachers | www.sscadda.com | www.bankersadda.com | www.adda247.com


Q68. 26% of 50 of 5 ÷ 26 =? The pandemic has led to many people buying more takeout
(a) 2.5 than before, while others have a newfound love of cooking.
(b) 2.25 That means _______________ still further from the utilitarian
(c) 3.5 model of the 18th century and before, and instead doubling
(d) 1.5 down on what they do best: offering those who need to eat a
(e) 2 taste of romance, glamour, and love.

Q69. (132 + 72) ÷ 12 × 17 =? +92 Q71. What effect of the pandemic on restaurants has been
(a) 208 mentioned by the author in the passage?
(a) A severe lack of essential items has hindered the supply
(b) 287
chain of restaurants.
(c) 370
(b) The period during the pandemic led to a stop in the growth
(d) 179
of restaurants.
(e) 168/
(c) The pandemic has made people realise the allure of eating
Q70. 0.26× 50 + 132 =? out at restaurants.
(a) 182 (d) Restaurants observe less customers due to a fear of lack of
(b) 181 social distancing measures.
(c) 180 (e) Due to the continuous waves of the pandemic, restaurants
(d) 189 struggle to remain open.
(e) 188 Q72. What compelled rich people to start eating out?
(a) Public spaces becoming safer for citizens
Directions (71-79): Read the passage given below and
(b) The rise of capitalism in the market
answer the following questions.
(c) Being able to show off wealth to society
The onset of Covid-19 has brought to a halt an astonishing (d) Both (b) and (c)
expansion of restaurants observed in the past decade. But (e) All of the above
being deprived of restaurants has made people realise how Q73. According to the passage, why did people prefer eating
much they value them. Eating out fulfils needs which seem out before the 17th century?
fundamental to human nature. People need to socialise, to seal (a) Eating at restaurants was cheaper for people than cooking
deals and to peer at their fellow humans. food.
People have long feasted outside the home. Before the use of (b) Restaurants provided a convenient way for people to have
coal became widespread in England in the 17th century, food.
preparing food at home involved spending a lot on wood or (c) Restaurants catered to all classes of society while avoiding
peat. Today dining out is seen as an indulgence, but it was the conflict
cheapest way to eat for most of human history. But by the 18th (d) Both (a) and (b)
century, capitalism took off, public spaces became sites of (e) All of the above
rational dialogue which were open to all. Eating out became Q74. What conundrum does the author present against the
less of a communal activity focused on calorie intake and more growth of restaurants the past century?
of a cultural experience—and a place where people could (a) Despite the economic instability during the period,
show off their wealth. restaurants continued to grow.
By the 20th century, however, cooking at home was becoming (b) Restaurants were popular despite industrialisation
ever easier. Average house sizes grew and appliances such as leading to many people becoming poor.
the food processor and the dishwasher reduced preparation (c) The interest in restaurants piqued in the period despite
and clean-up times. Dining out became relatively more many nations being war-ridden.
expensive. And yet, a few economic changes ensured that (d) The growth of restaurants despite cooking becoming
demand for restaurants grew despite rising prices. cheaper and more convenient.
One of them is immigration. Over the last century the net flow (e) The deadly plagues during the period did not dissuade
of migrants into rich countries, relative to population, more people from going out to eat.
than quadrupled. Starting a restaurant is a good career move Q75. How did migrants contribute to a rise in the growth of
for new arrivals. Migrants also tend to improve the quality of restaurants?
an area’s restaurants. Another factor was changing working (a) Migrants added to the population of a country and thus
patterns. Historically poor people have tended to work longer contributed to increased consumption.
hours than rich ones. But in the latter half of the 20th century (b) A significant portion of the migrant population started
the opposite became true. The upshot was that the people restaurants in the countries they came to.
with the most money to spend on dining out increasingly (c) Countries with a high rate of immigration gave relaxations
needed it most, since they had the least free time. on items of necessity for the public.
7 adda247.com/teachers | www.sscadda.com | www.bankersadda.com | www.adda247.com
(d) An immigrant population brought with it a different Q81. A fall in demand is enough to demonstrate that
culture, thus reviving interest in cuisine. consumers have forget about the product and moved on to
(e) Restaurants cashed up on the opportunity to serve food to other products.
an immigrant population for revenues. (a) has forgot about the product
(b) had forgotten of the product
Q76. According to the passage, which of the following is true (c) have forgotten about the product
regarding the influence of changing work patterns in the past (d) has forgotten for the product
century? (e) No replacement needed
(a) Poor people stopped eating out as much as they would in
the past. Directions (82- 85): In each sentence, a word is omitted.
(b) Restaurants improved their services to cater to more Choose the word from the given options that can fit into the
people for a longer time. blank without changing the intended meaning.
(c) Rich people having more money would want to spend Q82. Jeff faced many hardships in life, but he always found a
more on their indulgences. way to be _____________________.
(d) People earning more often had less tome to cook food at (a) lethargy
home. (b) gloomy
(e) Eating out for luxury became difficult for people due to (c) desperate
lack of time. (d) resilient
(e) pervasive
Q77. Which of the following words can most appropriately fill
the blank given in the passage? Q83. With this _______________ rainfall, we might get flooded
(a) coping out.
(b) closing (a) incessant
(c) making (b) irreversible
(d) moving (c) indecisive
(e) living (d) amiable
(e) dormant
Q78. Which of the following words is most similar in meaning
to the word “flow” as is highlighted in the passage? Q84. The plant has a pungent smell which makes it very
(a) movement ________________________ to bees.
(b) implication (a) valiant
(c) obstruction (b) vigorous
(c) appealing
(d) deduction
(d) robust
(e) increase
(e) diffusing
Q79. Choose the word that can most appropriately replace the
Q85. If the thermostat does not ______________________the
highlighted word “upshot” in the passage?
temperature properly, it may not trigger the heating unit.
(a) reminder
(a) ramble
(b) history (b) gauge
(c) requirement (c) transit
(d) consequence (d) flush
(e) condition (e) rejig
Directions (80-81): In the given question a part of the Directions (86-90): In each of the following questions, a
sentence is given in bold, which may or may not be sentence has been divided into four parts in which one part
grammatically correct. Choose the correct alternative for the has an error. Identify the part that has an error and mark that
highlighted phrase to make the sentence grammatically and part as your answer. If all parts are error-free then choose ‘No
logically correct. Error’ as your correct choice.
Q80. Because the man is angry on the teacher, he wrote a Q86.The hospital has receive (A)/ funding for medical (B)/
complaining letter to the principal. devices and (B)/ clinical studies (D)/ No Error (E).
(a) is angry at the teacher (a) A
(b) was angry with the teacher (b) B
(c) were angry to the teacher (c) C
(d) is angry with the teacher (d) D
(e) No replacement needed (e) No Error

8 adda247.com/teachers | www.sscadda.com | www.bankersadda.com | www.adda247.com


Q87. Aryabhata noted that (A)/ the luminosity of the (B)/ Q92. Which is the LAST sentence after rearrangement?
Moon and other planets are (C)/ due to reflected sunlight (D)/ (a) A
No Error (E) (b) B
(a) A (c) C
(b) B (d) D
(c) C (e) E
(d) D
(e) No Error Q93. Which is the THIRD sentence after rearrangement?
(a) A
Q88. Rainbows can been (A)/ observed whenever there (B)/ (b) B
are water drops in the (C)/ air and sunlight shining (D)/ No (c) C
Error (E). (d) D
(a) A (e) E
(b) B
(c) C Q94. Which is the FIRST sentence after rearrangement?
(d) D (a) A
(e) No Error (b) B
(c) C
Q89. Indraprastha was one (A)/ of the five place demanded (d) D
(B)/ for the sake of peace and (C)/ to avert a disastrous war (e) E
(D)/ No Error (E).
(a) A Q95. Which is the FOURTH sentence after rearrangement?
(b) B (a) A
(c) C (b) B
(d) D (c) C
(e) No Error (d) D
(e) E
Q90. Due to climate change, (A)/ deserts are expanding, (B)/
while heat waves (C)/ are becoming more commons (D)/ No Directions (96-100): In each question, three words are given
Error. in bold, which may or may not be in their correct positions.
(a) A Choose the correct sequence of words from the given options
(b) B to make the sentence grammatically correct and meaningful.
(c) C
Q96. A wildlife (A)of an eagle was carved (B)out of wood for
(d) D
the sculpture (C)event.
(e) No Error
(a) BCA
Directions (91-95): In the following questions, five (b) CBA
sentences are given. Rearrange these sentences to make a (c) ACB
contextually meaningful paragraph then, answer the (d) BAC
following questions. (e) No rearrangement needed
(A) An insufficient amount of sleep can lead to serious
Q97. Blood was analyzed (A)through a/an diagnostic
repercussions.
(B) Also, it loses the abilities to concentrate, think clearly, and (B)machine to figure out whether the patient was suffering
process memories. (C)from cancer or the flu.
(C) Sleep is an essential function that allows your body and (a) BCA
mind to recharge and leaves you refreshed. (b) CBA
(D) For instance, without enough sleep, the brain cannot (c) ACB
function properly (d) BAC
(E) Healthy sleep also helps the body to remain healthy and (e) No rearrangement needed
stave off diseases. Q98. Without any solitude (A) a/an crazy (B) is bound to go
Q91. Which is the SECOND sentence after rearrangement? a bit introvert (C)
(a) A (a) BCA
(b) B (b) CBA
(c) C (c) ACB
(d) D (d) BAC
(e) E (e) No rearrangement needed

9 adda247.com/teachers | www.sscadda.com | www.bankersadda.com | www.adda247.com


Q99. The allowed(A) material permeable (B)a large amount Q100. The astronomer observation (A)a new star during
of water to seep (C)through. a/an discover(B) recent(C)
(a) BCA (a) BCA
(b) CBA (b) CBA
(c) ACB (c) ACB
(d) BAC (d) BAC
(e) No rearrangement needed (e) No rearrangement needed

Solutions
Solution (1-4): S14. Ans.(c)
S15. Ans.(d)
S16. Ans.(c)
Sol. 32547685
S1. Ans.(b) 13355793
S2. Ans.(d)
Solution (17-18):
S3. Ans.(c)
S4. Ans.(d) S17. Ans.(a)
Sol. I: V≤Z (True) II: J<C (False)
Solution (5-8):
S18. Ans.(b)
S5. Ans.(c)
Sol. I: I<Q (False) II: D>Y (True)
S6. Ans.(c)
Solution (19-23):
Sol. Two - R © 5, H % 6
Floors Persons
S7. Ans.(d) 7 Q
Sol. 98X, 46R, 61P 6 T
5 P
S8. Ans.(b)
4 V
Solution (9-13): 3 S
2 R
1 U

S19. Ans.(a)
S20. Ans.(c)
S21. Ans.(a)
S22. Ans.(a)
S23. Ans.(c)
S9. Ans.(c)
Solution (24-26):
S10. Ans.(d)
S24. Ans.(b)
S11. Ans.(e) Sol.
S12. Ans.(a)
S13. Ans.(d)
S25. Ans.(d)
Directions (14-15): Sol.

S26. Ans.(e)
Sol.

10 adda247.com/teachers | www.sscadda.com | www.bankersadda.com | www.adda247.com


Solution (27-31): S38. Ans.(d)
Sol. Selling price of the article
90
= 800 × = 720 𝑅𝑠.
100
100
Cost price of the article = 720 × 120 = 600 𝑅𝑠.

S39. Ans.(e)
Sol. Let total boys in school be X
60 3𝑋
So, total passed boys in school = 𝑋 × =
100 5
ATQ –
3𝑋 2
5
× 3 = 160
2𝑋
= 160
5

S27. Ans.(c) X = 400


S28. Ans.(e) Total girls passed in school
3
S29. Ans.(b) = (360 – 160 × ) = 120
2
S30. Ans.(c) Total girls in school = 120 ×
100
= 300
40
S31. Ans.(d)
So, total students in school = (400 + 300) = 700
Solution (32-33):
S40. Ans.(b)
Sol. Let three consecutive numbers be (n – 1), n and (n+1)
respectively
ATQ –
(n-1)2 + n2 + (n+1)2 = 1085
3n2 + 2 = 1085
3n2 = 1083
n2 = 361
S32. Ans.(e)
S33. Ans.(c) so, n = 19
largest number = (19+1) =20
S34. Ans.(d)
S41. Ans.(b)
Sol.
Sol. The ratio of marks obtained by Ria, Pooja and Ram
respectively
Ria : Pooja = 4 : 5
Pooja : Ram = 3 : 2
Ria : Pooja : Ram = 12 : 15 : 10
10
So, marks score by Ram = 74 × = 20
S35. Ans.(e) (12+15+10)

S36. Ans.(b) S42. Ans.(b)


Sol. Total work = 72 unit (LCM of 24 & 18) Sol. Let speed of stream be ‘y’ km/hr.
72
Efficiency of A = 24 = 3 unit/day ATQ –
20 10
72 = (12−𝑦)
Efficiency of B = 18 = 4 unit/day (12+𝑦)
24 – 2y = 12 + y
Total work done by A in four days = 3 × 4 = 12 units
3y = 12
Remaining work = 72 −12 = 60 units
60 4 y=4
Required days = (4+3) + 4 = 12 7 days
S43. Ans.(a)
S37. Ans.(c) Sol. Let present age of A and B be ‘a’ & ‘b’ respectively
Sol. Let Q joined business after ‘t’ months Given, a – b = 14
ATQ – a = 14 + b --- (i)
21000×12 1
= ATQ –
28000×(12−𝑡) 1 𝑎−12 2
9 =12-t =
𝑏−12 1
t=3 a – 12 = 2b – 24
t = 3 months a = 2b – 12 ----- (ii)

11 adda247.com/teachers | www.sscadda.com | www.bankersadda.com | www.adda247.com


Form (i) & (ii) we get S52. Ans.(b)
14 + b = 2b – 12 Sol. Wrong number = 355
b = 26 Pattern of series –
So, present age of A = 26 + 14 = 40 years
S44. Ans.(b)
Sol. ATQ –
(100−20) (100−𝑥)
2450 × 100
× 100
= 1470 S53. Ans.(e)
1960 ×
(100−𝑥)
= 1470 Sol. Wrong number = 215
100
(100−𝑥) 3
Pattern of series –
= There are two alternate series
100 4
x = 25 One is - 225 + 25 = 250
250 + 25 = 275
S45. Ans.(d)
275 + 25 = 300
Sol. Let each A and B invested Rs. X
ATQ – Second is - 200 – 25 = 175
18×5 12×5 175 – 25 = 150
X× 100
−𝑋 × 100
= 1440
30 S54. Ans.(a)
X × 100 = 1440
Sol. Wrong number = 24
X = 4800 Rs.
Pattern of series –
So, total amount invested by A and B = 4800 + 4800 = 9600 Rs.
There are two alternate series
S46. Ans.(a) One is – 550 ÷ 5 = 110
Sol. Sum of the 25 numbers = 25 × 90 = 2250 110 ÷ 5 = 22
When a new number added then total = (90 +5) × 26 = 2470 22 ÷ 5 = 4.4
So, new number = 2470 – 2250 = 220 Second is – 4400 ÷ 10 = 440
440 ÷ 10 = 44
S47. Ans.(b)
1505
Sol. Required percentage = (2864+2228+2408) × 100 S55. Ans.(c)
1505 Sol. Wrong number = 15
= 7500 × 100 = 20.06 ≈ 20%
Pattern of series –
S48. Ans.(c) 146 – 13 = 145
Sol. Total male patients admitted in A in April 145 + 22 = 149
40 149 – 33 = 122
= 2800 × 100 = 1120
122 + 42 = 138
Total male patients admitted in B in March 138 – 53 = 13
3
= 3676 × 4 = 2757 13 + 62 = 49
Required difference
S56. Ans.(c)
= (1120 + 2757) −2408 = 1469
Sol. Wrong number = 21.8
S49. Ans.(c) Pattern of series –
Sol. Total male patients admitted in C in March = 2456 – 956 1.7 + 1.3 = 3.0
= 1500 3.0 + 2.6 = 5.6
120
So, total male patients admitted in C in June = 1500 × = 5.6 + 3.9 = 9.5
100
1800 9.5 + 5.2 =14.7
Total female patients admitted in C in June = (2228 + 872) – 14.7 + 6.5 =21.2
1800 = 1300 21.2+7.8 = 29

S50. Ans.(c) S57. Ans.(b)


256 4
Sol. Required ratio = 2800 : (2864 + 2228 + 2408) = 2800 ∶ Sol. 4
+ 4 × 8 =? +10
7500 = 28 ∶ 75 64 + 8 =? + 10
S51. Ans.(e) ? = 62
Sol. Total patients admitted in B & C in April S58. Ans.(e)
= (1976 + 2804) = 4780 Sol. ? = 274 −141 − 98
2864
Required percentage = 4780 × 100 = 59.91 ≈ 60% ? = 35

12 adda247.com/teachers | www.sscadda.com | www.bankersadda.com | www.adda247.com


S59. Ans.(d) S71. Ans.(b)
36 26
Sol. 100 × 250 + 2 ×? = 207 Sol. Refer to the first line of the first paragraph in the passage
for justification. “The onset of Covid-19 has brought to a halt
13 ×? = 207 − 90
an astonishing expansion of restaurants observed in the past
?=9
decade.” Although the statement given by option (c) is a
S60. Ans.(e) correct statement according to the passage, it does not answer
Sol. 600 + 10 +15 = ?2 the question, which only asks about the effect of the
? = 25 pandemic on restaurants. The other statements are simply
not mentioned as an issue faced by restaurants in the passage.
S61. Ans.(a)
Sol. 4080 ÷ (? + 17) = 102 + 2 S72. Ans.(d)
4080 Sol. Refer to the following lines in the second passage for
= (? +17)
100 + 2 justification of the answer. “But by the 18th century, capitalism
? = 23 took off… people could show off their wealth.”
The statements given by option (b) and (c) can be
S62. Ans.(a)
corroborated from these lines, but the statement given by
Sol. 31.5 ÷ 3.5 × 12 – 8 = ?2
31.5×12 option (a) cannot find a reference in the passage. Thus, option
− 8 =?2 (d) is the correct answer.
3.5
100 =?2
S73. Ans.(a)
?=10
Sol. Refer to the following lines in the second paragraph for
S63. Ans.(a) justification of option (a) as answer.
Sol.
945
× 20 =? “People have long feasted outside the home… to eat for most
35
of human history.” Options (b) and (c) find no mention in the
? = 540
passage as reasons for choosing restaurants over cooking
S64. Ans.(e) during the time, and thus, are eliminated.
60 85
Sol. 100 × 700 − 450 + 100 × 120 =? S74. Ans.(d)
420 − 450 + 102 =? Sol. Refer to the entirety of the third paragraph for
? = 72 justification of option (d) as the answer. None of the other
options give a suitable answer to the question.
S65. Ans.(e)
Sol. 30 ×
12
+ 12× 20 = ? S75. Ans.(b)
5
? = 312 Sol. Refer to the following lines in the fourth paragraph for
justification of option (b).
S66. Ans.(a) “One of them is immigration. Over the last century the net flow
4 1
Sol. ? = (2 + 4 − 3) + (7 + 3 − 3 + 21)
2 16
of migrants… the quality of an area’s restaurants.”
?= 3 +1 S76. Ans.(d)
?= 4 Sol. Refer to the following lines in the fourth paragraph for
justification of option (d).
S67. Ans.(e)
“Another factor was changing working patterns. Historically
Sol. 8.1 + 2.9=? +11
poor people… since they had the least free time”
?=0
S68. Ans.(a)
26 1
Sol. 100
× 50 × 5 × 26 =?
? = 2.5
S69. Ans.(a)
204
Sol. 12 × 17 =? +81
289 =? +81
? = 208
S70. Ans.(a)
26
Sol. 100 × 50 + 169 =?
13 + 169 =?
? = 182

13 adda247.com/teachers | www.sscadda.com | www.bankersadda.com | www.adda247.com


S77. Ans.(d) Amiable: having or displaying a friendly and pleasant manner.
Sol. cope – to deal with and attempt to overcome problems Dormant: having normal physical functions suspended or
and difficulties slowed down for a period of time
close – to move so as to bar passage through something
S84. Ans.(c)
make – to bring into being by forming, shaping, or altering
Sol. The correct word for the blank is ‘appealing’
material
move – to proceed toward a certain state or condition Valiant: possessing or showing courage or determination.
Vigorous: strong, healthy, and full of energy.
live – to maintain oneself
Appealing: attractive or interesting
S78. Ans.(a) Robust: strong and healthy; vigorous.
Sol. flow – a smooth uninterrupted movement or progress Diffusing: spread over a wide area or between a large number
movement – the act or process of moving of people.
implication – something implied: such as
obstruction – the state of being obstructed S85. Ans.(b)
deduction – the state or fact of being destroyed Sol. The correct word for the given blank is ‘gauge’
increase – made or become greater Ramble: walk for pleasure in the countryside.
Gauge: an instrument that measures and gives a visual display
S79. Ans.(d) of the amount, level, or contents of something.
Sol. upshot – the final result Transit: an act, process, or example of passing through or over
reminder – something that calls a memory or thought to the Flush: become red and hot, typically as the result of illness or
mind strong emotion.
history – an established record Rejig: organize (something) differently; rearrange.
requirement – something wanted or needed
consequence – a result or effect S86. Ans.(a)
condition – a premise upon which the fulfilment of an Sol. In part (A), ‘receive’ should be replaced with ‘received’ as
agreement depends we use third form of the verb after ‘has/have/had’.

S80. Ans.(b) S87. Ans.(c)


Sol. The correct replacement for the highlighted phrase is (b) Sol. In part (C), helping verbs ‘are’ should be replaced with ‘is’
The given sentence is in past tense, thus ‘was’ should be the for noun ‘luminosity’.
correct helping verb. “Angry with” is the correct phrasal verb
S88. Ans.(a)
as when directed at a person "angry with" should always be
Sol. In part (A), ‘been’ should be replaced with ‘be’ because
used. If directed at a situation or thing, "angry at" would also
‘can’ is a model verb which is always followed by infinitive.
be acceptable.
S89. Ans.(b)
S81. Ans.(c)
Sol. In part (B), ‘place’ should be in plural form because
Sol. The correct replacement for the given phrase is option (c)
phrase ‘one of’ is always followed by plural noun.
Here, ‘consumers’ is plural thus plural verb ‘have’ should be
used. Also, the structure for present perfect tense is S90. Ans.(d)
‘has/have+ V3’. Sol. In part (D), ‘commons’ should be replaced with ‘common’
S82. Ans.(d) because we need an adjective here which is always used in a
Sol. The correct word that can fit into the given blank is singular form.
‘resilient’. Commons: the cultural and natural resources accessible to all
Lethargy: a lack of energy and enthusiasm. members of a society
Gloomy: causing or feeling depression or despondency. S91. Ans.(e)
Desperate: feeling or showing a hopeless sense that a Sol. The passage is mentioning the importance of sleep.
situation is so bad as to be impossible to deal with. Therefore, statement (C) will be the first statement of the
Resilient: able to withstand or recover quickly from difficult paragraph as it introduces the theme and is an independent
conditions. statement. Next will be statement (E), which mentions the
Pervasive: spreading widely throughout an area or a group of benefits of proper sleep. Statement (E) then will be followed
people. by statement (A), which mentions that insufficient sleep has
S83. Ans.(a) serious repercussions. The fourth sentence of the paragraph
Sol. The correct word that can fit into the given blank is will be statement (D), which mentions the consequences of
‘incessant’ improper sleep. In statement (B), the word ‘also’, states that it
Incessant: continuing without pause or interruption will follow statement (D) and mentions further side-effects of
Irreversible: not able to be undone or altered. improper sleep. Hence the correct sequence of the statement
Indecisive: not providing a clear and definite result. will be ‘CEADB’.

14 adda247.com/teachers | www.sscadda.com | www.bankersadda.com | www.adda247.com


S92. Ans.(b)
S95. Ans.(d)
Sol. The passage is mentioning the importance of sleep.
Sol. The passage is mentioning the importance of sleep.
Therefore, statement (C) will be the first statement of the
Therefore, statement (C) will be the first statement of the
paragraph as it introduces the theme and is an independent
paragraph as it introduces the theme and is an independent
statement. Next will be statement (E), which mentions the
statement. Next will be statement (E), which mentions the
benefits of proper sleep. Statement (E) then will be followed
benefits of proper sleep. Statement (E) then will be followed
by statement (A), which mentions that insufficient sleep has
by statement (A), which mentions that insufficient sleep has
serious repercussions. The fourth sentence of the paragraph
serious repercussions. The fourth sentence of the paragraph
will be statement (D), which mentions the consequences of
will be statement (D), which mentions the consequences of
improper sleep. In statement (B), the word ‘also’, states that it
improper sleep. In statement (B), the word ‘also’, states that it
will follow statement (D) and mentions further side-effects of
will follow statement (D) and mentions further side-effects of
improper sleep. Hence the correct sequence of the statement
improper sleep. Hence the correct sequence of the statement
will be ‘CEADB’.
will be ‘CEADB’.
S93. Ans.(a)
S96. Ans.(b)
Sol. The passage is mentioning the importance of sleep.
Sol. The correct rearrangement of the highlighted words is
Therefore, statement (C) will be the first statement of the
CBA.
paragraph as it introduces the theme and is an independent
Thus, the correct sentence will be, “A sculpture of an eagle was
statement. Next will be statement (E), which mentions the
carved out of wood for the wildlife event.”
benefits of proper sleep. Statement (E) then will be followed
by statement (A), which mentions that insufficient sleep has S97. Ans.(e)
serious repercussions. The fourth sentence of the paragraph Sol. All the highlighted words in the given sentence are
will be statement (D), which mentions the consequences of correctly placed. Hence, no rearrangement needed.
improper sleep. In statement (B), the word ‘also’, states that it
S98. Ans.(c)
will follow statement (D) and mentions further side-effects of
Sol. The correct rearrangement of the highlighted words is
improper sleep. Hence the correct sequence of the statement
ACB
will be ‘CEADB’.
Thus, the correct sentence will be, “Without any solitude, an
S94. Ans.(c) introvert is bound to go a bit crazy.”
Sol. The passage is mentioning the importance of sleep.
S99. Ans.(d)
Therefore, statement (C) will be the first statement of the
Sol. The correct rearrangement of the highlighted words is
paragraph as it introduces the theme and is an independent
BAC
statement. Next will be statement (E), which mentions the
Thus, the correct sentence will be, ‘The permeable material
benefits of proper sleep. Statement (E) then will be followed
allowed a large amount of water to seep through.”
by statement (A), which mentions that insufficient sleep has
serious repercussions. The fourth sentence of the paragraph S100. Ans.(a)
will be statement (D), which mentions the consequences of Sol. The correct rearrangement of the highlighted words is
improper sleep. In statement (B), the word ‘also’, states that it BCA
will follow statement (D) and mentions further side-effects of Thus, the correct sentence will be, “The astronomer
improper sleep. Hence the correct sequence of the statement discovered a new star during a recent observation”
will be ‘CEADB’.

15 adda247.com/teachers | www.sscadda.com | www.bankersadda.com | www.adda247.com


Part - 2
Directions (1-5): Read each sentence to find out whether Q5. Sometimes very terrible (A)/ accidents happens, (B)/ and
there is any grammatical or idiomatic error in it. The error, if many people are burned (C)/ and injured. (D)
any, will be in one part of the sentence. The number of that (a) D
part is the answer. If there is no error the answer is (e). (b) C
(Ignore errors of punctuation, if any.) (c) B
(d) A
Q1. Despite of the rising share of (A) / the Delta variant, the
(e) No Error
overall incidence in (B)/ Germany has been steadily (C)/
declining in recent weeks (D)/ Directions (6-10): In each of the questions given below a
(a) C sentence is given with three words in bold. Choose the option
(b) B which gives the correct sequence of these words to make the
(c) A sentence grammatically and contextually correct.
(d) D
Q6. His culture (A) manners and varied pleasant(B), not less
(e) No Error
than his artistic skill, contributed(C) to render him popular.
Q2. The recently discovery of hundreds of graves (A)/ in (a) ACB
Canada has put the spotlight back (B)/ on its dark past, when (b) BAC
indigenous people (C)/ faced systemic discrimination and (c) CAB
violence (D)/ (d) BCA
(a) A (e) No rearrangement required
(b) B
Q7. It was fairly (A) detrimental (B) that his prison stay,
(c) C
especially during the pandemic, was obvious (C) to his well-
(d) D
being
(e) No Error
(a) ACB
Q3. I would've gotten (A)/ a perfect score(B)/ accept I made (b) BAC
(C)/ simple mistakes. (D) (c) CAB
(a) D (d) BCA
(b) C (e) No rearrangement required
(c) B
Q8. To make matters worse, credible reports that some of the
(d) A electronic evidence malware (A) in this case could have been
(e) No Error
planted (B) remotely by gathered (C) were never
Q4. He lifted the nest (A)/ gently and put it in a (B)/ safety investigated.
place in the (C)/ forks of the tree. (D)/ (a) ACB
(a) A (b) BAC
(b) D (c) CAB
(c) C (d) CBA
(d) B (e) No rearrangement required
(e) No Error Q9. We must understand mental health conditions (A) for
what they are and for how they are disadvantages (B) with
associated (C).
(a) ACB
(b) BAC
(c) CAB
(d) BCA
(e) No rearrangement required
Q10. Aviation websites are infrastructure (A) with reports
of a rapid abuzz (B) of aviation upgradation (C) in Tibet.
(a) ACB
(b) BAC
(c) CAB
(d) BCA
(e) No rearrangement required

16 adda247.com/teachers | www.sscadda.com | www.bankersadda.com | www.adda247.com


Directions (11-18): Given below is a paragraph that has Q16. (a) absolute
blank spaces. Corresponding to each blank, five options are (b) desperate
given, out of which only one is appropriate. Choose the option (c) disability
that fits most suitably in the given blank making sentence (d) emotional
grammatically and contextually correct. (e) chronic
Q17. (a) retrieval
Getting a good night’s sleep is just as essential as a healthy diet
(b) estimate
and ________(11)______ exercise. Unfortunately, according to the
(c) optimize
National Sleep Foundation, 62% of the American adult (d) decline
population________(12)______ from sleep issues every couple of (e) degrade
nights per week.
Sleep deprivation negatively ________(13)______ the quality of Q18. (a) lower
(b) corrupt
your life, as well as your overall health. In fact, multiple
(c) aggregate
studies show that lack of ________(14)______ sleep can
(d) improve
negatively impact your hormones, exercise performance, (e) impair
brain functioning, and energy levels.
In addition, it can ________(15)______ to moodiness, memory Directions (19-23): In the following question, sentences are
issues, and trouble thinking properly and focusing. Worse of given with a part in bold. The given phrase in the bold may or
may not contain an error. The options following can replace
all, ________(16)______ sleep can cause high blood pressure,
the incorrect phrase. The correct phrase that is to be replaced
weight gain, and a weak immune system. In-depth studies also
will be your answer. If the sentences are correct then select
reveal that chronic poor sleepers are at a higher risk of ‘No improvement required’ as your answer.
developing cancer and dying from it.
If you’re planning to ________(17)______ your health, Q19. The petition contended that the establishments and
________(18)______ the quality of your sleep, and lose weight, factories were operates in violation of environmental
norms and affected public health.
you must get a good night’s sleep.
(a) has been operating in violation of
Q11. (a) regular (b) were operating in violation of
(b) system (c) were operates in violation on
(c) seldom (d) was operating in violation of
(d) active (e) No improvement required
(e) unusual Q20. The Delhi Commission for Women (DCW) was issued a
notice to the police regarding the same and sought a detailed
Q12. (a) address
action taken report.
(b) indicate (a) had issued a notice to the police
(c) results (b) have issue a notice to the police
(d) suffers (c) has been issuing a notice to the police
(e) remarks (d) has issue a notice to the police
(e) No improvement required
Q13. (a) affect
(b) effect Q21. Ravi was sent to Taloja Jail, where some of his co-
(c) impacts accused have now spend more than three years without
(d) observes bail or trial.
(e) facilitate (a) have now spent more than three year
(b) have now spend more then three years
Q14. (a) additional (c) have now spent more then three years
(b) adequate (d) have now spent more than three years
(c) advanced (e) No improvement required
(d) anticipated Q22. With technology becoming smarter, there has been a
(e) barely corresponding increase in cyber fraud, harassments and
threats.
Q15. (a) lead
(a) On technology becoming smarter
(b) indicated
(b) At technology becoming smarter
(c) consist (c) On technology became smarter
(d) transformed (d) With technology has become smarter
(e) caused (e) No improvement required

17 adda247.com/teachers | www.sscadda.com | www.bankersadda.com | www.adda247.com


Q23. A good coach had the skills of his entire team accounted Q24. Choose the statement(s) which is/are TRUE based on
for, and knows where to put them for the best results. the information given in the passage.
(a) have the skills (I) Our systems are built in a way that only allows us to scream
(b) have a skill in dangerous or threatful situations.
(c) has the skills (II) One of the ways humans tackle fear is by going into a
(d) had skills freeze mode and acting dead which is a similar action to
(e)No improvement required
animals.
Directions (24-30): Read the following passage and answer (III) The brain sorts the noises we hear regularly on the basis
the given questions below it with the help of the information of factors such as gender or high or low tone, age, etc.
provided in it. (a) Both II and III
Screaming is a unique noise. Research suggests when we (b) Both II and I
scream in fear, the noise serves the dual purpose of (c) Only III
sharpening our own focus in the face of a threat as well as of (d) Both I and III
warning others. In fact, our brains process screams in a unique (e) Only I
way. Most noises we hear are delivered from our ear to an area
Q25. According to the article, what contributes to alerting
of the brain devoted to analyzing a sound and breaking it into
its component parts, such as gender, age, and tone; high or someone of the danger a person is in?
low; a brass, stringed or electric instrument — or several (a) The gestures and expressions we make when we are
combined; a ribbit or a coo, and what animal makes that confronted with a seemingly dangerous situation.
sound. This indicates that we differentiate all the noises we (b) The scream one lets out when faced with danger holds an
hear everyday, even subconsciously. element of roughness which is crucial for letting others know
A scream is different though, according to David Poepple, PhD, of the possible danger.
a professor of psychology and neural science at New York (c) Secretion of certain fluids and juices in our bodies by our
University. Poepple is the lead author of a 2015 study that organs play a part in accomplishing the above said thing.
looked into what happens in the body when people scream in (d) The frequency of our screams, i.e., how often we shout in
fear. A scream goes straight from the ear to the amygdala, the the given amount of time can alert others of the presence of
part of the brain that processes fear and kickstarts the body’s danger.
fight-or-flight response. The sound jolts our brains into
(e) None of the above
increased alertness and analysis. This is especially true for
screams when we’re scared. These have a unique sound Q26. Among the following statements, choose option(s)
signature encompassed by the term “roughness.” This which are FALSE in accordance with the information given in
roughness is what serves to alert others to danger. Humans the passage.
and other animals scream for many reasons — in joy, in (I) The part of the brain that understands the fear of the
surprise, in fear. But we’ve evolved to be able to produce situation and sends us into a full blown fight/flight response
different shrieks and to be able to discern the difference when
is called amygdala.
we hear them. (Incidentally, roughness is also why we find the
(II) The number of muscles used in the process of speaking
sound of a baby crying and fingernails on chalkboards
are over 100.
disturbing; both sounds have high roughness.) If we focus on
the sounds that gain our attention, we will be able to tell which (III) The purpose of a scream is to vent out your anger and
one is rough and which is not. release the emotions a person has been withholding inside.
But what about people who have the opposite reaction when (a) Both I and II
they’re scared? What about the people who don’t scream in (b) Only III
fear, but become speechless in the face of a threat? This has (c) Only II
been less studied, but it’s likely a part of the third, overlooked (d) Only I
response to terror: paralysis. Contrary to pop medicine, fight (e) Both II and III
and flight aren’t the only two reactions to something scary.
Humans, like other animals, also evolved the option to freeze, Q27. Choose the option which states the reason for why some
or “play dead” in the face of a threat. This happens people go mute when they are scared.
involuntarily, when the cerebellum, the part of the brain that (a) The brain finds it hard to concentrate between how to
regulates muscle activity, overpowers other terrified brain combat the dangerous situation and figuring out how to talk
responses and basically shuts down all movement. Since which leaves a person mute.
screaming takes quite a few muscles (probably a little less (b) Some people find that they don't want to express their
than the 100 used for speaking), a person frozen in fear is feelings of terror and fright which is why they choose to
rendered mute as well as immobile. remain quiet in those moments.
18 adda247.com/teachers | www.sscadda.com | www.bankersadda.com | www.adda247.com
(c) A certain part of the brain responsible for overlooking Q31. No. of vacant flats in building C and D together are
muscle activity takes over other responses of the brain and approximate what percentage of no. of occupied flats in
forces every movement of the body to shut down, including building D and E together.
speech. (a) 60%
(d) Being quiet has been found to help do away with the (b) 84%
dangerous and harmful factors that a person might be facing (c) 67%
at that exact moment. (d) 55%
(e) None of the above (e) 80%
Q28. Among all the three responses to fear, how does a scream Q32. What is the respective ratio of occupied flats in building
help us in possible dangerous situations? A and E together to vacant flats in C and D together?
(a) The brain processes the scream in a part which helps (a) 23 : 20
activate our fight or flight mode which then helps us become (b) 17 : 19
aware and scan the situation. (c) 5 : 4
(b) Scream can notify the people in your surroundings like (d) 11 : 17
your neighbours or anybody in close vicinity to you. (e) 15 : 23
(c) When you scream, your vocal cords stretch and become
flexible enough to scream louder and warn others of the Q33. Average no. of vacant flats in building A and E is what
situation. percentage of occupied flats in building C.
(d) None of the above (a) 25%
(e) All of the above (b) 50%
(c) 75%
Q29. Select the word which is similar in meaning to the (d) 60%
highlighted word INDICATES. (e) 40%
(a) announces
(b) signals Q34. Total flats in building B and D together are how much
(c) bargains more or less than that in building A and C together.
(d) divides (a) 160
(e) flows (b) 180
(c) 140
Q30. Choose the phrase which will replace the phrase ‘focus (d) 120
on’ as highlighted in the passage. (e) 100
(a) drain out
(b) tear apart Q35. If 3 people live in each occupied flat of building A and 2
(c) fire away people live in each occupied flat of building B, then find total
(d) concentrate on no. of people living in building A and B.
(e) mix into (a) 1000
(b) 1600
Direction (31-35): Read the data given below and answer the
(c) 1400
following questions.
(d) 1200
The bar graph given below shows the total no. of flats and no.
(e) 1800
of vacant flats in five buildings.

700
600
500
400
300
200
100
0
A B C D E
Total Flats Vacant Flats

Note – Total Flats = Vacant Flats + Occupied flats

19 adda247.com/teachers | www.sscadda.com | www.bankersadda.com | www.adda247.com


Directions (36-40): What will come in the place of the Q42. No. of students in group X of class B are what percentage
question mark (?) in the following number series? of no. of students in groups Y of class C.
(a) 160%
Q36. 0.2, 2.7, 7.7, 15.2, ?, 37.7 (b) 80%
(a) 24.2 (c) 100%
(b) 30.2 (d) 150%
(c) 25.2 (e) 75%
(d) 21.2
Q43. There is total 34 girls in class A. If 12 girls are in group
(e) 27.2
X, then find the no. of boys in group Y and Z together in class
Q37. 15, 60, 210, 630, 1575, ? A.
(a) 2450 (a) 38
(b) 2700 (b) 34
(C) 2550 (c) 30
(d) 3150 (d) 32
(e) 3250 (e) 36
Q44. Total no. of students in group X is how much more or less
Q38. 45, 49, 40, 65, 16, ?
than that in group Z.
(a) 137
(a) 20
(b) 121 (b) 25
(c) 97 (c) 15
(d) 143 (d) 50
(e) 109 (e) 35
Q39. 4, 9, 28, ?, 566, 3397 Q45. Find the respective ratio between average students in
(a) 123 group X in class A and B to total students in class C.
(b) 113 (a) 12: 17
(c) 177 (b) 7: 23
(d) 141 (c) 4: 21
(e) 219 (d) 7: 12
(e) 11: 16
Q40. 22, 30, 41, 56, ?, 102
(a) 74 Q46. Ratio between length of trains A and B is 3 : 5. Speed of
train A is 72 km/h and that of train B is 54 km/h & they are
(b) 76
running opposite to each other. If train A crosses train B in 16
(c) 72
seconds, then find length of train B.
(d) 84 (a) 350 m
(e) 86 (b) 250 m
Direction (41-45): the data given below shows the (c) 450 m
information of total students in class A, B and C which are (d) 150 m
divided in three groups X, Y and Z. (e) 320 m
Class A: Total no. of students are 90. 30 students are in group Q47. The perimeter of circle of radius 28 cm is 32 cm more
X. ratio of number of students in group Y and Z is 1: 2. than perimeter of square. Find the area of the square.
Class B: Total no. of students are 80. The ratio of no. of (a) 1225
students in group X, Y and Z is 8: 5: 3. (b) 1296
Class C: the no. of students in group Y are half of that in group (c) 1156
X. The no of students in group X of class C are 20 more than (d) 1089
that in group X of class A. no. of students in group Z of class C (e) 1444
are 20% less than that in group X of same class. Q48. A boat in upstream can travel 5 km in 30 minutes. If the
speed of the stream is 1.5 kmph, then find the total distance
Q41. What is the total no. of students in group Y of all the
cover by the boat when boat takes 46 hours to travel from A
classes?
to b then return to B.
(a) 95 (a) 260 km
(b) 70 (b) 220 km
(c) 120 (c) 280 km
(d) 80 (d) 520 km
(e) 60 (e) 144 km

20 adda247.com/teachers | www.sscadda.com | www.bankersadda.com | www.adda247.com


Q49. Difference of the interest received when a sum is Q55. Average of present age of A & B is 30 years and C is four
invested at 15% p.a. at SI for two years and the interest years older than A. Four years ago ratio of age of A to C was 3
received when that sum is invested at 20% p.a. for one year : 4. Find the sum of ages of A, B & C five years hence will be?
compounded half yearly is Rs 432, find the sum? (a) 85 years
(a) Rs. 5400 (b) 95 years
(b) Rs. 5000 (c) 90 years
(c) Rs. 4500 (d) 105 years
(d) Rs. 4000 (e) 75 years
(e) Rs. 4800
Directions (56-65): - What will come in place of (?) question
Q50. In an election only two candidates participate. Candidate mark in the following questions.
‘P’ got 50% less votes than ‘Q’. Had Q got 200 votes less there
would have been a tie. What is the 8 times of the number of Q56. 512 ÷? × 6= 192
total votes polled. (a) 11
(a) 800 (b) 18
(b) 7200 (c) 12
(c) 3200 (d) 16
(d) 9600 (e) None of these
(e) 3600 Q57. (48% of 625) ÷ 0.75 = ?
Q51. A pen was marked up by 50% above cost price and (a) 800
allowed Rs 100 discount on marked price. If shopkeeper still (b) None of these
made a profit of Rs. 100, then find the selling price of the pen (c) 40
(in Rs.)? (d) 4000
(a) 450 Rs. (e) 400
(b) 600 Rs. Q58. 65 % of 480 - ? +175 =350
(c) 500 Rs. (a) 125
(d) 400 Rs. (b) 129
(e) 350 Rs. (c) 137
Q52. A container contains total 120 liters mixture of milk and (d) 147
water in which quantity of water is 20 liters. If 36 liters (e) 153
mixture taken out from container and 6 liters milk added, then 460
Q59. 45 + 20 % of ? =
find quantity of water in what percent (approximate) less than 4
that of milk in final mixture? (a) 250
(a) 82% (b) 450
(b) 88% (c) 400
(c) 78% (d) 350
(c) 86% (e) 360
(e) 90% (16)2
Q60. (4)? × 2 = 4
√16
Q53. A and B starts a partnership business by investing Rs. (a) 2
35000 & Rs. 40000 respectively. After six months C joined (b) 3
them and at the end of year profit share of C is Rs. 9000 out of (c) 4
total profit of Rs. 39000. Find the investment of C? (d) 1
(a) 54000 Rs. (e) None of these
(b) 36000 Rs.
(c) 60000 Rs. Q61. 4 × (? +120) = (8)3
(d) 45000 Rs. (a) 6
(e) 40000 Rs. (b) 12
(c) 8
Q54. Pipe A & pipe B together can fill half of a tank in 7 hours,
(d) 4
while pipe C can empty 40% of the same tank x hours. When
(e) 16
all three pipes open together in same empty tank, then it
1 56
completely filled in 26 4 hours. Find value of x? Q62. ?
+ (8)3 = 13% of 4000
(a) 18 (a) 7
(b) 12 (b) 8
(c) 16 (c) 14
(d) 24 (d) 28
(e) 9 (e) None of these

21 adda247.com/teachers | www.sscadda.com | www.bankersadda.com | www.adda247.com


Q63. 4
3 3 3
+ 1 – ? = √64 Q70. Which of the following statement is true about H?
5
12 4
(a) H sits second to the left of F
(a) 2 (b) H faces B
(b) 1
1
(c) D sits third to the right of H
2
(d) E is not an immediate neighbor of H
(c) 1
(e) Both (b) and (d)
(d) 4
(e) 2 Directions (71-73): In each of the questions below are given
some statements followed by some conclusions. You have to
Q64. 756 ÷ 14 × 5 –√7921=? take the given statements to be true even if they seem to be at
(a) 183 variance with commonly known facts. Read all the
(b) 179 conclusions and then decide which of the given conclusions
(c) 178 logically follows from the given statements, disregarding
(d) 181 commonly known facts. Give answer
(e) 187 Q71. Statements:
Q65. 462 ÷ 5.25 + 24×12 =? All Cloud are Rain.
(a) 372 Only a few Cloud are Water.
(b) 390 Conclusions:
(c) 376 I. Some Water is Rain
II. All Rain is Water
(d) 370
(a) Both I and II follow
(e) 400
(b) Either I or II follows
Directions (66-70): Study the following information to (c) Only II follows.
answer the given questions: (d) Only I follows.
(e) Neither I nor II follows
Eight persons sit around a circular table. All of them face
inside. F sits third to the right of E. Only one person sits Q72. Statements:
between H and F. A sits opposite to D. C sits third to the left of Only a few Apple is Mango.
G. C is not an immediate neighbor of B. D is not an immediate Some Mango is Guava.
Conclusions:
neighbor of H. G is an immediate neighbour of E.
I. All Guava is Apple
Q66. Who among the following sits opposite to G? II. Some Guava is not Apple
(a) H (a) Both I and II follow
(b) B (b) Either I or II follows
(c) D (c) Only II follows.
(d) F (d) Only I follows.
(e) None of these (e) Neither I nor II follows
Q73. Statements:
Q67. Who among the following sits second to the left of the
All Green is Blue.
one who sits third to the right of H?
All Blue is Yellow.
(a) B No Blue is Pink.
(b) D Conclusions:
(c) A I. No Green is Pink
(d) E II. Some Yellow is not pink
(e) None of these (a) Both I and II follow
Q68. What is the position of C with respect to D? (b) Either I or II follows
(c) Only II follows.
(a) Third to the right
(d) Only I follows.
(b) Second to the right
(e) Neither I nor II follows
(c) Immediate left
(d) Second to the left Q74. How many pairs of letters are there in the word
(e) None of these ‘CONDITION’, each of which have as many letters between
them (both forward and backward direction) in the word as
Q69. Who among the following person sit third to the left of they have between them according to English alphabetical
A? order?
(a) F (a) None
(b) B (b) Two
(c) G (c) Four
(d) C (d) Three
(e) None of these (e) None of these

22 adda247.com/teachers | www.sscadda.com | www.bankersadda.com | www.adda247.com


Directions (75-79): Study the following arrangement Directions (81-85): Study the following information
carefully and answer the questions given below. carefully and answer the questions given below.
Eight persons A, B, C, D, E, F, G and H sit in a row facing to the
@EK&YR©CAM®UL$VQ#P%SIW*OTɸG
north direction but not necessarily in the same order.
Q75. If all consonants are replaced its next letter, then how B sits 3rd from the right end. Three persons sit between H and
many alphabets are repeated in the new arrangement? B. D is the only neighbour of F. E sits immediate left of G, who
(a) Four sits 2nd to the right of A.
(b) One
(c) Three Q81. Who among the following sits 2nd to the left of F?
(d) Two (a) B
(e) None (b) H
(c) A
Q76. How many such symbols are there in the above (d) D
arrangement each of which is immediately preceded by a (e) None of these
consonant but not immediately followed by a Vowel?
(a) None Q82. Which of the following statement is true as per the given
(b) Four information?
(c) Two (a) Two persons sit between E and D
(d) Three (b) A sits immediate right of C
(e) More than Four (c) C sits at one of the extreme ends
(d) Both (a) and (c)
Q77. Four of the following five are alike in a certain way based
on their position in the above arrangement and so form a (e) All are true
group. Which is the one that does not belong to that group? Q83. Four of the following five are alike in a certain way and
(a) ER& so form a group. Find the one who does not belong to that
(b) CUM group?
(c) L#V (a) C
(d) PW% (b) H
(e) SOW (c) G
Q78. How many such alphabets are there in the above (d) A
arrangement each of which is immediately preceded and (e) B
immediately followed by a symbol?
Q84. Who among the following are immediate neighbours of
(a) None
G?
(b) One
(a) F
(c) Two
(b) C
(d) Three
(c) B
(e) More than three
(d) E
Q79. Which of the following element is 10th to the right of the (e) Both (c) and (d)
element which is 18th from the right end?
(a) # Q85. What is the position of D with respect to G?
(b) P (a) 3rd to the left
(c) % (b) Immediate left
(d) S (c) 2nd to the right
(e) None of these (d) 3rd to the right
(e) None of these
Q80. If we form a meaningful word by the 3rd, 4th, 7th and 8th
letter of the word ‘CONSISTENT”, then which of the following Q86. Five persons A, B, C, D and E arranged in a row. One
will be the first letter of the word thus formed. If more than person sits between A and B. One person sits between A and
one word is formed mark Y as your answer. If no meaningful D. C sits 3rd to the right of D. What is the position of A with
word is formed, mark X as your answer? respect to E?
(a) X (a) Immediate right
(b) N (b) 2nd to the left
(c) Y (c) Immediate left
(d) S (d) 3rd to the right
(e) E (e) None of these

23 adda247.com/teachers | www.sscadda.com | www.bankersadda.com | www.adda247.com


Directions (87-89): Study the following information Q93. Which of the following box is placed immediate below
carefully and answer the questions given below. box S?
Point A is in 8m north of point B. Point C is in 4m east of point (a) V
B. Point D is in 5m south of point C. Point F is in 5m north of
(b) R
Point E, which is 8m east of point D. Point F is in 4m west of
point G. Point H is in 8m north of Point G. (c) P
(d) W
Q87. What is the shortest distance between Point A and Point
(e) None
H?
(a) 12m Directions (94-96): In each of the questions below are given
(b) 14m some statements followed by two conclusions. You have to
(c) 16m
take the given statements to be true even if they seem to be at
(d) 18m
(e) None of these variance with commonly known facts. Read all the conclusions
and then decide which of the given conclusions logically
Q88. What is the direction of Point H with respect to Point D?
follows from the given statements disregarding commonly
(a) North-east
(b) North known facts.
(c) South-east (a) If only conclusion I follows.
(d) North-west (b) If only conclusion II follows.
(e) None of these (c) If either conclusion I or II follows.
Q89. What is the shortest distance between Point F and Point (d) If neither conclusion I nor II follows.
C? (e) If both conclusions I and II follow.
(a) 5m
(b) 8m Q94. Statements: M < N ≤ O < R = S < T
(c) 10m Conclusion I: M = O II: N < S
(d) 12m
(e) None of these Q95. Statements: C ≥ D = E ≤ F < G ≤ H
Conclusion I: C ≥ F II: F > C
Directions (90-93): Study the following information to
answer the given questions: Q96. Statements: S > U ≥ V = W ≤ X; W ≤ Y ≤ Z
Eight boxes are placed one above the other in a stack. Not Conclusion I: S > W II: U ≥ Y
more than two boxes are placed above box Q. Three boxes are
placed between box Q and T. Two boxes are placed between Direction (97-100): Study the following information
box U and S, which is placed below box T. Box U does not place carefully and answer the questions given below.
immediate above box T. Box P is placed immediate above box Ten persons sit in two parallel rows containing 5 persons in
V. Box R is placed above box W but does not place above box each row in such a way that there is an equal distance between
V.
adjacent persons. In the 1st row P, Q, R, S and T sit and all of
Q90. Which of the following box is placed immediate above them face
box W? south. In the 2nd row A, B, C, D and E sit and all of them face
(a) S
north but not necessarily in the same order.
(b) T
(c) Q Two persons sit between E and D, who sits diagonally opposite
(d) U to S. E faces Q, who sits 3rd to the right of R. C faces P but not
(e) None of these an immediate neighbour of D. B sits to the left of A.
Q91. How many boxes are placed between P and R? Q97. What is the position of D with respect to A?
(a) Four (a) Immediate left
(b) One
(c) Two (b) 3rd to the right
(d) Three (c) Immediate right
(e) None of these (d) 2nd to the left
(e) None of these
Q92. The number of boxes are placed between Q and W is
same as the number of boxes are placed between ____ and Q98. Who among the following faces to T?
_____?
(a) A
(a) P, Q
(b) V, U (b) D
(c) R, S (c) B
(d) W, T (d) Either (a) or (c)
(e) None of these (e) None of these

24 adda247.com/teachers | www.sscadda.com | www.bankersadda.com | www.adda247.com


Q99. Four of the following five are alike in a certain way and
so form a group. Find the one who does not belong to that
group?
(a) T
(b) Q
(c) C
(d) B
(e) P
Q100. How many persons sit between Q and T?
(a) One
(b) Three
(c) None
(d) Two
(e) None of these

Solutions
S1. Ans.(c) S8. Ans.(d)
Sol. The correct choice is option (c). In part (A), ‘despite of’ is Sol. The correct sequence is CBA. Therefore the correct
incorrect as ‘despite’ is never followed by ‘of’. sentence will be” To make matters worse, credible reports
that some of the electronic evidence gathered in this case
S2. Ans.(a) could have been planted remotely by malware were never
Sol. The error lies in part (A) of the sentence. Here ‘recently’ investigated”
should be ‘recent’ as we need adjective here while ‘recently’ is
adverb. S9. Ans.(a)
Sol. The correct sequence is ACB. Therefore the correct
S3. Ans.(b) sentence will be “We must understand mental health
Sol. The error lies in part (C). Here ‘accept’ should be ‘except’. conditions for what they are and for how they are associated
Accept is contextually incorrect in the given sentence. with disadvantages”
accept means to agree to take something that somebody offers
S10. Ans.(d)
you
Sol. The correct sequence is BCA. Therefore the correct
except means not including somebody/something; apart from
sentence will be “Aviation websites are abuzz with reports of
the fact that a rapid upgradation of aviation infrastructure in Tibet.”
S4. Ans.(c) S11. Ans.(a)
Sol. The error lies in part (C). Here ‘safety’ should be ‘safe’ as Sol. The paragraph illustrates the importance of sleep just as
we need an adjective here for the noun ‘place’. nutrition and exercise is to maintain a healthy body and mind.
S5. Ans.(c) Therefore, the most suitable word for the given blank is
‘regular’ as it contributes to the theme of the passage about
Sol. The error lies in part (B). Here ‘happens’ should be
addressing healthy sleep routines.
replaced by ‘happen’ as to comply with the rule of subject verb
Regular- recurring at uniform intervals.
agreement, we need plural verb for the plural subject
System- a set of things working together as parts of a
(accidents).
mechanism or an interconnecting network; a complex whole.
S6. Ans.(b) Seldom- not often; rarely
Sol. The correct sequence of words is BAC. Therefore the active- engaging or ready to engage in physically energetic
correct sentence will be “His pleasant manners and varied pursuits.
culture, not less than his artistic skill, contributed to render Unusual- not habitually or commonly occurring or done.
him popular.” S12. Ans.(d)
S7. Ans.(a) Sol. The given sentence contributes to the central theme of the
paragraph which is about healthy sleeping habits by
Sol. The correct sequence is ACB. Therefore the correct
providing a statistical data of adults experiencing sleeping
sentence will be” It was fairly obvious that his prison stay,
issues. Thus, the most suitable word for the given blank is
especially during the pandemic, was detrimental to his well-
‘suffers’. All the other words are either grammatically or
being”
contextually incorrect.
25 adda247.com/teachers | www.sscadda.com | www.bankersadda.com | www.adda247.com
Suffers- experience or be subjected to (something bad or S17. Ans.(c)
unpleasant). Sol. The passage mentions that chronic sleep deprivation
Address- think about and begin to deal with (an issue or leads to all sorts of negative effects on physical and social
problem). well-being. Hence, it is concluded by advising the readers to
Indicate- point out; show. improve the quality of sleep if one wants to achieve a healthy
Results- a favourable outcome of an undertaking or contest. lifestyle. Therefore the most suitable word for the given blank
Remarks- say something as a comment; mention. is ‘optimize’.
S13. Ans.(c) Optimize- make the best or most effective use of (a situation
Sol. The given paragraph mentions the harmful repercussions or resource).
of sleep deprivation. Therefore, the most suitable word for the Retrieval- the process of getting something back from
given blank is ‘impacts.’ somewhere.
Impacts- a marked effect or influence. Estimate- roughly calculate or judge the value, number,
Affect- Affect is usually a verb, and it means to impact or quantity, or extent of.
change. Decline- diminish in strength or quality; deteriorate.
Effect- Effect is usually a noun that you would use to indicate Degrade- lower the quality of; cause to deteriorate.
the result of a change.
Observes- notice or perceive (something) and register it as S18. Ans.(d)
being significant. Sol. The most suitable word for the given blank is ‘improve’
Facilitate- make (an action or process) easy or easier. which means make or become better. All the other words are
contextually incorrect.
S14. Ans.(b) Lower- less high in position.
Sol. The most suitable word for the given blank is adequate Corrupt- evil or morally depraved.
which means ‘satisfactory or acceptable in quality or quantity’
Aggregate- a whole formed by combining several separate
as the passage refers to the harmful impacts on quality of life
elements.
due to deprived sleep. All the other words are contextually
Impair- weaken or damage (something, especially a faculty or
incorrect.
function).
Additional- added, extra, or supplementary to what is already
present or available S19. Ans.(b)
Advanced- modern and recently developed. Sol. The given highlighted phrase is incorrect. The sentence is
Anticipated- regard as probable; expect or predict. in past continuous tense hence ‘operates’ in statement is
Barely- only just; almost not. incorrect hence option (c) is also incorrect. In option (a), ‘has’
S15. Ans.(a) is incorrect because subject (establishments and factories) is
Sol. A Modal verb is followed by another verb in the base form. plural here therefore option (d) is also incorrect. Hence
Since ‘can’ is a modal verb, the main verb should be in its base correct choice is option (b).
form. This shortlists our options to ‘lead’ and ‘consist’. Lead
S20. Ans.(a)
means cause (a person or animal) to go with one by holding
Sol. The highlighted phrase in the statement is incorrect as
them by the hand, a halter, a rope, etc. while moving forward;
sentence is in Past indefinite. In option (b), ‘have’ is incorrect
while consist means be composed or made up of. Hence. ‘lead’
is the most suitable word. as subject (DCW) is singular. Option (c) is in Perfect
Indicated- point out; show. continuous hence incorrect. In Option (d), ‘issue’ is incorrect
Transformed- make a marked change in the form, nature, or as ‘has+v3’ is correct.
appearance of.
Caused- make (something, especially something bad) happen.
S16. Ans.(e)
Sol. The given sentence of the passage mentions the
conditions which further aggravate the condition of sleep
deprivation. Thus, the most suitable word for the given blank
is ‘chronic’ which means (of an illness) persisting for a long
time or constantly recurring.
Absolute- not qualified or diminished in any way; total.
Desperate- (of an act) tried in despair or when everything else
has failed.
Disability- a physical or mental condition that limits a
person's movements, senses, or activities.
Emotional- relating to a person's emotions.

26 adda247.com/teachers | www.sscadda.com | www.bankersadda.com | www.adda247.com


S21. Ans.(d) Statement III is also false because it can be found in the article
Sol. In the highlighted phrase ‘spend’ is incorrect because that: Research suggests when we scream in fear, the noise
sentence is in past tense and third form of spend is ‘spent’ serves the dual purpose of sharpening our own focus in the face
therefore option (b) is also incorrect. In option (a), ‘year’ of a threat as well as of warning others.
should be ‘years’. In option (b), ‘then’ should be ‘than’ hence The facts present in the passage help us figure out why the
option (c) is also incorrect. statements II and III are false.
Hence, option (e) is the correct choice for the answer.
S22. Ans.(e)
Sol. The highlighted phrase in the sentence is correct hence S27. Ans.(c)
no improvement required is correct choice. Sol. The correct choice for the answer is option (c): A certain
part of the brain responsible for overlooking muscle activity
S23. Ans.(c)
takes over other responses of the brain and forces every
Sol. The correct replacement of the highlighted phrase is “has
movement of the body to shut down, including speech.
the skills” as the sentence follows the structure of present
tense. Moreover, the subject of the sentence is singular. Hence, The answer is found towards the end of the last paragraph:
This happens involuntarily, when the cerebellum, the part of the
option (c) is the correct choice.
brain that regulates muscle activity, overpowers other terrified
S24. Ans.(a) brain responses and basically shuts down all movement. Since
Sol. The correct choice for the answer is option (a): Both II and screaming takes quite a few muscles (probably a little less than
III. the 100 used for speaking), a person frozen in fear is rendered
Statement II is true because it is stated in the last paragraph mute as well as immobile.
of the passage that “humans, like other animals, also evolved All the other options are nowhere to be found in the above
the option to freeze, or “play dead” in the face of a threat.” passage.
Statement III is true because in the first paragraph it says: Hence, option (c) is the correct choice for the answer.
Most noises we hear are delivered from our ear to an area of the
brain devoted to analyzing a sound and breaking it into its S28. Ans.(a)
component parts, such as gender, age, and tone; high or low; a Sol. The correct choice for the answer is option (a): The brain
brass, stringed or electric instrument — or several combined; a processes the scream in a part which helps activate our fight or
ribbit or a coo, and what animal makes that sound. flight mode which then helps us become aware and scan the
Statement I is incorrect because in the last paragraph it says situation.
“Humans and other animals scream for many reasons — in joy, This answer is present in the second paragraph: A scream goes
in surprise, in fear. “ This tells us that there can be many straight from the ear to the amygdala, the part of the brain that
reasons for humans screaming besides fear. processes fear and kickstarts the body’s fight-or-flight response.
Hence, option (a) is the correct choice for the answer. The sound jolts our brains into increased alertness and analysis.
Hence, option (a) is the correct choice for the answer.
S25. Ans.(b)
Sol. The correct choice for the answer is option (b): The S29. Ans.(b)
scream one lets out when faced with danger holds an element of Sol. The correct choice for the answer is option (b): signals.
roughness which is crucial for letting others know of the To signal means to convey information or instructions by
possible danger. means of a gesture, action, or sound.
This answer is found in the second paragraph where it says: Indicate means to express an intention, opinion, or wish in an
This is especially true for screams when we’re scared. These indirect way.
have a unique sound signature encompassed by the term Hence, signals is the word most similar to indicates.
“roughness.” This roughness is what serves to alert others to To announce means to make a formal public statement about
danger. a fact, occurrence, or intention.
The rest of the options are not mentioned anywhere in the To bargain means an agreement between two or more people
passage given. Therefore they cannot be considered for the or groups as to what each will do for the other.
answer. To divide means to separate or be separated into parts.
Hence, option (b) is the correct choice for the answer. Flows mean the action or fact of moving along in a steady,
continuous stream.
S26. Ans.(e)
Sol. The correct choice for the answer is option (e): Both II and S30. Ans.(d)
III Sol. The correct choice for the answer is option (d):
Statement II is false as the passage says: Since screaming takes concentrate on.
quite a few muscles (probably a little less than the 100 used Focus on or concentrate on have similar meanings which can
for speaking), a person frozen in fear is rendered mute as well be explained as to give most of your attention to someone or
as immobile. something.
27 adda247.com/teachers | www.sscadda.com | www.bankersadda.com | www.adda247.com
The rest of the phrases are not applicable because they all S39. Ans.(b)
have different meanings. Hence, they cannot be used to Sol. The pattern of the series is –
replace the given phrase. 4×2+1 = 9
9 × 3 + 1 = 28
Solutions (31-35):
28 × 4 + 1 = 113
Building Total Flats Vacant Occupied 113 × 5 + 1 = 566
Flats Flats 566 × 6 + 1 = 3397
A 400 160 240
B 460 120 340 S40. Ans.(b)
Sol. Pattern of series –
C 540 180 360
D 600 220 380
E 420 200 220

S31. Ans.(c) Solutions (41-45):


Sol. Required percentage =
(180+220) 2
× 100 = 66 % ≈ 67% Group X Group Y Group Z Total
(380+220) 3
A 30 20 40 90
S32. Ans.(a) B 40 25 15 80
240+220 460
Sol. Required ratio = 180+220 = 400 C 50 25 40 115
Total 120 70 95 285
= 23 ∶ 20
S33. Ans.(b) S41. Ans.(b)
Sol. Average vacant flats in A and E =
160+200
= 180 Sol. Total students in group Y of all the classes = 70
2
180
Required percentage = 360 × 100 = 50% S42. Ans.(a)
40
Sol. Required percentage = 25 × 100 = 160%
S34. Ans.(d)
Sol. Required Difference = (460 + 600) − (400 + 540) S43. Ans.(a)
= 1060 − 940 = 120 Sol. Total girls in group Y and Z together = 34 − 12 = 22
Total boys in group Y and Z together on class A
S35. Ans.(c) = (90 − 30) − 22 = 38
Sol. Required no. = 3 × 240 + 2 × 340 = 720 + 680
= 1400 S44. Ans.(b)
Sol. Required difference = 120 − 95 = 25
S36. Ans.(c)
Sol. The pattern of the series is – S45. Ans.(b)
40+30
0.2 + 2.5 × 1 = 2.7 Sol. Average students in group X of class and B = 2 = 35
2.7 + 2.5 × 2 = 7.7 35 7
Required ratio = 115 = 23
7.7 + 2.5 × 3 = 15.2
= 750m
15.2 + 2.5 × 4 = 25.2
25.2 + 2.5 × 5 = 37.7 S46. Ans.(a)
Sol. Let length of train A = 3x
S37. Ans.(d) Length of train B = 5x
Sol. The pattern of the series is – 5
Speed of train A = 72 × = 20 m/sec
15 × 4 = 60 18
5
60 × 3.5 = 210 Speed of train B = 54 × = 15 m/sec
18
210 × 3 = 630 ATQ,
8𝑥
630 × 2.5 = 1575 = 16
20+15
1575 × 2 = 3150 ⇒ x = 70
S38. Ans.(a) ∴ Length of train B = 5 × 70 = 350 m
Sol. The pattern of the series is – S47. Ans.(b)
45 + 22 = 49 22
Sol. Perimeter of the circle = 2 × 7 × 28 = 176 𝑐𝑚
49 − 32 = 40
Perimeter of square = 176 − 32 = 144
40 + 52 = 65 144
65 − 72 = 16 Side of square = 4 = 36
16 + 112 = 137 So, area of square = 1296 𝑐𝑚2

28 adda247.com/teachers | www.sscadda.com | www.bankersadda.com | www.adda247.com


S48. Ans.(d) S55. Ans.(b)
Sol. Let speed of boat be x kmph Sol. Total present age of A and B = 30 × 2 = 60 𝑦𝑒𝑎𝑟𝑠
ATQ Let four year ago age of A and C be 3x and 4x respectively
𝑥 − 1.5 = 10 ATQ –
𝑥 = 11.5 (4x + 4) – (3x + 4) = 4
Let the distance be D. x = 4 years
𝐷 𝐷
13
+ 10 = 46 Present age of C = (4 × 4 + 4) = 20 years
𝐷 = 260 𝑘𝑚 So, sum of ages of A, B & C five years hence will be = 60 + 20 +
Total distance travel by boat = 2 × 260 = 520 5 × 3 = 95 𝑦𝑒𝑎𝑟𝑠
S49. Ans.(e) S56. Ans.(d)
Sol. Let the amount be Rs x Sol. 512÷?× 6= 192
ATQ ?=
512×6
192
𝑥×30100−𝑥×21100=432
= 16
𝑥=𝑅𝑠 4800
S57. Ans.(e)
S50. Ans.(d) 48 4
Sol. Q got = 100x votes Sol. 100 × 625 × 3 =?
So, P got = 50x votes ? = 400
Total votes = 150x votes
For tie → Q’s votes = P’s votes = 75x S58. Ans.(c)
⇒ 100x – 75x = 200 Sol. 65% × 480−? +175 = 350
⇒x=8 ? = 137
Total votes → 8 × 150 = 1200 S59. Ans.(d)
Required number = 1200 × 8 = 9600 20 460
Sol. 45 + 100 × ? =
4
S51. Ans.(c) ?
45 + 5 = 115
Sol. Let cost price of pen = 100x Rs.
50 ? = 350
So, marked price of pen = 100x × (1 + 100) = 150𝑥 Rs.
And, selling price of pen = (150x – 100) Rs. S50. Ans.(b)
162
ATQ – Sol. 4? × 2 = 4
√16
(150x – 100) – 100x = 100 256
50x = 200 4? × 2 = 2
x = 4 Rs. 4? = 64
So, selling price of pen = (150 × 4 − 100) = 500 𝑅𝑠. 4? = (4)3
?=3
S52. Ans.(a)
Sol. Ratio of milk to water in mixture = (120 – 20) : 20 = 5 : 1 S61. Ans.(c)
1
Total water in final mixture = 20 - 36 × = 14 liters Sol. 4 × (? +120) = (8)3
6
1 4 × ? = 512 − 480
Total milk in final mixture = 100 - 36 × + 6 = 76 liters 32
6
76−14 ?=
Required percentage = × 100 = 81.67 = 82% 4
76 ?=8
S53. Ans.(d)
S62. Ans.(a)
Sol. Let investment of C = X Rs. 56 13
ATQ – Sol. + 512 = × 4000
? 100
𝑋×6 9000 56
(35000 ×12+40000×12+𝑋×6)
= 39000 ?
= 520 − 512
X = 45000 Rs. ?=7
S54. Ans.(b) S63. Ans.(e)
Sol. Pipe (A +B) together fill the whole tank = 7 × 2 = 51 7
Sol. 12 + 4 − ? = 4
14 ℎ𝑜𝑢𝑟𝑠 51+21
100 5𝑥 ?= −4
Pipe C empty the whole tank in = 𝑥 × = hours 12
40 2
?=6–4=2
ATQ –
1 2 4
− = S64. Ans.(d)
14 5𝑥 105
x = 12 Sol. ? = 270-89 = 181

29 adda247.com/teachers | www.sscadda.com | www.bankersadda.com | www.adda247.com


S65. Ans.(c) S76. Ans.(e)
Sol. ? = 88 +288 S77. Ans.(d)
= 376 S78. Ans.(b)
S79. Ans.(d)
Solutions (66-70):
S80. Ans.(c)
Sol. Sent and Nest
Solutions (81-85):

S81. Ans.(a)
S66. Ans.(d) S82. Ans.(d)
S67. Ans.(c) S83. Ans.(a)
S68. Ans.(b) S84. Ans.(e)
S69. Ans.(a) S85. Ans.(c)
S70. Ans.(e) S86. Ans.(a)
Solutions (71-73): Sol.

S71. Ans.(d)
Sol.
Solutions (87-89):

S72. Ans.(b)
Sol.

S73. Ans.(a) S87. Ans.(c)


S88. Ans.(a)
Sol.
S89. Ans.(b)
Solutions (90-93):
Boxes
P
V
Q
R
S74. Ans.(d)
U
Sol.
W
T
S

S90. Ans.(d)
Solutions (75-79):
S91. Ans.(c)
S75. Ans.(b) S92. Ans.(b)
Sol. U S93. Ans.(e)

30 adda247.com/teachers | www.sscadda.com | www.bankersadda.com | www.adda247.com


Solutions (94-96): S97. Ans.(c)
S98. Ans.(a)
S94. Ans.(b)
S99. Ans.(d)
Sol. I: M = O (False) II: N < S (True) S100. Ans.(a)
S95. Ans.(c)
Sol. I: C ≥ F (False) II: F > C (False)

S96. Ans.(a)
Sol. I: S > W(True) II: U ≥ Y (False)

Solutions (97-100):

31 adda247.com/teachers | www.sscadda.com | www.bankersadda.com | www.adda247.com


Part - 3
Direction (1-5): Study the following information 6. Statements: Some Banana are Orange.
carefully and answer the questions given below: All Orange are Apple.
Conclusions: I. Some Banana are not Apple.
There are eight employees of a company and all of them II. Some Orange are Banana.
are working on eight different designation of a bank viz.
Chairman, CFO, GM, DGM, AGM, Manager, Junior Manager 7. Statements: Only a few East are North.
and Clerk. All the designations given are to be considered Few North are South.
All South are West.
in a given order (as Chairman is considered as Senior-
Conclusions: I. All East being North is a possibility.
most and Clerk is considered as the Junior-most).
II. Some East are West.
Only two persons are senior to B. One designation lies
8. Statements: Only a few Song are Melody.
between B and G. The number of persons junior to G is Only a few Melody are Film.
same as the number of persons senior to C. H is just senior No Film is Award.
to E, but junior to C. More than four designations lie Conclusions: I. All Song being Film is a possibility.
between H and F. D is junior to A. II. Some Melody are not Award.
1. How many persons are junior to H? Direction (9-12): Study the following information
(a) None (b) One (c) More than four carefully and answer the questions given below:
(d) Four (e) Three In a certain code language:
“Club house near located” is coded as “ol gp ox ot”
2. Four of the following five are alike in a certain way “both club view near” is coded as “mt ox sq ot”
and hence they form a group. Which one of the “make located house view” is coded as “nk ol gp sq”
following does not belong to that group? “near club view area” is coded as “ot ox sq tm”
(a) A-F (b) B-A (c) H-D
(d) G-A (e) E-H 9. What is the code for “both” in the given code
language?
3. How many designation gaps are between A and D? (a) ox (b) sq (c) mt
(a) More than three (b) Two (c) Three (d) ot (e) Either (b) or (c)
(d) One (e) None 10. What is the code for “house” in the given code
4. Who among the following is just senior to B? language?
(a) A (b) D (c) C (a) ol (b) ox (c) gp
(d) E (e) None of these (d) Either (a) or (c) (e) None of these
11. The code “nk” is stands for?
5. Who among the following is AGM?
(a) make (b) both (c) area
(a) F (b) B (c) G
(d) club (e) None of these
(d) D (e) None of these
12. What may be the possible code for “both house” in the
Direction (6-8): In each of the questions below are given given code language?
some statements followed by some conclusions. You have (a) gp mt (b) mt nk (c) mt sq
to take the given statements to be true even if they seem (d) ox mt (e) gp sq
to be at variance with commonly known facts. Read all the
Directions (13-17): Study the following information
conclusions and then decide which of the given
carefully and answer the questions given below:
conclusions logically follows from the given statements
There are eight persons A, B, C, D, E, F, G and H sitting
disregarding commonly known facts. around a circular table facing towards the centre of the
(a) If only conclusion I follows. table but not necessarily in the same order.
(b) If only conclusion II follows. B sits third to the right of A. Only one person sits between
(c) If either conclusion I or II follows. B and D. E faces C who is not an immediate neighbour of
(d) If neither conclusion I nor II follows. B. G sits third to the left of F. A is not an immediate
(e) If both conclusions I and II follow. neighbour of F.
13. Who among the following sits to the immediate left of 21. What is the sum of the 3rd digit of second number
C? from left and 2nd digit of third number from right?
(a) G (b) D (c) F (a) 10 (b) 15 (c) 14
(d) H (e) None of these (d) 12 (e) None of these
14. Who among the following sits to the opposite of A? 22. If the position of first and third digits of each of the
(a) H (b) G (c) F numbers are interchanged, then which among the
(d) D (e) None of these following is the highest number?
15. How many persons sit between H and G when (a) 258 (b) 469 (c) 189
counted from the left of G? (d) 427 (e) 625
(a) Two (b) One (c) Three 23. If all the digits in the number are arranged in the
(d) Four (e) None of these descending order within the number from left to
16. If all the persons sit around the circle in clockwise right, then which among the following will be the
direction by their names according to English lowest number after rearrangement?
alphabetical order starting from A, then how many (a) 427 (b) 189 (c) 258
persons remain unchanged except A? (d) 625 (e) 469
(a) Four (b) One (c) Three 24. What is the product of 3rd digit of 2nd lowest number
(d) Two (e) None of these and 1st digit of 2nd highest number?
17. What is the position of B with respect to H? (a) 32 (b) 72 (c) 24
(a) Fourth to the left (d) 36 (e) None of these
(b) Fourth to the right
25. If 2 is subtracted from each number then how many
(c) Third to the right
numbers thus formed are odd numbers?
(d) Both (a) and (c)
(a) One (b) Two (c) Three
(e) Both (a) and (b)
(d) More than three (e) None
Direction (18-20): Study the following information
26. How many such numerals are there in the number
carefully and answer the questions given below:
‘645903287’ which will remain at the same position
Point E is in 15m north of Point D. Point F is in 20m north when arranged in ascending order from left to right?
of Point C. Point A is in 35m east of Point F. Point P is in (a) Three (b) Two (c) One
25m south of Point A. Point E is in 20m east of Point P. (d) Four (e) None of these
18. What is the direction of point F with respect to point 27. If it is possible to make only one meaningful word
P? with the 1st, 3rd, 5th and 11th letters of the word
(a) North-west (b) North-east (c) South-west ‘INHERITENCE’, which would be the third letter of the
(d) South-east (e) North word from the left? If more than one such word can
19. Four of the following five are alike in a certain way be formed give ‘Y’ as the answer. If no such word can
and hence they form a group. Which one of the be formed, give ‘Z’ as your answer.
following does not belong to that group? (a) Y (b) R (c) I
(a) C-P (b) A-E (c) A-C (d) E (e) Z
(d) P-D (e) F-E Direction (28-32): Study the following information
20. If point X is 20m south of point C, then what is the carefully and answer the questions given below:
total distance between point X and point D? Eight persons A, B, C, D, E, F, G and H are buying some
(a) 40m (b) 35m (c) 55m products one after another but not necessarily in the
(d) 65m (e) 85m same order.
Directions (21-25): Following questions are based on At most two persons are buying products before F. Only
the five numbers given below, Study the given one person is buying between D and F. C is buying just
information and answer the following questions. before H. One person is buying between C and D. A is
625 427 189 258 469 buying just before E. B is buying before G and after E.
28. How many persons are buying their products after D? 32. How many persons are buying between E and C?
(a) None (a) Five (b) Four (c) Three
(b) Two (d) None (e) Two
(c) More than three
(d) One Directions (33-35): In each of the question, relationships
(e) None of these between some elements are shown in the statements.
These statements are followed by conclusions numbered I
29. Who among the following is buying just after G? and II. Read the statements and give the answer.
(a) E (b) F (c) B (a) If only conclusion I follows.
(d) C (e) None of these (b) If only conclusion II follows.
30. If all the persons are arranged in alphabetical order (c) If either conclusion I or II follows.
from left to right starting from A, then find how many (d) If neither conclusion I nor II follows.
persons remains at the same position (excluding A)? (e) If both conclusion I and II follows.
(a) One (b) None (c) Two 33. Statements: P > Q ≥ R = S < T = U
(d) Four (e) More than Four Conclusions: I. Q > S II. S = Q
31. Who among the following is buying exactly between D 34. Statements: J < D = L ≤ K ≤ Q ≥ R
and F? Conclusions: I. J < Q II. L ≤ R
(a) E (b) B (c) A
35. Statements: O > P = G > B ≥ X = M ≤ H
(d) H (e) None of these
Conclusions: I. G > X II. O > M
Quantitative Aptitude
Directions (36-40): Table given below shows the 44. 6, 8, 14, 26, 46, ?
number of male and female participated in an event from (a) 72 (b) 84 (c) 96
five different schools (A, B, C, D & E). Study the table (d) 80 (e) 76
carefully and answer the following questions. 45. 72000, 36000, 12000, 3000, 600, ?
Schools Male Female (a) 120 (b) 200 (c) 300
A 650 450 (d) 150 (e) 100
B 540 420 46. 12 men can do a work in 10 days while 10 women can
C 720 500 do the same work in 18 days. In how many days 4
D 560 450 men & 6 women together can do the same work?
120 180
E 680 320 (a) 7 days (b) 24 days (c) 13 days
(d) 15 days (e) 18 days
36. Find average number of female participated from
school – A, B & D. 47. A car can cover a distance in 4 hour at speed 60 kmph
then by what percent should the speed of car be
(a) 400 (b) 380 (c) 350
increased to cover the same distance in 2.5 hr?
(d) 440 (e) 450
(a) 60% (b) 40% (c) 50%
37. Total male participated from school – B & D together (d) 100% (e) 75%
are how much more or less than total female 48. The ratio of the ages of Ram and Rahim 10 years ago
participated from school – A & C together? was 1 : 3. The ratio of their ages five years hence will
(a) 150 (b) 110 (c) 170 be 2 : 3. Then, the ratio of their present ages is :
(d) 120 (e) 240 (a) 1 : 2 (b) 3 : 5 (c) 3 : 4
38. Total male participated from school – B & C together (d) 2 : 5 (e) None of these
are what percent more or less than total female 49. Two trains of length 140m & 120m are running in
participated from school – A & D together? same direction on parallel tracks with speeds 132
(a) 20% (b) 60% (c) 50% kmph & 80 kmph respectively. How much time will
(d) 40% (e) 30% they take to cross each other?
(a) 7.09 sec (b) 18 sec (c) 11.7 sec
39. If total male participated from school – F are 40%
(d) 4.42 sec (e) Cannot be determined
more than that of from school – A and ratio of female
participated from school – B to that of from school – F 50. A person sold a book at 20% profit. If he had bought it
is 21:32, then find total students participated from at 10% less cost and sold for Rs 90 more then he
school – F. would have gained 40% profit. Find cost price of
(a) 1420 (b) 1550 (c) 1580 book.
(d) 1460 (e) 1490 (a) Rs 800 (b) Rs 1600 (c) Rs 1500
40. Find total number of male students participated from (d) None of these (e) Rs 1200
all the five schools together. Direction (51–55): In each question two equations
(a) 2860 (b) 3150 (c) 2940 numbered (I) and (II) are given. You have to solve both
(d) 3200 (e) 3020 the equations and mark appropriate answer.
Direction (41–45): What will come in the place of (a) If x = y or no relation can be established
question (?) mark in following number series: (b) If x > y
(c) If x < y
41. ?, 100, 150, 375, 1312.5 (d) If x ≥ y
(a) 100 (b) 200 (c) 150 (e) If x ≤ y
(d) 400 (e) 50
51. I. 𝑥 = √25 II. y3 = 125
42. 104, ?, 96, 120, 88, 128
(a) 112 (b) 110 (c) 114 52. I. x2 + 2x – 35 = 0 II. y2 + 15y + 56 = 0
(d) 118 (e) 108 53. I. x2 = 81 II. y2 = 64
43. 15, 8, 9, 15, 32, ?
54. I. 17x2 – 14x – 83 = - 80 II. y2 = 2y + 35
(a) 66 (b) 99 (c) 80
(d) 82.5 (e) 80.5 55. I. x2 + 4x – 45 = 0 II. y2 − 13𝑦 + 40 = 0
56. A container contains mixture of milk & water in ratio Direction (61–70): What will come in the place of (?)
5 : 3 respectively. If 8 lit milk is added in it then ratio mark in following question.
of milk to water becomes 11 : 5. Find difference 61. 280 ÷ 4 ÷ 2 = 170 – ?
between initial quantity of milk & that of water. (a) 105 (b) 115 (c) 125
(a) 5 lit (b) 38 lit (c) 18 lit (d) 135 (e) 145
(d) 30 lit (e) 10 lit 62. (√144 + √169 ) × 3 = 5
?

57. Rs 6000 when invested at a certain rate at SI for 2 (a) 375 (b) 325 (c) 350
(d) 275 (e) 475
years, it fetches Rs 1200. If same sum is invested at
same rate for a year compounded half – yearly then 63. (12 × 5 ÷ 4) × 8 = ?
find compound interest. (a) 100 (b) 140 (c) 120
(d) 80 (e) 90
(a) Rs 615 (b) Rs 600 (c) Rs 1200
(d) Rs 585 (e) Rs 1260 64. (120% of 750) ÷ ? = 25
(a) 30 (b) 36 (c) 24
58. A boat can cover 28 km downstream in 42 min. ratio (d) 18 (e) 48
of speed of boat in still water to speed of stream is 7 : 1
65. 8 − 4 =? −3
5 7

3. Find difference between time taken by boat to 2


1
6 12
5 7
(a) 3 4 (b) 3 12 (c) 2 12
cover 60 km downstream & 40 km upstream.
1 2
(a) 2.25 hr (b) 1 hr (c) 1.5 hr (d) 7 4 (e) 5 3
(d) 0.4 hr (e) 0.9 hr 66. 275 + 64% of 750 = 750 + ?
(a) 25 (b) 8 (c) 10
59. A & B entered into a business by investing total
(d) 15 (e) 5
capital of Rs 17000. B withdraws Rs 1500 after 6
months and gets Rs 8100 as profit out of total profit of 67. √225 + √81 + 122 =?
Rs 19500 at the end of year. Find capital of B after 6 (a) 168 (b) 164 (c) 162
(d) 172 (e) 182
months from starting.
510
(a) Rs 7000 (b) Rs 9500 (c) Rs 7500 68. ?
= √324 + 3.25
(d) Rs 6000 (e) Rs 6500 (a) 12 (b) 48 (c) 24
(d) 6 (e) 18
60. If length of a rectangle increases by 40% while
69. 12.5% of (120 + ?) = 45
keeping breadth constant then area of rectangle
(a) 160 (b) 180 (c) 360
increased by 24 m2 and perimeter of original (d) 240 (e) 120
rectangle is 32 m. find breadth of rectangle.
70. 572 ÷ 13 × 12 – 16 = (8)?
(a) 8.4 m (b) 10 m (c) 6 m (a) 4 (b) 2 (c) 3
(d) 14 m (e) 8 m (d) 5 (e) None of these
English Language

Directions (71-77) Read the given passage carefully and Meanwhile, the impact of mosquitoes carrying these new
answer the following questions. Certain parts have been genes remains largely unknown. One significant worry is
highlighted to help answer the questions. that a new breed of mosquito might emerge that is more
difficult to control. These new genes could also potentially
Every year, around one million people die of mosquito-
alter evolutionary pressures on viruses carried by
borne diseases according to the World Health
mosquitoes, like dengue fever, in unpredictable ways.
Organization (WHO). This is why mosquitoes are
This includes potentially increasing their virulence or
considered one of the deadliest living creatures on the
changing their host-insect interactions. These are
planet — not because they are lethal themselves, but
hypothetical risks that have been raised by scientists, and
because many of the viruses and parasites they transmit
reflect the need for further study.
are.
In the absence of an effective vaccine for dengue fever, 71. Why are mosquitoes considered as one of the
Zika fever, chikungunya and other mosquito-borne deadliest living creatures on earth?
diseases, researchers have developed genetic strategies to (a) Capability to transmit life threatening diseases.
reduce mosquito populations. One such strategy involves (b) Mosquito bites cause fatal reactions in human
the release into the wild of genetically modified (GM) beings
mosquitoes that express a lethal gene — a strategy (c) Mosquitoes cannot be killed easily
believed to have little impact on the overall DNA of wild (d) Both (a) and (c)
populations of mosquitoes. (e) None of the above
The transfer of new genes from GM organisms to wild or 72. What is one of the methods being developed to
domesticated non-GM populations is a key criticism of GM control population of mosquitoes?
crops like soybean and corn. There are concerns that the (a) Spraying of pesticides using established
introduction of GM genes into non-target species could procedures.
have negative consequences for both human and (b) Introduction of a lethal gene into wild populations
environmental health. of mosquitoes using genetically modified ones.
Oxitec, a company that spun out of research at Oxford (c) Not letting any stagnant water build-up near
University in the early 2000s, developed and trademarked homes
GM Friendly™ mosquitoes (also known as strain OX513A (d) Both (a) and (b)
of Aedes aegypti). These male GM mosquitoes have what (e) None of the above
the company describes as a “self-limiting” gene, which
means that when these so-called friendly mosquitoes’ 73. What can be the potential side effect(s) of genetically
modified mosquito offspring?
mate, their offspring inherit the self-limiting gene which is
(a) Evolution of a breed of mosquitoes which are
supposed to prevent them surviving into adulthood.
more difficult to control
In theory, when these mosquitoes are released in high
(b) Increase in the severity of diseases being
numbers, a dramatic reduction in the mosquito
currently transmitted by mosquitoes.
population should follow. According to research
(c) Change in host-insect interactions.
published by Oxitec researchers in 2015, field trials
(d) Loss of natural populations of mosquitoes
involving recurring releases of Friendly™ mosquitoes
(e) (a), (b) and (c)
demonstrated a reduction of nearly 95 per cent of target
populations in Brazil. In these field trials, experiments 74. Which of the statements can be considered as true
were not performed to assess whether GM mosquitoes with respect to the passage given?
might persist in the wild. (a) Field trials on Genetically modified mosquitoes
A recent study from the Powell lab at Yale University has showed reduction of 96% target populations.
since confirmed that some of the offspring of the GM (b) Oxitec trademarked the GM Friendly mosquitoes
mosquitoes didn’t succumb to the self-limiting lethal gene in 2015.
and survived to adulthood. They were able to breed with (c) The female GM mosquito has a self-limiting gene
native mosquitoes and thereby introduce some of their (d) Both (a) and (b)
genes into the wild population. (e) None of the above
75. What is the desired effect of releasing GM mosquitoes 80. ESCALATE
containing the ‘Self-limiting Gene’? (i) The pedestrian was jaywalking when he escalate
(a) A substantial reduction in the population of the busy street, walking straight into the path of
mosquitoes. an oncoming vehicle.
(b) Birth of mosquitoes without a biting mechanism (ii) We've tried to escalate the most likely problems,
(c) Decrease in the number of diseases caused due to but it's impossible to be prepared for every
mosquitoes eventuality.
(d) Increase in the number of male mosquitoes. (iii) The decision to escalate UN involvement has
(e) Both (b) and (c) been made in the hopes of a swift end to the
hostilities.
76. Which of the following word is similar to deadliest as (a) Only (i)
mentioned in the passage given? (b) Both (i) and (ii)
(a) fatal (b) demeaning (c) fulfil (c) Both (ii) and (iii)
(d) harmless (e) None of the above (d) Only (iii)
77. Which of the statements can be considered as true (e) All of these
with respect to the passage given?? Directions (81-85): In the following question, four
(a) GM mosquitoes are able to introduce some of sentences are given which may be grammatically and
their genes into the wild population. contextually incorrect. You need to find the one which has
(b) Mosquitoes having GM genes have been no error and mark that as your answer. If all the given
thoroughly researched upon. sentences are incorrect then mark option (e) i.e. ‘all are
(c) Oxitec released its friendly mosquitoes in Brazil incorrect’ as your answer.
(d) (a) and (c)
81. (a) India’s chocolate market is pegged at Rs.11,000
(e) All of the above
crore, of which premium chocolates can be sized
Directions (78-80): A word has been given in each anywhere among 10% and 15%.
question and has been used in the sentences given below. (b) South Korea reported 52 new case of coronavirus.
Identify the statements where the word has been used in (c) ITC has plans to work with cocoa farmers directly
a contextually and grammatically correct manner. If the in future.
word has been used correctly in all the statements, mark (d) The company initially invested Rs. 100 crore to
(e), “All of these”, as your answer. set up a greenfield chocolate manufacture facility
in Haridwar.
78. HOSTILE (e) All are incorrect
(i) He made hostile efficiency a key part of his
budget plan. 82. (a) Ratnakar Bank was rename as RBL Bank by the
(ii) There has been a hostile reaction to the government.
government's proposed tax increase. (b) The government expects all the mergers of public
(iii) The security forces exercised great restraint by sector banks, which were announced in 2019, to
not responding to hostile attacks and threats. become effective from April 1.
(a) Only (i) (c) Analysts believe that lower gas prices would be
(b) Both (i) and (ii) bad with exploration
(d) The Central Drugs Standard Control Organisation
(c) Both (ii) and (iii)
will be regulate the medical devices now.
(d) Only (iii)
(e) All are incorrect
(e) All of these
83. (a) Mahindra Renewables will sell its entire stake in 3
79. VAGUE
subsidiaries to CLP India, a part of Hong Kong-
(i) The judges determined that the law was too
based CLP Group, as nearly ₹340 crore.
vague to be fairly enforced. (b) The agreement will be signed to Doha city
(ii) The merger of these two companies would vague between Taliban representatives and U.S. special
the world's biggest accounting firm envoy Zalmay Khalilzad.
(iii) The president had a vague reception in Ohio this (c) The U.S. intelligence community publicly
morning. concluded that Russia intervened in there
(a) Only (i) security matters.
(b) Both (i) and (ii) (d) The injured animal was nursed back to health by
(c) Both (ii) and (iii) the zookeeper and then released back into the
(d) Only (iii) wild
(e) All of these (e) All are incorrect
84. (a) English language have the potential to connect the Directions (91-95): In the following questions, a
world without any barriers grammatically correct and meaningful sentence is given
(b) P.K. Sinha was appointed in the PMO when which is divided into five parts, where the first part is
Nripendra Misra, the then Principal Secretary, fixed and highlighted , in the remaining parts of the
resigns. sentence namely (A),(B),(C) and (D) You have to arrange
(c) She drink the caramel liquid too fast and was the four parts to make a contextually and grammatically
meaningful sentence. If no such rearrangement is possible
soon too dizzy to stand.
mark (e) as your answer i.e. 'No rearrangement required'.
(d) You’re coming to the party, isn’t you?
(e) All are incorrect 91. (A) the merger of Bharti Infratel
(B) the world’s second largest
85. (a) Tamil Nadu is likely to get three textile parks and (C) and Indus Towers will create
a technology research centre under schemes that (D) tower company
the Union Ministry of Textiles is formulating. (a) BCDA (b) CBDA (c) ACBD
(b) What is the uses of a house if you haven't got a (d) DCAB (e) No rearrangement is required
tolerable planet to put it on?
92. (A) upcoming visit to India
(c) You have brushed your teeth today? (B) the long-awaited trade agreement
(d) Rahul had barely nothing to help the poor (C) during US President Donald Trump's
children. (D) appears to be off the table
(e) All are incorrect (a) DABC (b) BDCA (C) CDAB
(d) ADCB (e) No rearrangement required
Directions (86-90): In the following questions, a
sentence is divided into four parts consisting of a 93. (A) Leaders of the Railway Employees Organization
highlighted word in each part. Choose the option (B) alleged that the government wants to
reflecting the word which is either misspelt or (C) hand over the rail infrastructure
grammatically incorrect. If all the highlighted words are (D) to the private operators for financial benefit
correct, choose option (e) i.e. “all are correct” as your (a) ADCB (b) BACD (c) CDBA
answer choice. (d) DABC (e) No rearrangement required
94. (A) China has not yet given
86. The manager balanced (A) the strength (B) of his
(B) India the go-ahead to send an
team against that of their opponent (C) and sighed. (C) to coronavirus hit Wuhan
(D) (D) aircraft with medical supplies
(a) balanced (b) strength (c) opponent (a) DACB (b) ABDC (c) CBDA
(d) sighed (e) All are correct (d) BCDA (e) No rearrangement is required
87. One evening Rohan pripared (A) a nice supper (B) 95. (A) mammals and birds move from
and put it on low heat in the oven to keep it tepid (C) (B) with a change in season, many
while his wife dressed. (D) (C) one country to another in search of food
(a) prepared (b) supper (c) tepid (D) and shelter, and for breeding
(d) dressed (e) All are correct (a) ACBD (b) CADB (c) DABC
(d) BACD (e) No rearrangement is required
88. She was able to breathe (A) easier when she stood
outside the massive(B) fortress (C) that sat on a Directions (96-100): In the following passage there are
blanks, each of which has been numbered. These numbers
clearing the size of two footbal (D) fields.
are printed below the passage and against each, five
(a) breathe (b) massive (c) fortress options are given. Find out the appropriate word which
(d) football (e) All are correct fits the blank appropriately.
89. From time to time Jack would looking over(A) Bumblebees, among the most important pollinators, are in
Mungo's shoulder, suggesting(B) tactics (C) which (96) ________________. Fuzzy and buzzy, they excel at
invariably proved disastrous.(D) spreading pollen and fertilizing many types of wild flora,
(a) looking over (b) suggesting (c) tactics as well as crucial agricultural crops like tomatoes,
(d) disastrous (e) All are correct blueberries, and squash. But their numbers are (97)
________________. New research using a massive dataset
90. Horrified (A) passengers saw Olive stumble (B) and found that the insects are far less common than they used
fall of (C) a platform as an express roared (D) in. to be; in North America, you are nearly 50 percent less
(a) Horrified (b) stumble (c) fall of likely to see a bumblebee in any given area than you were
(d) roared (e) All are correct prior to 1974. Moreover, several once-common species
have (98) __________________ from many areas they were 96. (a) extinct (b) trouble (c) dropped
once found, becoming locally extinct in those places. For
(d) growth (e) difficult
example, the rusty patched bumblebee, which used to
flourish in Ontario, is no longer found in all of Canada—in 97. (a) increasing (b) dripped (c) removing
the U.S., it’s endangered. In a new paper published this
week in the journal Science, researchers used a complex (d) dropping (e) generating
modeling process to (99) ________________ that their
98. (a) multiplied (b) concerning (c) disappeared
(100)___________________ is driven in large part by climate
change. Specifically, the scientists found that in areas that (d) certain (e) vanish
have become hotter in the last generation, or have
99. (a) proposed (b) recommending
experienced more extreme temperature swings,
bumblebees are less abundant. In Europe, they are 17 (c) implying (d) expresses (e) suggest
percent less plentiful than they were in the early 20th
century. The scientists examined the abundance of 66 100. (a) decline (b) deteriorate (c) rejection
species across the two continents. (d) lessen (e) reduced
Reasoning Ability (Solutions)

Solutions (1-5):
Solutions (13-17):
Designation Persons
Chairman F
CFO A
GM B
DGM C
AGM G
Manager D
Junior manager H
Clerk E 13. (b); 14. (c); 15. (b);
16. (d); 17. (e);
1. (b) 2. (d) 3. (c) Solutions (18-20):
4. (a) 5. (c)
6. (b);

18. (a);
19. (c);
20. (c);
7. (d);

8. (e);
21. (b); 22. (c); 23. (d)
24. (a); 25. (d);
Solutions (9-12): 26. (c); 6 4 5 9 0 3 2 8 7
Word Code 0 2 3 4 5 6 7 8 9
Club Ox 27. (a); Hire, Heir
Near Ot
House/Located Ol/gp Solutions (28-32):
Both Mt
Make Nk
Area Tm 28. (e); 29. (d); 30. (a);
View Sq 31. (b); 32. (b);
9. (c) 33. (c); I. Q > S (False) II. S = Q (False)
10. (d)
11. (a) 34. (a); I. J < Q (True) II. L ≤ R (False)
12. (a) 35. (e); I. G > X (True) II. O > M(True)
Quantitative Aptitude (Solutions)

36. (d); Required average =


450+420+450
= 440 48. (b); Let 10 years ago, ages of Ram and Rahim were x
3
years and 3x years, respectively.
37. (a); Total male participated from school – B & D Then, present age of Ram = (x + 10)
together = 540 + 560 = 1100 and present age of Rahim = (3x + 10)
Total female participated from school – A & C According to the question,
x+10+5 2
together = 450 + 500 = 950 =
3x+10+5 3
Required difference = 1100 − 950 = 150 ⇒ 3x + 45 = 6x + 30
38. (d); Total male participated from school – B & C ⇒ 3x = 15
together = 540 + 720 = 1260 ∴x=5
5+10
Total female participated from school – A & D Hence, required ratio =
3×5+10
together = 450 + 450 = 900 15
= 25 = 3 ∶ 5
1260−900
Required % = × 100 = 40% 140+120
900
49. (b); required time = 5
(132−80)×
39. (b); Total students participated from school F = 260×18
18
140
× 650 + 420 × 21
32 = 52×5
= 18 sec
100
= 910 + 640 = 1550 50. (c); let CP of book be Rs x
SP = Rs 1.2x
40. (b); Total number of male students participated from
New CP = Rs 0.9x
all the five schools New SP = Rs 1.2x + 90
= (650 + 540 + 720 + 560 + 680) = 3150 140
ATQ, 0.9𝑥 × = 1.2𝑥 + 90
100
41. (b); Pattern of series – 1.26x = 1.2x + 90
x = Rs 1500
51. (a); I. x = 5
42. (a); Pattern of series - II. y = 5
So, x=y
52. (d); I. x2 + 7x – 5x – 35 =0
x (x + 7) – 5 (x + 7) =0
43. (d); Pattern of series -
(x + 7) (x – 5) = 0
x = −7, 5
II. y2 + 7y + 8y + 56 = 0
44. (e); Pattern of series - y(y + 7) + 8(y + 7) = 0
(y + 7) (y + 8) = 0
y = − 8, −7
So, x≥y

45. (e); Pattern of series - 53. (a) I. x = ± 9


II. y = ± 8
So, no relation can be established

46. (d); let total work be 360 units 54. (a); I. 17x2 – 14x – 3 = 0
360 17x2 – 17x + 3x – 3 = 0
Efficiency of 1 man = 12×10 = 3 units/day
17x (x – 1) + 3(x – 1) = 0
360
Efficiency of 1 woman = 10×18 = 2 units/day (17x + 3) (x – 1) = 0
3
360
Required time = 4×3+6×2 = 15 days x = − ,1
17
II. y2 – 2y – 35 = 0
47. (a); distance = 240 kms y2 – 7y + 5y – 35 = 0
240
Required speed = 2.5 = 96 kmph y(y −7) + 5(𝑦 − 7) = 0
96−60 y = 7, - 5
Required % = × 100 = 60%
60 So, no relation can be established
55. (e); I. x2 + 9x – 5x – 45 = 0 60. (c); let length & breadth of rectangle be x & y m
x(x + 9) – 5(x + 9) = 0 respectively
(x – 5) (x + 9) = 0 ATQ, 1.4xy – xy = 24
x = 5, - 9 xy = 60 ………………(i)
II. y2 – 5y – 8y + 40 = 0 also, 2(x + y) = 32
y(y – 5) – 8(y – 5) = 0 x + y = 16 ………….(ii)
(y – 5) (y – 8) = 0 from (i) & (ii)
y = 5, 8 x = 10 m, y = 6 m
So, x≤y breadth of rectangle = 6 m

56. (e); let initial quantity of milk & water be 5x & 3x lit 61. (d); ? = 170 – 35
? = 135
respectively
5𝑥+8 11 ?
ATQ, 3𝑥 = 5 62. (a); (12 + 13) × 3 =
5
25x + 40 = 33x ⇒ x = 5 ? = 375
required difference = 5x – 3x = 2x = 10 lit 63. (c); ? = (3 × 5) × 8
57. (a); let rate of interest be R% ? = 120
6000×𝑅×2
ATQ, 1200 = 120
64. (b); ( 100 × 750 ) ÷? = 25
100
R = 10% ? = 900 ÷ 25
Since compounding is done half-yearly, rate of ? = 36
interest = 5% 6−10+7
Effective rate of interest = 5 + 5 +
5×5
= 10.25% 65. (d); ? = (8 – 4 + 3)+
12
100 1
Required interest =
6000×10.25×1
= Rs 615 ?=7
4
100
64
58. (b); let speed of boat in still water & speed of stream 66. (e); 275 + × 750 = 750 + ?
100
be 7x & 3x kmph respectively 275 + 480 = 750 + ?
28 42
ATQ, 7𝑥+3𝑥 = 60 ?=5
x=4 67. (a); ? = 15 + 9 + 144
40 60 4
Required difference = 7𝑥−3𝑥 − 7𝑥+3𝑥 = 𝑥 = 1 hour ? = 168
510
59. (d); let amount invested by A be Rs x 68. (c); ?
= 18 + 3.25
Profit ratio; A : B = (x × 12) : (17000 – x) × 6 + ? = 24
(15500 – x) × 6 69. (d);
12.5
× (120+? ) = 45
= 2x : (32500 – 2x) 100
19500 120 + ? = 360
ATQ, × (32500 − 2𝑥) = 8100
32500−2𝑥+2𝑥 ? = 240
32500 – 2x = 13500
x = Rs 9500 70. (c); 44 × 12 − 16 = (8)?
Required capital of B after 6 months 528 – 16 = (8)?
= 15500 – x = Rs 6000 ?=3
English Language (Solutions)
71. (a); Reading the first paragraph of the passage it can 76. (a); Among the given words, ‘fatal’ is the synonym of
be deduced that the correct answer is option(a). ‘deadliest’. Hence, option (a) is the
The relevant sentences of the mentioned most suitable answer choice.
sentences have been quoted below: Fatal: causing death.
“This is why mosquitoes are considered one of Demeaning: causing someone to lose their
the deadliest living creatures on the planet — not dignity and the respect of others.
because they are lethal themselves, but because
77. (d); Reading the above passage it can be seen that the
many of the viruses and parasites they transmit
correct statements are (a) and (c). The sentences
are.”
have been quoted below for your reference-
72. (b); Reading the second paragraph of the passage it “According to research published by Oxitec
can be deduced that the correct answer is researchers in 2015, field trials involving
option(b). The sentences of the passage which recurring releases of Friendly™ mosquitoes
substantiate this are given below: demonstrated a reduction of nearly 95 per
“One such strategy involves the release into the cent of target populations in Brazil. A recent
wild of genetically modified (GM) mosquitoes study from the Powell lab at Yale University has
that express a lethal gene — a strategy believed since confirmed that some of the offspring of the
to have little impact on the overall DNA of wild GM mosquitoes didn’t succumb to the self-
populations of mosquitoes.” limiting lethal gene and survived to adulthood.
They were able to breed with native
73. (e); Reading the last paragraph of the passage it can
mosquitoes and thereby introduce some of
be deduced that the correct answer is
their genes into the wild population.”
option(e). The relevant sentences of the
mentioned sentences have been quoted below: 78. (c); Hostile means showing or feeling opposition or
“One significant worry is that a new breed of dislike; unfriendly. Therefore, sentence (ii) and
mosquito might emerge that is more difficult to (iii) depict the correct usage of the given word.
control. These new genes could also potentially Hence, option (c) is the most suitable answer
alter evolutionary pressures on viruses carried choice.
by mosquitoes, like dengue fever, in
79. (a); Vague means thinking or communicating in an
unpredictable ways. This includes potentially
unfocused or imprecise way. Therefore, only
increasing their virulence or changing their host-
sentence (i) depicts the correct usage of the given
insect interactions.”
word. Hence, option (a) is the most suitable
74. (e); Reading the passage carefully we can see that answer choice.
none of the sentences given above are factually
80. (d); Escalate means increase rapidly. Therefore, only
correct hence the correct answer will be option
sentence (iii) depicts the correct usage of the
(e).
given word. Hence, option (d) is the most suitable
75. (a); Reading the fourth and fifth paras of the passage answer choice.
it can be deduced that the correct answer is
option(a). The relevant sentences of the 81. (c); As we go through the analysis of the sentence, we
mentioned sentences have been quoted below: find that the sentence of option (c) is correct.
“These male GM mosquitoes have what the (a) replace ‘among’ with ‘between’. Among is
company describes as a “self-limiting” gene, used for a greater number.
which means that when these so-called friendly (b) replace ‘case’ with ‘cases’.
mosquitoes’ mate, their offspring inherit the self- (d) replace’ manufacture’ with ‘manufacturing’.
limiting gene which is supposed to prevent them 82. (b); As we go through the analysis of the sentence, we
surviving into adulthood In theory, when these find that the sentence of option (b) is correct.
mosquitoes are released in high numbers, a (a) replace ‘rename’ with ‘renamed’.
dramatic reduction in the mosquito population (c) replace ‘with’ with ‘for’.
should follow.” (d) replace ‘regulate’ with ‘regulating’.
83. (d); As we go through the analysis of the sentence, we “The long-awaited trade agreement appears to be
find that the sentence of option (d) is correct. off the table during US President Donald Trump’s
(a) replace ‘as’ with ‘for’. upcoming visit to India.”
(b) replace ‘to’ with ‘in’.
(c) replace ‘there’ with ‘their’. 93. (e); In the given sentence the sentence elaborates on
allegations made against the government by
84. (e); As we go through the sentences, we find that all leaders of railway employee organisations with
sentences are incorrect. respect to their stand on handing over railway
(a) replace ‘have’ with ‘has’. infrastructure to private companies. Hence,
(b) replace ‘resigns’ with ‘resigned’. option (e) is the most suitable answer choice
(c) replace ‘drink’ with ‘drank’.
which means the sentence requires no
(d) replace ‘isn’t’ with ‘aren’t’.
rearrangement. The statement thus formed will
85. (a); As we go through the analysis of the sentence, we be:
find that the sentence in option (a) is correct. “Leaders of the Railway Employees Organization
(b) replace ‘uses’ with ‘use’. alleged that the government wants to hand over
(c) interchange ‘have’ with ‘you’ to maintain the the rail infrastructure to the private operators for
interrogative form of the sentence. financial benefit.”
(d) replace ‘nothing’ with ‘anything’ because
‘barely’ will not take ‘nothing’ after it. 94. (b); The correct rearrangement of the parts of
statement will be “ABDC” which will make a
86. (a); Except “balenced” which is spelt as “balanced” all
grammatically correct and contextually
other words are correctly spelt so our answer
meaningful statement. The given statement talks
choice will be option (a).
about India not being granted the permission to
87. (a) ; Except “pripared” which is spelt as “prepared” all send an aircraft carrying medical supplies to the
other words are correctly spelt so our answer coronavirus hit Wuhan region. The correct
choice will be option (a). statement thus formed will be:
88. (d); Except “footbal” which is spelt as “football” all “China has not yet given India the go-ahead to
other words are correctly spelt so our answer send an aircraft with medical supplies to
choice will be option (d). coronavirus hit Wuhan.”
89. (a); Except “looking over” which is grammatically 95. (d); The correct rearrangement of the parts of
wrong all other words are correctly spelt and are statement will be “BACD” which will make a
grammatically flawless so our answer choice will grammatically correct and contextually
be option (a). Replace “looking over” with “look meaningful statement. The given sentence talks
over”. about migration in animals with changing
90. (c); Except “fall of” which is spelt as “fall off” all other seasons in search of their basic needs. Hence,
words are correctly spelt so our answer choice option (d) is the most suitable answer choice.
will be option (c). The correct statement thus formed will be:
“With a change in season, many mammals and
91. (c); The correct rearrangement of the parts of birds move from one country to another in
statement will be “ACBD” which will make a
search of food and shelter, and for breeding.”
grammatically correct and contextually
meaningful statement. Hence, option (c) is the 96. (b); The given paragraph is describing the alarming
most suitable answer choice. The correct situation of Bumblebees where their numbers
statement thus formed will be: are continuously declining. Thus, the most
“The merger of Bharti Infratel and Indus Towers suitable word for the given blank is “trouble”.
will create the world’s second largest tower Hence, option (b) is the correct answer choice.
company.”
97. (d); The given paragraph is describing the alarming
92. (b); The correct rearrangement of the parts of situation of Bumblebees where their numbers
statement will be “BDCA” which will make a are continuously declining. Thus, the most
grammatically correct and contextually
suitable word for the given blank is “dropping”.
meaningful statement. Hence, option (b) is the
All the other words are contextually incorrect.
most suitable answer choice. The correct
Hence, option (d) is the correct answer choice.
statement thus formed will be:
98. (c); The given paragraph is describing the alarming 99. (e); The most suitable word for the given blank is
situation of Bumblebees where their numbers “suggest”. All the other words are either
are continuously declining. Thus, the most grammatically or contextually incorrect. Hence,
suitable word for the given blank is option (e) is the correct answer choice.
“disappeared”. All the other words are either
grammatically or contextually incorrect. Hence, 100.(a); The given paragraph is describing the alarming
option (c) is the correct answer choice. situation of Bumblebees where their numbers
Disappeared means cease to exist or be in use. are continuously declining. Thus, the most
Vanish means disappear suddenly and suitable word for the given blank is “decline”. All
completely. the other words are either grammatically or
Certain means able to be firmly relied on to contextually incorrect. Hence, option (a) is the
happen or be the case. correct answer choice.
Part - 4

Reasoning Ability

Direction (1-5): Study the following information carefully and answer the questions given below:
Eight persons are sitting around a circular table facing to the center but not necessarily in the same order.
Two persons sit between Q and P (either from left or right). R sits immediate to the right of Q. One person
sits between R and S, who faces to T. Q and T are not immediate neighbors of each other. W sits 2 nd to the
left of V. Three persons sit between U and V.
1. Four of the following five are alike in a certain way and hence they form a group. Which one of the
following does not belong to that group?
(a) Q-W
(b) P-U
(c) S-W
(d) V-T
(e) Q-P
2. Who among the following sits immediate right of U?
(a) W
(b) R
(c) T
(d) S
(e) V
3. The number of persons sit between Q and T , when counted to right of Q is same as the number of
persons sit between W and ___, when counted to the left of ___?
(a) P
(b) S
(c) T
(d) U
(e) None of these
4. Who among the following faces R?
(a) U
(b) V
(c) P
(d) W
(e) Q
5. Who among the following sits 3rd to the right of S?
(a) P
(b) U
(c) Q
(d) T
(e) None of these
Direction (6-10): Study the following information carefully and answer the questions given below:
Ten persons are sitting in two parallel rows containing five persons in each row such a way that there is an
equal distance between adjacent persons. In the first row, A, B, C, D and E are seated and all of them are
facing north. In the second row, P, Q, R, S and T are seated and all of them are facing south. Therefore, in
the given seating arrangement, each member seated in a row faces another member of the other row.
E sits 2nd from one of the extreme end of the row. P faces the one who sits 2 nd to the right of E. D sits 2nd to
the left of B, who does not sit at the extreme end. Two persons sit between S and Q. R sits immediate left of
S. C sits next to B.
6. Who among the following faces Q?
(a) B
(b) D
(c) A
(d) C
(e) None of these
7. Who among the following sits at the extreme end of the row?
(a) E
(b) T
(c) C
(d) R
(e) P
8. Four of the following five are alike in a certain way and hence they form a group. Which one of the
following does not belong to that group?
(a) Q
(b) A
(c) T
(d) D
(e) P
9. Who among the following sits 2nd to the right of P?
(a) S
(b) R
(c) Q
(d) T
(e) None of these
10. What is the position of A with respect to E?
(a) 3rd to the left
(b) Immediate to the left
(c) 2nd to the left
(d) 3rd to the right
(e) 2nd to the right
11. In the word ‘PRODUCE’, how many pairs of the letters have
the same number of letters between them in the given word
as they have in the English alphabet series?
(a) Four
(b) Two
(c) One
(d) Three
(e) More than four
Direction (12-15): In each of the questions below are given some statements followed by some
conclusions. You have to take the given statements to be true even if they seem to be at variance with
commonly known facts. Read all the conclusions and then decide which of the given conclusions logically
follows from the given statements disregarding commonly known facts.
12. Statements: All Chocolate are Toffee. No Toffee is Muffin. Only a few Muffin are Dark.
Conclusions: I. No Chocolate is Muffin.
II. Some Toffee can never be Dark.
(a) If only conclusion II follows.
(b) If only conclusion I follows.
(c) If neither conclusion I nor II follows.
(d) If either conclusion I or II follows.
(e) If both conclusions I and II follow.
13. Statements: No Party is Gathering. All Summary are Gathering. Only a few Gathering are
Committee.
Conclusions: I. Some Committee can be Summary.
II. Some Party can be Committee.
(a) If either conclusion I or II follows.
(b) If only conclusion I follows.
(c) If neither conclusion I nor II follows.
(d) If only conclusion II follows.
(e) If both conclusions I and II follow.
14. Statements: No Market is Home. Only a few Home are Room. Only a few Room is Vance.
Conclusions: I. Some Home are Vance.
II. Some Market can never be Room.
(a) If only conclusion I follows.
(b) If neither conclusion I nor II follows.
(c) If either conclusion I or II follows.
(d) If only conclusion II follows.
(e) If both conclusions I and II follow.
15. Statements: Some Ball are Garden. All Garden are Trade. Only a few Trade are Pump.
Conclusions: I. Some Ball are Pump.
II. Some Garden can be Pump.
(a) If only conclusion II follows.
(b) If neither conclusion I nor II follows.
(c) If either conclusion I or II follows.
(d) If only conclusion I follows.
(e) If both conclusions I and II follow.
Direction (16-20): Study the following information carefully and answer the questions given below:
Seven persons are buying different things. Only one person buy thing between C and A. B is buying
thing immediately before C. Two persons buy things between C and D. C buy things after D. E buys thing
immediately before D. More than three persons buy things between E and F. Only three persons buy things
between D and G.
16. How many persons buy things between A and F?
(a) One
(b) Two
(c) Three
(d) Four
(e) More than four
17. Who among the following buy thing immediately after F?
(a) D
(b) B
(c) G
(d) A
(e) No one
18. Who among the following buy thing immediately before G?
(a) D
(b) B
(c) C
(d) A
(e) No one
19. How many person buy things before A?
(a) One
(b) Two
(c) Three
(d) Four
(e) More than four
20. As many as persons are buying things before D as after___?
(a) E
(b) B
(c) F
(d) A
(e) G
Direction (21-23): Study the following information carefully and answer the questions given below:
Point S is 15m west of point R. Point R is 30m south of point Q. Point P is 20m west of point Q. Point U is
15m south of point P. Point T is 35 north of point S.
21. If point V is exactly between point Q and R, then how far and in which direction is point U with respect
to V?
(a)15m, North-East
(b) 15m, East
(c) 10m, North-west
(d) 20m, West
(e) 20m, North-East
22. Four of the following are alike in a certain way, so form a
group. Which of the following does not belong to that
group?
(a) P, T
(b) U, Q
(c) R, P
(d) S, Q
(e) U, T
23. If point W is in 5m east of point U, then what is the
distance between point W and Point S?
(a) 5m
(b) 15m
(c) 25m
(d) 10m
(e) 20m
Direction (24-26): Study the following information carefully and answer the questions given below:
A certain number of persons are sitting in the row. All of them are facing towards north. Q sits sixth from
the right of S. T sits forth to left of Q. Only two persons sit between Q and P. R sits forth to the left of S. U
sits between S and T. V sits second to the right of U. W is third from any of the end. S is eight from the left
end of the row. Six persons sit between W and V.
24. How many numbers of persons could sit in the row?
(a) 14
(b) 18
(c) 23
(d) 15
(e) 20
25. What is the position of ‘W’ with respect to Q?
(a) Fourth to the right
(b) Fifth to the right
(c) Fourth to the left
(d) Eighth to the right
(e) Sixth to the left
26. If X sits immediate right of V then how many persons sit between X and P?
(a) Five
(b) Six
(c) Four
(d) Three
(e) None of these
27. If it is possible to make only one meaningful word with the 2nd ,4th, 6th and 7th letters of the word
‘UNILATERAL’ which would be the second letter of the word from the right end? If more than one
such word can be formed give ‘Y’ as the answer. If no such word can be formed, give ‘Z’ as your
answer.
(a) Y
(b) N
(c) L
(d) T
(e) Z
Directions (28-30): In each of the question, relationships between some elements are shown in the
statements. These statements are followed by conclusions numbered I and II. Read the statements and give
the answer.
(a) If only conclusion I follows.
(b) If only conclusion II follows.
(c) If either conclusion I or II follows.
(d) If neither conclusion I nor II follows.
(e) If both conclusions I and II follow.
28. Statements: A ≥ B ≥ C = D > E ≤ F < G
Conclusions: I. E < B
II. G > E
29. Statements: P≤R<T=U; Q≥T≤S≥V
Conclusions: I. Q > P
II. V < R
30. Statements: L > M = O ≥ P ; N ≤ M ≥ S ≥ T
Conclusions: I. T ≤ P
II. N < L
Directions (31-35): Study the following sequence and answer the given questions.
A@34%ENM$86&LDS#986QYZ17%ROG@2IB2U&
31. Which of the following element is sixth to the left of the fourteenth from the left end of the given
arrangement?
(a) 6
(b) %
(c) $
(d) M
(e) None of these
32. If all the symbols are dropped from the series, which element will be twelfth from the right end?
(a) 9
(b) Q
(c) R
(d) Y
(e) None of these
33. How many such numbers are there in the given series which are immediately preceded by a symbol
and followed by a letter?
(a) None
(b) One
(c) Two
(d) Three
(e) Four
34. How many such letters are there in the given series which are immediately preceded by number and
immediately followed by a symbol?
(a) One
(b) Two
(c) Three
(d) More than three
(e) None of these
35. Find the odd one out?
(a) N64
(b) D86
(c) Y%8
(d) R27
(e) 8EL
Numerical Ability

Directions (36-40): What will come in the place of question mark (?) in the following number series:
36. 11, ?, 16, 21, 29, 41
(a) 12
(b) 14
(c) 15
(d) 13
(e) 11
37. 1800, ?, 60, 15, 5, 2.5
(a) 300
(b) 600
(c) 120
(d) 240
(e) 360
38. 4, 3, 4, 9, 32, ?
(a) 75
(b) 155
(c) 125
(d) 175
(e) 165
39. ?, 100, 150, 375, 1312.5
(a) 50
(b) 100
(c) 75
(d) 25
(e) 200
40. 0, 6, 24, 60, ?, 210
(a) 130
(b) 170
(c) 90
(d) 120
(e) 150
Directions (41-45): Study the bar chart given below and answer the following questions.
Bar chart shows the number of books read by 4 different persons (A, B, C & D) in 2005 and 2006.
100

90

80

70

60
2005
50 2006

40

30

20

10
A B C D

41. Find average number of books read by A, C & D in 2005.


(a) 64
(b) 70
(c) 75
(d) 60
(e) 56
42. Find ratio of books read by B & C together in 2005 to books read by A & D together in 2006.
(a) 15 : 16
(b) 5 : 6
(c) 1 : 5
(d) 4 : 7
(e) 2 : 3
43. Books read by A & D together in 2005 are what percent
more than books read by C in 2006?
(a)
(b)
(c)
(d)
(e)
44. Books read by A & C together in 2005 are how much
more or less than books read by B & D together in
2006?
(a) 24
(b) 14
(c) 18
(d) 22
(e) 28
45. Books read by B & C together in 2006 are what percent of books read by B in 2005?
(a) 100%
(b) 120%
(c) 250%
(d) 200%
(e) 160%
Directions (46-55): - What will come in place of (?) question mark in the following questions?
46. = 200
(a) 120
(b) 1.20
(c) 12
(d) 0.12
(e) None of these
47. 486 ÷ ? × 7392 ÷ 66 = 1008
(a) 54
(b) 55
(c) 52
(d) 53
(e) 51
48. of 4200 ÷ √ ( )
(a) 125
(b) 225
(c) 25
(d) 5
(e) 625
49.
(a) 1208
(b) 1108
(c) 1008
(d) 1128
(e) 1348
50. (0.05 × 6.25) ÷ 2.5 =?
(a) 12.55
(b) 0.125
(c) 0.115
(d) 1.25
(e) None of these
51. 1496 ÷ 17 = ?% of 220
(a) 25
(b) 40
(c) 50
(d) 75
(e) None of these
52. (36% of 180) ÷ 0.4 = ?
(a) 160
(b) 164
(c) 166
(d) 162
(e) 180
53. 0.08% of 55500 – 16.4 = ?
(a) 26.6
(b) 28
(c) 29.2
(d) 30.4
(e) 32
54. 35% of 150 × 16 = ? – 22
(a) 865
(b) 932
(c) 864
(d) 862
(e) None of these
55. (3080 + 6160) ÷ ? = 330
(a) 26
(b) 22
(c) 28
(d) 29
(e) 18
56. Difference of the compound interest received in first year and second year at 20% per annum at CI is
Rs 1200 then find the sum?
(a) Rs 25,000
(b) Rs 36,000
(c) Rs 35,000
(d) Rs 24,000
(e) Rs 30,000
57. Find the total distance covered by boat in each
upstream and downstream in 7 hours if the speed of
boat in still water and speed of current is 21 km/h and
3 km/h respectively?
(a)
(b)
(c)
(d)
(e)
58. Ratio of income of A to that of B is 5:9. If expenditure of
A is th of his income and expenditure of B is th of his income and sum of their saving is Rs 1950 then
find the difference between their income?
(a) Rs 900
(b) Rs 1000
(c) Rs 880
(d) Rs 960
(e) Rs 920
59. A alone can do a work in 12 days while A and B together can do that work in 7.5 days. Find the time
taken by C alone to do that work if C takes 3 days more than that of B alone to do that work?
(a) 33 days
(b) 30 days
(c) 23 days
(d) 27 days
(e) 28 days
60. Ratio of base and perpendicular side of a right-angled triangle is 3:4 and its base is equal to the side
of a square having area 81 cm2. Find the perimeter of the triangle?
(a) 30 cm
(b) 36 cm
(c) 33 cm
(d) 42 cm
(e) 40 cm
Directions (61-65): In each of these questions, two equations (I) and (II) are given. You have to solve
both the equations and give answer
(a) if x y
(b) if x y
(c) if x y
(d) if x y
(e) if x = y or no relation can be established between x and y.
61. I. x² - 13x + 40 = 0
II. 2y² - y – 15 = 0
62. I. 5x² + 17x + 6 = 0
II. 2y² + 11y + 12 = 0
63. I. 7x² - 19x + 10 = 0
II. 8y² + 2y – 3 = 0
64. I.
II.
65. I. 3x² –7x + 4 = 0
II. 2y² – 9y + 10 = 0
66. A person travels half of the distance at the speed of x km/h and remaining half of the distance at 4x
km/h. Find the value of ‘x’ if the average speed is 36.8 km/h?
(a) 21
(b) 25
(c) 24
(d) 23
(e) 20
67. A, B and C invested in a ratio of 7: 8: 5 in a business. They got an annual profit of Rs. 136800. If A and
C withdrew their amount at the end of 3 months and 7 months respectively. Then find the difference
between A and C’s share of profit?
(a) Rs. 12,600
(b) Rs. 11,500
(c) Rs. 13,500
(d) Rs. 10,500
(e) Rs. 13,000
68. Retailer sold one article at profit and another at 100% profit. Find his overall profit
percentage if the selling price of both the article is same?
(a) 60%
(b) 55%
(c)
(d) 75%
(e)
69. A mixture has milk and water in the ratio 4: 1. When 50% of the mixture is taken out and replaced by
24 liters of water then the ratio of milk to water in the mixture becomes 1: 1. Find initial quantity of
mixture.
(a) 80 liters
(b) 45 liters
(c) 70 liters
(d) 60 liters
(e) 75 liters
70. 4 years ago, ratio of Shivam’s age to Deepak’s age was 2: 3 and ratio of Shivam’s age 4 years ago to
Deepak’s age 5 years hence is 8: 15. Find present age of Shivam.
(a) 32 years
(b) 28 years
(c) 40 years
(d) 24 years
(e) 36 years
English Language

Directions (71-78)- Read the following passage and answer the following questions based on the given
passage. Some of the words are highlighted which would help you to answer some of the questions given.

For years, world-wide organisations have become increasingly excited about the prospect of a cloud-based
future. As the dream becomes an ever closer reality for many kinds of business and, Forrester predicted
that enterprise spending on cloud services is set to surge. IDC also predicted that global spending on public
cloud services and infrastructure would reach $210bn in 2019, an increase of 24% from 2018. But one
obstacle stands create friction and introduce risk: the process of migration.
As all indications point to a massive shift in data deployments to the cloud, it is more important than ever
that the transition from on-premises to Cloud is as risk free as possible. In today's climate any loss or
disruption to data can have a huge business impact. It’s a complex process, is frequently underestimated
and many organisations have found there’s lots that can go wrong that can impact the business.
Organisations across the globe have found the cloud to be an ideal place to run modern data applications
due to big data’s elastic resource requirements. Furthermore, with the lack of data talent an ever-looming
issue for most companies today they have been determined to adopt a cloud-first strategy to ensure
business operations are accessible for a range of employees.
The cloud offers great promise for developers especially, as it can increase the speed at which they develop
software features and increase the resilience of applications once they are deployed - along with enhanced
security through the use of multiple server locations. With all this considered, it is no surprise that 42% of
UK businesses leverage some kind of cloud service, according to Eurostat.
However, all the perceived benefits of leveraging the cloud are redundant if organisations come up against
barriers to accessing cloud services. Cloud-based data pipelines still suffer from complexity challenges at
the moment, along with the lack of visibility into cost and resource usage at the application and user level.
The answer to this is automation fueled by robust Machine learning training models and artificial
intelligence. These concepts and the tools that enable them can determine the prerequisites of cloud
infrastructure, application dependencies,the appropriate target cloud instance profiles, and provide
troubleshooting in real-time.
To summarise, the promise of the cloud has created a sense of excitement amongst enterprises, however,
they have proceeded to go full steam ahead into adopting a cloud service, without sufficient data to
ensure performance service level agreements (SLAs).
71. How can we tackle the risks associated with the process of migration?
(a) By ensuring that organisations only migrate the apps to the cloud that will thrive in the cloud.
(b) The surge in the investment in the cloud technology.
(c) The use of predictive power of Artificial Intelligence.
(d) The transition from on-premises to Cloud
(e) None of these
72. Why the promise of the cloud has created a sense of excitement amongst enterprises?
(a) As it ensures that business operations are accessible for a range of employees.
(b) As the enterprise spending on cloud services is surging.
(c) Reduction in the cost of operation it may entail.
(d) The secure platform it provides to the data it stores.
(e) None of these
73. How the process of migration stands as an obstacle to the cloud technology?
(a) The expensive cost of providing safety to the data.
(b) The lack of sound technology in Artificial Intelligence.
(c) Complexities of the data migrated to the cloud
(d) It exposes the companies to the risk of losing important information.
(e) None of these
74. Which of the following statement(s) is/are correct in context of the passage?
(1) The cloud technology can help in increasing the speed at which the software features are
developed.
(2) There is no risk associated with the cloud technology.
(3) Cloud technology is not a very complex process.
(a) Only (2)
(b) Both (1) and (2)
(c) Only (3)
(d) Only (1)
(d) All are correct
75. What is the meaning of the phrase “full steam ahead” ?
(a) Interrupting an activity
(b) Acting cautiously
(c) Great increase
(d) Progressing quickly.
(e) Prudent progression
76. Which of the following words is most similar to the word “ADOPT” as given in the passage?
(a) mold
(b) deduce
(c) Embrace
(d) Creative
(e) Fluster
77. Which of the following words is most similar to the word “ENSURE” as given in the passage?
(a) Confirm
(b) Paced
(c) Redundant
(d) Weaken
(e) Reject
78. Which of the following words is most OPPOSITE to the word
“MASSIVE” as given in the passage?
(a) Prominent
(b) Insignificant
(c) Enlarged
(d) Filthy
(e) Vouch
Directions (79-85): Read each sentence to find out if there is any error in it. The error, if any will be in
one part of the sentence. The number of that part is the answer if there is no error, the answer is (e).
79. It is widely believe (a)/ that the village which is situated on the (b)/ eastern coast of India will be
wiped out (c)/ within a decade. (d)/ No error. (e)
80. Yesterday I met (a)/ an old friend (b)/ when I am going (c)/ to the market. (d)/ No error.(e)
81. The criminal was (a)/ sentenced to the (b)/ death and was (c) hung for his crime (d)/ No error.(e)
82. FATF strongly urges (a)/ Pakistan to swiftly complete its action plan (b)/ by October 2019 to stop
(c)/ terror funding in the country (d)/ No error.(e)
83. Elon musk is determined (a)/ to be success (b)/ in whatever field (c)/ he chooses. (d)/ No error.(e)
84. New purchasing power will increasingly (a)/ come from Asia and Africa where (b)/ the demographics
are still favorable (c)/ for high income growth. (d)/ No error (e).
85. Not only the (a)/ students but also (b)/ the principal were(c)/ laughing at the joke he cracked.(d)/No
error (e).
Directions (86-90): In each of the following sentences, there is a blank space. Below in each sentence,
there are five words out of which one can be used to fill the blank to make the sentence grammatically and
coherently correct. Find the most appropriate word that fit into the blank contextually.
86. There is nothing to indicate the building's past, ________ the fireplace.
(a) except
(b) included
(c) belonged
(d) barred
(e) Foster
87. Two circus elephants that performed together ________ when crossing paths 23 years later
(a) happy
(b) believe
(c) angered
(d) rejoiced
(e) greater
88. The law _________ farmers not only to save and resow (multiply) seeds, but also to sell them to other
farmers, no matter what the original source of the seed is.
(a) coverts
(b) implements
(c) emits
(d) permits
(e) deserts
89. Small farmers must be educated and __________ with proper incentive structures, to engage with
agriculture.
(a) fascinated
(b) encouraged
(c) discouraged
(d) harvested
(e) invited
90. Raipur despite being ________________ between two rivers, the Krishna and the Tungabhadra, it is a
dust bowl in May first week.
(a) Surrounded
(b) Located
(c) Revealed
(d) Settle
(e) None of these.
Directions (91-95): Given sentences are not in their exact position. Rearrange them to make a coherent
paragraph and then answer the questions given below.
A. It can even make exercise more fun and productive.
B. Plus, hearing others laugh, even for no apparent reason, can often trigger genuine laughter.
C. To add simulated laughter into your own life, search for laugh yoga or laugh therapy groups. Or you can
start simply by laughing at other people’s jokes, even if you don’t find them funny.
D. A Georgia State University study found that incorporating bouts of simulated laughter into an exercise
program helped improve older adults’ mental health as well as their aerobic endurance.
E. Believe it or not, it’s possible to laugh without experiencing a funny event—and simulated laughter can
be just as beneficial as the real thing.
91. Which of the following should be the FOURTH sentence after the rearrangement?
(a) A
(b) B
(c) C
(d) D
(e) E
92. Which of the following should be the SECOND sentence after the rearrangement?
(a) A
(b) B
(c) C
(d) D
(e) E
93. Which of the following should be the THIRD sentence after the rearrangement?
(a) A
(b) B
(c) C
(d) D
(e) E
94. Which of the following should be the FIRST sentence after the rearrangement?
(a) A
(b) B
(c) C
(d) D
(e) E
95. Which of the following should be the FIFTH sentence after the
rearrangement?
(a) A
(b) B
(c) C
(d) D
(e) E
Directions (96-100): In the following questions, a sentence is
divided into four parts consisting of a highlighted word in each part.
Choose the option reflecting the word which is either misspelt or
grammatically incorrect. If all the highlighted words are correct,
choose option (e) i.e. “all are correct” as your answer choice.

5
96. A British court has ruled that British arms sales to Saudi Arabia wear unlawful on Humanitarian
Grounds.
(a) ruled
(b) sales
(c) wear
(d) Grounds
(e) All are correct
97. I'm tyred of hearing politicians making pious pronouncements about their devotion to the people
(a) tyred
(b) hearing
(c) pious
(d) devotion
(e) All are correct
98. It is likely that weaving prospered and partly relocated towards areas within easy reach of the army
contractors.
(a) prospered
(b) relocated
(c) within
(d) contractors
(e) All are correct
99. This cat and mouse tactic was purposely designed to provoke, inrage and panic the unemployed.
(a) tactic
(b) purposely
(c) inrage
(d) panic
(e) All are correct
100. Indian democracy’s robustness is underscored by high voter turnouts, and large number of
candedates in the fray.
(a) robustness
(b) turnouts
(c) candedates
(d) fray
(e) No correction required.
Solutions

Reasoning Ability

Direction (1-5):

S1. Ans. (a);

S2. Ans. (c);

S3. Ans. (b);

S4. Ans. (c);

S5. Ans. (b);

Direction (6-10):

S6. Ans.(c);

S7. Ans.(b);

S8. Ans.(e);

S9. Ans.(a);

S10. Ans.(d);
S11. Ans.(c);

Direction (12-15):

S12. Ans.(b);
Sol.

S13. Ans.(e);
Sol.

S14. Ans. (b);


Sol.

S15. Ans.(a);
Sol.
Direction (16-20):
Persons
E
D
A
B
C
G
F

S16. Ans.(c);

S17. Ans.(e);

S18. Ans.(c);

S19. Ans.(b);

S20. Ans.(e);

Direction (21-23):
S21. Ans. (d);
Sol.

S22. Ans.(c);
Sol.
S23. Ans.(b);
Sol.

Direction (24-26):

S24. Ans.(e);

S25. Ans.(a);

S26. Ans.(c);

S27. Ans.(b);
Sol.
One word formed- lent

Direction (28-30):

S28. Ans.(e);
Sol.

I. E < B (True)

II. G > E (True)

S29. Ans.(a);
Sol.

I. Q > P (True)

II. V < R (False)


S30. Ans.(b);
Sol.

I. T ≤ P (False)

II. N < L (True)

Direction (31-35):

S31. Ans.(d);
Sol.
M

S32.Ans.(d);
Sol.
Y

S33. Ans.(b);
Sol.
One–@2I

S34. Ans.(a);
Sol.
One – 2U&

S35. Ans.(e);
Sol.
8EL
Numerical Ability
S36. Ans (d)
Sol.

S37. Ans (a)


Sol.

S38. Ans (b)


Sol.

S39. Ans (e)


Sol.

S40. Ans (d)


Sol.
S41. Ans.(b)
Sol.
72+90+48
Required average = 3
= 70

S42. Ans.(a)
Sol.
60+90
Required ratio = 90+70
150
= 160
= 15: 16

S43. Ans.(e)
Sol.
{(72+48)−72}
Required % = × 100
72
200
= %
3
2
= 66 3 %

S44. Ans.(b)
Sol.
Required difference = (72 + 90) – (78 + 70)
= 162 – 148
= 14

S45. Ans.(c)
Sol.
78+72
Required % = 60 × 100
150
= 60 × 100
= 250%

S46. Ans (d)


Sol.
17.28 ÷? = 200 × 3.6 × 0.2
17.28
?= 144
= 0.12

S47. Ans (a)


Sol.
486 7392
× = 1008
? 66
486 1008
=
? 112
486
?= 9
? = 54
S48. Ans (e)
Sol.
1
100 1
× 4200 × 24 = (? )2
700
1
25 = (? )2
? = 625

S49. Ans (c)


Sol.
90×7×8
? = 5 = 1008

S50. Ans (b)


Sol.
0.3125
? = 2.5
? = 0.125

S51. Ans. (b)


Sol.
1496 ?
= 100 × 220
17
? = 40

S52. Ans (d)


Sol.
36 10
× 180 × =?
100 4
? = 162

S53. Ans (b)


Sol.
0.08 × 555 − 16.4 =?
? = 44.4 − 16.4
? = 28

S54. Ans (d)


Sol.
35
× 150 × 16 + 22 =?
100
? = 840 + 22 = 862

S55. Ans (c)


Sol.
9240
= 330
?
? = 28
S56. Ans.(e)
Sol.
Let the sum be Rs 100x
CI in first year= Rs 20x
CI in two years= 44% of 100x= Rs 44x
CI in 2nd year= 44x-20x= Rs 24x
ATQ
24x-20x=1200
x=300
Required sum=Rs 30,000

S57. Ans.(b)
Sol.
Speed in upstream=18 km/hr
Speed in downstream= 24 km/hr
Required total distance= (24 + 18) × 7 = 294 𝑘𝑚

S58. Ans.(d)
Sol.
Let income of A and B be Rs 5x and Rs 9x respectively
15
Expenditure of A=Rs 8 𝑥
25
Saving of A=𝑅𝑠 𝑥
8
Expenditure of B= Rs 4x
Saving of B= Rs 5x
ATQ
65
𝑥 = 1950
8
x=240
required difference= Rs 960

S59. Ans.(c)
Sol.
Let total work be 60 units (LCM of 12 and 7.5)
Efficiency of A= 5 units/ day
Efficiency of A and B together= 8 units/ day
Efficiency of B= 3 units/ day
Time taken by B alone to do that work=20 days
Time taken by C alone=23 days

S60. Ans.(b)
Sol.
Side of the square=9 cm
Perpendicular side of the triangle= 12 cm
Hypotenuse of the triangle=√81 + 144 = √225 = 15 𝑐𝑚
Perimeter of the triangle= 36 cm
S61. Ans.(a)
Sol.
I. x² -13x + 40= 0
x² - 5x – 8x + 40 = 0
x (x -5) – 8 (x - 5) = 0
x = 5, 8
II. 2y² - y – 15 = 0
2y² - 6y + 5y – 15 = 0
2y (y - 3) + 5 (y - 3) = 0
y=3, -5/2
x>y

S62. Ans.(e)
Sol.
I. 5x² + 17x + 6 = 0
5x² + 15x + 2x + 6 = 0
5x (x + 3) +2(x + 3) = 0
2
𝑥 = −3, −
5

II. 2y² + 11y + 12 = 0


2y² + 8y + 3y + 12 = 0
2y (y + 4) + 3 (y + 4) = 0
3
𝑦 = −4, −
2
No relation

S63. Ans.(a)
Sol.
7x² - 19x + 10 = 0
7x² - 14x – 5x + 10 = 0
7x (x - 2) – 5 (x - 2) = 0
5
x = 2, 7
II. 8y² + 2y – 3 = 0
8y² + 6y – 4y – 3 = 0
2y (4y + 3) – 1 (4y + 3) = 0
−3 1
𝑦= ,
4 2
x >y
S64. Ans.(a)
Sol.
I. 𝑥 2 − 8𝑥 + 15 = 0
⇒ 𝑥 2 − 5𝑥 − 3𝑥 + 15 = 0
⇒ 𝑥 (𝑥 − 5) − 3(𝑥 − 5) = 0
⇒ (𝑥 − 3)(𝑥 − 5) = 0
∴ 𝑥 = 3 𝑜𝑟 5
II. 𝑦 2 − 3𝑦 + 2 = 0
⇒ 𝑦 2 − 2𝑦 − 𝑦 + 2 = 0
⇒ 𝑦 (𝑦 − 2) − 1(𝑦 − 2) = 0
⇒ (𝑦 − 1)(𝑦 − 2) = 0
∴ y = 1 or 2
∴𝑥>𝑦

S65. Ans.(c)
Sol.
I. 3x² –7x + 4 = 0
⇒ 3x²– 4x - 3x +4 = 0
⇒ (3x – 4) (x -1) = 0
4
𝑥 = 3 𝑜𝑟 1
II. 2y² -9y + 10 = 0
⇒ 2y² - 4y - 5y + 10 = 0
⇒ (2y - 5) (y -2) =0
5
⇒ y = 2 or 2
𝑦>𝑥

S66. Ans.(d)
Sol.
Let the distance be D km
ATQ
𝐷
𝐷 𝐷 = 36.8
( + )
2𝑥 8𝑥
x=23

S67. Ans.(a)
Sol.
Ratio of their profit sharing
A: B: C = 7 × 3 ∶ 8 × 12 ∶ 5 × 7 = 21 ∶ 96 ∶ 35
Annual profit = 136800
Difference b/w A and C’s share of profit
14
= × 136800
152
= Rs 12,600
S68. Ans.(a)
Sol.
Let SP of both article = 8x
ATQ,

16𝑥−10𝑥
Profit % = × 100 = 60%
10𝑥

S69. Ans.(a)
Sol.
Let initial quantity of milk and water in the mixture be ‘40x liters’ and ‘10x liters’ respectively.
ATQ,
1
(40𝑥× ) 1
2
1 =1
10𝑥× +24
2
20𝑥 1
=1
5𝑥+24
20x = 5x + 24
15x = 24
x = 1.6
So, required quantity = 40x + 10x
= 50x
= 80 liters

S70. Ans.(b)
Sol.
Let age of Shivam and Deepak 4 years ago be ‘2x years’ and ‘3x years’ respectively.
ATQ,
2𝑥 8
= 15
3𝑥+4+5
2𝑥 8
= 15
3𝑥+9
30x = 24x + 72
6x = 72
x = 12
So, present age of Shivam = 2x + 4
= 28 years
English Language
S71. Ans.(c)
Sol.
Refer the last few lines of the 5th paragraph of the passage, “The answer to this is automation fueled by
robust Machine learning training models and artificial intelligence……………. provide troubleshooting in
real-time.”

S72. Ans.(a)
Sol.
Refer the 3rd paragraph of the passage, “Furthermore, with the lack of data talent an …………..ensure
business operations are accessible for a range of employees.”

S73. Ans.(d)
Sol.
Refer to the 2nd line of the 2 nd paragraph of the passage, “In today's climate any loss or disruption to data
can have a huge business impact.”

S74. Ans.(d)
Sol.
Statement (1) is correct. Refer the 4th paragraph of the passage, “The cloud offers great promise for
developers …………….the use of multiple server locations”

S75. Ans.(c)
Sol.
If something such as a plan or a project goes full steam ahead, it progresses quickly.

S76. Ans.(c)
Sol.
Embrace means accept (a belief, theory, or change) willingly and enthusiastically hence it is similar in
meaning to adopt.

S77. Ans.(a)
Sol.
Ensure means make certain that (something) will occur or be the case hence confirm is the word which is
most similar in meaning.
S78. Ans.(b)
Sol.
Massive means exceptionally large hence insignificant is the word which is most opposite in meaning.

S79. Ans.(a)
Sol.
There is an error in part (a) of the sentence in place of “believe” we will use “believed”.

S80. Ans.(c)
Sol.
There is an error in part (c) of the sentence.
In place of “am” we will use “was” because the events of the sentences are in past.

S81. Ans.(d)
Sol.
Use “hanged” in place of “hung”

S82. Ans.(e)
Sol.
There is no error in the sentence.

S83. Ans.(b)
Sol.
There is an error in part (b) of the sentence. Use “to succeed” in place of “to be success” because after
“determined” we use “to infinitive”.

S84. Ans.(e)
Sol.
There is no error in the sentence.

S85. Ans.(c)
Sol.
Replace 'were' with was'.
When two subjects are joined by 'not only...but also', the verb must agree with the second subject.
The correct sentence should be: “Not only the students but also the
principal was laughing at the joke he cracked.
The same rule applies when two subjects are joined by 'or', the verb must
agree with the second subject.
For e.g. Krish and his brothers were there. (Second subject is 'plural')
Mohan or Sohan is responsible for this. (Second subject is 'singular')

S86. Ans.(a)
Sol.
“Except” which means “not including” is the word which should fit the
blank.
S87. Ans.(d)
Sol.
“Rejoiced” which means feel or show great joy or delight will fit the blank most appropriately.

S88. Ans. (d)


Sol.
Here, “permits” which means “officially allow (someone) to do something” perfectly fits in the given blank
making the sentence grammatically correct and contextually meaningful. Hence, option (d) is the most
suitable answer choice.
Coverts: a thicket in which game can hide.
Implements: put (a decision, plan, agreement, etc.) into effect.
Emits: produce and discharge (something, especially gas or radiation).
Deserts: what a person deserves with regard to reward or (more usually) punishment.

S89. Ans.(b)
Sol.
Here, “encouraged” which means “give support, confidence, or hope to (someone)” perfectly fits in the
given blank making the sentence grammatically correct and contextually meaningful. Hence, option (b) is
the most suitable answer choice.
Fascinated: strongly attracted and interested.
Discouraged: having lost confidence or enthusiasm; disheartened.
Harvested: gather (a crop) as a harvest.
Invited: make a formal or polite request for (something) from someone

S90. Ans.(b)
Sol.
‘Surrounded’ should be followed by ‘By’ or ‘with’, hence it is an incorrect option. Also, option (d) is
grammatically incorrect and option (c) doesn’t make a meaningful sentence. So, option (b) is the only
option which makes the sentence both meaningfully and grammatically correct. Hence, option (b) is the
most suitable answer choice.

S91. Ans.(b)
Sol.
The correct sequence for the given sentences is EADBC.

S92. Ans.(a)
Sol.
The correct sequence for the given sentences is EADBC.

S93. Ans.(d)
Sol.
The correct sequence for the given sentences is EADBC.

S94. Ans.(e)
Sol.
The correct sequence for the given sentences is EADBC.
S95. Ans.(c)
Sol.
The correct sequence for the given sentences is EADBC.

S96. Ans.(c)
Sol.
“Were” should be used in place of “wear”

S97. Ans.(a)
Sol.
Correct Spelling is Tired

S98. Ans.(e)
Sol.
All the given words are correct.

S99. Ans.(c)
Sol.
The correct spelling is “enrage”

S100. Ans.(c)
Sol.
From the given highlighted words, ‘Candedates’ is misspelled, correct spelling is ‘Candidates’ Hence,
option (c) is the correct answer.
Part - 5

Directions (1-5): Read the following passage carefully and answer the questions given below them.
Certain words/phrases have been printed in bold to help you locate them while answering some of
the questions.

Life is messy. If we want our best and brightest in the workforce, we need to accept that they have
complex lives. We need to be flexible when it comes to the realities of balancing career and family.
Being flexible at work doesn’t just benefit people trying to balance their outside lives with work. An
extensive body of research demonstrates the business benefits of flexible working. Yet despite this
overwhelming evidence, access to flexible work and careers is not widespread. Flexible work is still
regarded as an add-on, something we do for mothers for a few months when they are back from
parental leave. But in the face of rapid changes to the way we work, organizations need to move beyond
just having policies for flexible working or making ad-hoc adjustments for certain individuals.
Companies need to fundamentally rethink the way they design work and jobs.
The World Economic Forum predicts that we are on the cusp of a fourth industrial revolution.
Technological, socioeconomic and demographic shifts are transforming the way we work, demanding
flexibility in the way individuals, teams and organizations work. We all have different things
happening in our lives at different times. Not just caring for young kids, but other family members,
community roles, study and volunteering. And all of these parts of our identities bring with them
different skill sets. In today’s workforce, fewer people identify with the stereotype of the ideal worker
– a full-time, fully committed employee without personal or family commitments that impact on
availability.
There are a few factors driving the demand for increased flexibility. Globalization is one. The
development of a 24/7 marketplace, and the rapid expansion of the services economy are also having
a transformational effect on the workplace, requiring organizations to think creatively about how they
can best organize jobs and work to respond to an increasingly diverse and demanding customer base.
Similarly technology is driving – and enabling – greater flexibility. It is dramatically reshaping our
workplaces, blurring the boundaries between work and home and diversifying where, when and how
employees work. Advances in mobile, internet and cloud technologies, the rapid development of
computing power, and the digital connection between multiple objects have all driven workplace
innovations such as remote working, telecommuting, co-working
spaces, video/teleconferencing, and virtual teams and
collaboration.
So the future of work demands new approaches to work design –
but have workplaces risen to the challenge? The evidence suggests
we have yet to grasp this opportunity to be more innovative.
While some employers are making flexible work more available,
there is still a high prevalence of bolted-on temporary
arrangements. These arrangements are seen as the exception to the
rule, with the full-time, “face-time”, long hours “ideal worker” still
the model to which everyone is expected to adhere.
Many people make assumptions about flexible workers, including that they’re not interested in
training and development, aren’t committed to the organization, or don’t have any career aspirations.
We need to explore and challenge these biases.
There are good international examples of successful work redesign that have involved the input of a
team of employees. For example, a UK bakery sat down with their bakers and came up with a flexible
system of two to three baking shifts a day to maintain a steady supply of fresh bread. The team agreed
to rotate their hours each week so no team member permanently worked a shift that did not suit. After
the change was made, bakery sales increased by more than 65% in the first year and employee
satisfaction in the bakery has risen 10% since the change to 93%. So work redesign is not only doable,
it can deliver business benefits, although it does require a completely new approach. By changing our
thinking and focusing on the team and the organization as a whole, rather than the individual, we have
the opportunity to create more adaptable and sustainable workplaces.

Q1. According to the author, how is flexible working still regarded as?
(a) the way of designing work and jobs.
(b) the way to identify the stereotype of an ideal worker.
(c) an add-on, something we do for mothers for a few months when they are back from parental leave.
(d) advances in mobile, internet and cloud technologies and the rapid development of computing
power.
(e) None of these.

Q2. According to the passage, in today’s workforce, which among the following is/are the
stereotype(s) of an ideal worker?
(I) a full time worker
(II) an employee who is fully committed to work
(III) the employee has no personal or family commitments that impact his availability
(a) Only (II)
(b) Only (III)
(c) Both (II) and (III)
(d) Both (I) and (III)
(e) All of these.

Q3. According to the passage, how is technology driving and enabling greater flexibility?
(a) It is identifying the stereotype of an ideal worker.
(b) It is dramatically reshaping our workplaces, blurring the boundaries between work and home and
diversifying where, when and how employees work.
(c) It is training the workers on how to transform the ways in which flexibility can be introduced
(d) It is one of the factors that is contributing in declining the demand for flexibility at the workplace
worldwide.
(e) None of these.
Q4. Which of the following were the steps taken by the UK Bakery with regards to the working time
of their bakers?
(I) The UK Bakery came up with a flexible system of two to three baking shifts a day to maintain a
steady supply of fresh bread.
(II) The Bakery team agreed to rotate their hours each week so no team member permanently worked
a shift that did not suit.
(III) The bakery sales increased by more than 65% in the first year and employee satisfaction in the
bakery has risen 10% since the change to 93%.
(a) Only (III)
(b) Both (I) and (III)
(c) Both (II) and (III)
(d) Both (I) and (II)
(e) All of these.

Q5. The most appropriate title of the passage is.


(a) Complex lives of the workforce
(b) Role of technology in increasing flexibility of workers
(c) The brightest workforce requires more flexible work
(d) How a UK Bakery increased their sales
(e) Importance of Globalization in increasing workforce

Directions (6): Choose the word which is most nearly the SAME in meaning as the word printed in
bold as used in the passage.

Q6. Predict
(a) ignore
(b) forecast
(c) misunderstand
(d) recount
(e) narrate

Directions (7): Choose the word which is most nearly the


OPPOSITE in meaning as the word printed in bold as used in
the passage.

Q7. Temporary
(a) substitute
(b) alternate
(c) expedient
(d) provisional
(e) permanent
Directions (8-15): Read each sentence to find out whether there is any grammatical or idiomatic error
in it. The error, if any, will be in one part of the sentence. The alphabet corresponding to that part
is your answer. If there is ‘No error’, the answer is (e). (Ignore errors of punctuation, if any.)

Q8. The priest together with (A)/his followers were fatally injured (B)/in the accident which
occurred last night (C)/near the unmanned railway crossing. (D)/No Error (E)
(a) A
(b) B
(c) C
(d) D
(e) E

Q9. The young woman who is watching television in that room (A)/lived here for more than (B)/a
year but she has never created (C)/any problem for us. (D)/No Error (E)
(a) A
(b) B
(c) C
(d) D
(e) E

Q10. Each of the employees, (A)/whom the company has chosen to take part (B)/in the international
seminar to be conducted (C)/in the City Hall, are up to the mark. (D)/ No Error (E)
(a) A
(b) B
(c) C
(d) D
(e) E

Q11. Needless to say, (A)/no sooner were all these large and rather expensive operations finished
(B)/when the main electricity was brought in (C)/and the turbine became obsolete. (D)/No Error (E)
(a) A
(b) B
(c) C
(d) D
(e) E

Q12. Plastic bags less than 50 microns thick (A)/are banned, (B)/but neither the states nor the city
corporations (C)/cares to enforce this rule. (D)/No Error (E)
(a) A
(b) B
(c) C
(d) D
(e) E
Q13. Tribal angst over economic issues (A)/leading to the scapegoating of nontribal longtime
residents (B)/reflects the continued failure (C)/to forge a more inclusive politics in Meghalaya.
(D)/No Error (E)
(a) A
(b) B
(c) C
(d) D
(e) E

Q14. The Prime Minister has great power of (A)/implementing some useful (B)/schemes but the
ministers (C)/have even greatest ability to foil them. (D)/No Error (E)
(a) A
(b) B
(c) C
(d) D
(e) E

Q15. The blunder mistake (A)/was the apparent failure of detectives (B)/to inform the Parole Board
that the murderer (C)/had threatened to return to kill her. (D)/No Error (E)
(a) A
(b) B
(c) C
(d) D
(e) E

Directions (16-20): In each question below some sentences are given which are divided into five
parts. The first part of the sentence (1) is correct and is given in bold followed by four parts named
A, B, C and D. Rearrange the four parts of the sentence to make a coherent paragraph. The rearranged
sequence of the parts will be your answer. If the given sentence is correct as it is then choose option
(e).

Q16. Crashes in the early days (1)/ to be caused by technical faults,


(A)/ of commercial jets tended (B)/ such as metal fatigue (C) /in
the airframe or engines (D)
(a) DCAB
(b) ABCD
(c) BACD
(d) CABD
(e) No arrangement required
Q17. There have been a lot of (1)/ drivers not obeying (A)/ complaints recently about (B)/ in
downtown Boston (C)/ the speed limits (D)/
(a) BADC
(b) ABCD
(c) BCAD
(d) CABD
(e) No arrangement required

Q18. Chinese officials say (1)/ dropped to a three-year (A)/ low because of (B)/ economic growth has
(C)/ the world economy (D)
(a) BADC
(b) CABD
(c) BCAD
(d) CABD
(e) No arrangement required

Q19. I think it's a shame that (1)/ some foreign language teachers(A)/ studied with a native speaker
(B)/ college without ever having (C)/ were able to graduate from (D)
(a) BADC
(b) CABD
(c) BCAD
(d) ADCB
(e) No arrangement required

Q20. The Gita is a spiritual philosophy (1)/ addressed to all and we know that (A)/ there are all kinds
of people, (B)/ each kind differing (C)/ quite significantly from the other (D)
(a) BADC
(b) CABD
(c) BCAD
(d) ADCB
(e) No arrangement required

Directions (21-25): Given below are sentences consisting a blank in each. Identify the most suitable
alternative among the five given that fits into the blank to make the sentence logical and
meaningful.

Q21. In the same amount of time it would take me to correct all the ________________ in your report,
I could write a better report myself.
(a) mistakes
(b) problems
(c) accuracies
(d) obstacles
(e) disputes
Q22. I have recently used the services of his ________________ agency to book a cruise in the
Mediterranean.
(a) progress
(b) deportation
(c) travel
(d) transfer
(e) mover

Q23. They would like local authorities to be given greater ________________ as to how the money is
spent.
(a) affairs
(b) function
(c) omission
(d) discretion
(e) statement

Q24. In a 10-billion-year-old galaxy there should have been ample ________________ for at least one
species to escape its own mess, and to spread across the stars, filling every niche.
(a) negligence
(b) opportunity
(c) surveillance
(d) supply
(e) advocacy

Q25. A true ________________ of the resources involved in sport would include the unpaid labour
services.
(a) growth
(b) consideration
(c) guidance
(d) suggestion
(e) estimation

Directions (26-30): In the question given below, there is a sentence in which one part is given in
bold. The part given in bold may or may not be grammatically correct. Choose the best alternative
among the four given which can replace the part in bold to make the sentence grammatically correct.
If the part given in bold is already correct and does not require any replacement, choose option (e),
i.e. “No replacement required” as your answer.

Q26. Despite of being most efficient method ever, it is still highly inefficient, and this inefficiency
inspires hope.
(a) Despite being the mostly efficient
(b) Despite of being a most efficient
(c) Despite of being the most efficient
(d) Despite being the most efficient
(e) no replacement required
Q27. A satisfactorily number of contestant must register for the
contest in order for it to take place.
(a) satisfactory number of contestants
(b) satisfaction of number of contestants
(c) satisfaction in the number of contestants
(d) satisfactory number of contestant
(e) no replacement required

Q28. The next class of wave or oscillation detector is the magnetic


detector depending in the powers of electric oscillations to affect
the magnetic state of iron.
(a) depend on the power in
(b) depending upon the power of
(c) depends upon the power in
(d) deepening upon the power of
(e) no replacement required

Q29. James had teaching at the university for more than a year before he left for Asia.
(a) was taught
(b) had been taught
(c) had been teaching
(d) has been teaching
(e) no replacement required

Q30. His tail was short and scraggly, and his harness had been broken in many places and fastened
together again with cords and bits of wire.
(a) was broke from
(b) has broke from
(c) have been broken in
(d) have been breaking on
(e) no replacement required
Direction (31-33): Read the information carefully and answer the questions:

A company ABC printed different number of books in different years 1947, 1956, 1987, 1998, 2002
such that number of books printed are not same in any year. 66 books were printed in an odd
numbered year which is not 1947.The number of books printed in 1947 is 10 less than that printed in
1987. 59 books were printed in an year before the year in which 61 books are printed but not
immediate before. The number of books printed in 2002 is 2 more than that printed in 1998.

Q31. How many books were printed in 1947?


(a) 56
(b) 66
(c) 63
(d) 61
(e) none of these

Q32. What is the difference between the number of books printed in 1956 and 2002?
(a) 7
(b) 10
(c) 8
(d) 4
(e) none of these

Q33. In how many years the number of books printed are more than that printed in 1998?
(a) two
(b) one
(c) none
(d) three
(e) four

Q34. How many words can be formed from the 1 st, 6th, 8th and 9th
letter of a word ‘EMANICIPATE’ by using each letter once in
the word?
(a) two
(b) one
(c) none
(d) three
(e) more than three
Q35. If all the letters in the word FIGURES are arranged in alphabetical order from left to right in
such a way that vowels are arranged first followed by consonants, then how many letters are there
in between U and R after the arrangement?
(a) two
(b) one
(c) none
(d) three
(e) four

Q36. If in the number 39682147, 1 is added to each of the digit which is less than five and 1 is
subtracted from each of the digit which is greater than five then how many digits are repeating in
the number thus formed?
(a) two
(b) one
(c) none
(d) three
(e) four

Direction (37-38): Read the information given below and answer the questions.

All the given members belong to the same the family. J is the brother of L. J is the only son of R. W is
the father-in-law of L. D is the maternal grandfather of P, who is a male. Q is the only son of W. W is
the grandfather of N and C is the daughter of N.

Q37. How L is related to C?


(a) Mother
(b) Son
(c) Brother
(d) Father
(e) None of these

Q38. How is P related to N?


(a) Mother
(b) Son
(c) Brother
(d) Father
(e) None of these
Direction (39-41): Read the information carefully and answer the question:

Point U is 10m north of point Q. Point T is 10m east of point U.


Point S is 15m south of point T. Point P is 20m south of point Q.
Point R is 25m east of point P. Point L is 15m east of point S. Point
M is the midpoint of point U and P.

Q39. What is the distance between point L and R?


(a) 10m
(b) 15m
(c) 5m
(d) 20m
(e) 25m

Q40. In which direction is point T with respect to P?


(a) north-west
(b) south-west
(c) south-east
(d) north-east
(e) none of these

Q41. Which of the following points are inline?


(a) P, R, S
(b) Q, M, L
(c) U, S, T
(d) M, S, L
(e) Q, S, L

Directions (42-46): Read the following information carefully and answer the given questions.

Twelve persons sitting in two rows. D, E, F, K, L and M sitting in row-1 and facing north. S, T, U, X, Y
and Z sitting in row-2 and facing south direction. E sits third from one of the extreme ends. S sits
second to the left of the one who faces E. Only three persons sit between S and T. K sits somewhere
right of M. More than three persons sit between X and T. F faces one of the immediate neighbours of
T. Z sits second to the right of Y. The one who faces L sits third to the left of U. D faces S.

Q42. Who among the following faces K?


(a) T
(b) S
(c) X
(d) Y
(e) none of these
Q43. Who among the following faces the immediate neighbor of M?
(a) Z
(b) K
(c) D
(d) L
(e) None of these

Q44. Four of the following five from a group, which among the following does not belong to this
group?
(a) T, E
(b) U, D
(c) Y, L
(d) Z, E
(e) Z, K

Q45. Who among the following faces the one who sit to the immediate left of Y?
(a) U
(b) D
(c) X
(d) Z
(e) none of these

Q46. How many persons sit between M and D?


(a) one
(b) two
(c) three
(d) five
(e) four

Direction (47): Five people A, B, X, Y, and Z live on five different floors of a building (such as
ground floor numbered as 1 and top is numbered as 6). There are three floors between A and B. X
lives one of the floors above Y.

Q47. Who among the following lives on third floor?


(a) B
(b) A
(c) X
(d) Z
(e) Cannot be determined
Q48. Which of the following elements should come in a place ‘?’ ?
AB3 CE6 FI10 JN15 ?
(a) OT20
(b) TO21
(c) OT21
(d) TS21
(e) None of these

Directions (49-51): Each of the questions below consists of a question and two statements
numbered I and II given below it. You have to decide whether the data provided in the statement
are sufficient to answer the question. Read both the statements and
Given answer:
(a) If the data in statement I alone are sufficient to answer the question, while the data in statement II
alone are not sufficient to answer the question.
(b) If the data in statement II alone are sufficient to answer the question, while the data in statement I
alone are not sufficient to answer the question.
(c) If the data either in statement I alone or in statement II alone are sufficient to answer the question.
(d) If the data even in both statements I and II together are not sufficient to answer the question.
(e) If the data in both statement I and II together are necessary to answer the question.

Q49. Who sits immediate to the left of Ravi, who is sitting in row. All the persons who are sitting
in a row facing north direction?
I. There are only two persons sit between Sahil and Geeta. More than three persons sit to the left of
Geeta.
II. Not more than 8 persons can sit in a row. Ravi sits second to the left of Sahil. Diya sits 6 places
away from Geeta.

Q50. What is the code of ‘right’ in a certain code language?


I. The code of ‘every right to reject’ is ‘%47 *32 $53 *95’,
II. The code of ‘never reject right turn’ is ‘%62 %47 $51 *32’.

Q51. Find the number of boys and number of girls in the row?
I. R sits 18th from left end of the row and Y sits 11th from the right end of the row. R and Y
interchange their positions, after interchanging the position R’s position is 20 th from left end.
II. Total 43 students are in the row and all are facing is same direction.

Directions (52-56): Study the following arrangement carefully and answer the questions given
below:

B 5 R 1 @ E K 4 F 7 © D A M 2 P 3 % 9 H I W 8 * 6 U J $ V Q #
Q52. Which of the following is the fifth to the left of the seventeenth from the left end of the
above arrangement?
(a) D
(b) W
(c) *
(d) 4
(e) None of these

Q53. Which of the following is exactly in the middle between D and U in the above arrangement?
(a) %
(b) H
(c) 9
(d) 3
(e) None of these

Q54. Four of the following five are alike in a certain way based on their position in the above
arrangement and so form a group. Which is the one that does not belong to that group?
(a) R1E
(b) F7D
(c) M23
(d) 9HW
(e) UJ6

Q55. How many such symbols are there in the above arrangement each of which is immediately
preceded by a number but not immediately followed by a consonant?
(a) None
(b) One
(c) Two
(d) Three
(e) More than three

Q56. Which of the following is the tenth to the left end of the
thirteenth from the right end?
(a) F
(b) M
(c) @
(d) %
(e) 3
Direction (57-58): Read the information carefully and answer the questions:

Eight persons A, B, C, D, E, F, G, H are sitting around a circular table facing centre. H faces B. Two
persons sit between F and B. E sits 2nd right to D. F sits 2nd right to C, who is one of the immediate
neighbors of G. C is not an immediate neighbor of B.

Q57. Who among the following sits 3rd left to F?


(a) D
(b) C
(c) B
(d) A
(e) none of these

Q58. Who among the following faces A?


(a) D
(b) C
(c) B
(d) G
(e) none of these

Directions (59-60): Study the following information carefully and answer the given questions.

In a certain code language,


‘good key friends’ is coded as ‘xo pe cm’
‘key law found’ is coded as ‘xo og bt’
‘data key good’ is coded as ‘tu xo pe’

Q59. Which of the following is the code for ‘good’?


(a) xo
(b) pe
(c) tu
(d) cm
(e) None of these

Q60. Which of the following word is coded as ‘og’?


(a) law
(b) good
(c) found
(d) Either (a) or (c)
(e) key
Directions (61-65): The following questions are based on the six three digits numbers given below:
563 218 732 491 929

Q61. If 2 is subtracted from the second digit of all odd numbers and 2 is added in the first digit of
all even numbers, then which number is lowest number after the arrangement?
(a) 218
(b) 732
(c) 491
(d) 929
(e) None of these

Q62. If third digit of highest number is divided by the first digit of lowest number, then what will
be the resultant?
(a) 4
(b) 6
(c) 4.5
(d) 5
(e) None of these

Q63. If all the digits in each number are arranged in increasing order, then which number will be
the highest number after the rearrangement?
(a) 218
(b) 732
(c) 491
(d) 563
(e) None of these

Q64. How many numbers will be there in the given series in which addition of first and third digit
is greater than second digit?
(a) One
(b) Two
(c) Three
(d) Four
(e) None of these

Q65. How many numbers will be there in the given series in


which difference of first and third digit is greater than second
digit?
(a) One
(b) Two
(c) Three
(d) Four
(e) None of these
Directions (66-70): What should come in place of question mark (?) in the following questions?

Q66. 18, 8, 6, 9, 32, ?


(a) 248
(b) 254
(c) 251
(d) 257
(e) 260

Q67. 36, 18, 6, 3, 1, ?


(a) 0.5
(b) 0.25
(c) 0.75
(d) 0.3
(e) 1

Q68. 18, 29, 42, 53, ?, 77


(a) 63
(b) 64
(c) 65
(d) 66
(e) 67

Q69. 1, 244, 163, 190, 181, ?


(a) 178
(b) 184
(c) 187
(d) 190
(e) 193

Q70. 250, ?, 190, 167, 148, 131


(a) 215
(b) 217
(c) 223
(d) 221
(e) 219
Direction (71-75): Pie chart given below shows total number of students who opted different
subjects for exam in July 2017. Study the data carefully and answer the following questions

Total students = 300

Hindi,
Geography,
13%
18%

English,
17% Maths,
12%

Physics,
Sanskrit, 16%
24%

Q71. Total number of students who opted for Geography and Maths together is how much less
than total number of students who opted for Sanskrit and Physics together?
(a) 24
(b) 27
(c) 30
(d) 33
(e) 36

Q72. Find the total number of students who gave exams in August 2017 if total number of students
is increased by 20% in August 2017 as compared to July 2017.
(a) 450
(b) 420
(c) 390
(d) 330
(e) 360

Q73. Find the central angle of total number of students who opted for Physics?
(a) 57.6°
(b) 54°
(c) 50.4°
(d) 43.2°
(e) 64.8°
Q74. Find the average number of students who opted for Hindi, Geography and Sanskrit together?
(a) 54
(b) 55
(c) 56
(d) 57
(e) 58

Q75. Find the ratio between total number of students who opted for English, Physics and
Geography together to total number of students who opted for Hindi, English and Sanskrit
together?
(a) 8 : 9
(b) 17 : 19
(c) 15 : 16
(d) 17 : 18
(e) 5 : 6

Directions (76-79): What should come in place of question mark (?) in the following questions?

Q76. ? = √𝟏𝟔 × 𝟏𝟓 + 𝟐𝟒 × 𝟏𝟐 + 𝟗𝟕
(a) 25
(b) 24
(c) 28
(d) 27
(e) 35

Q77. 28% of 420 + 36% of 540 = ?


(a) 312
(b) 288
(c) 296
(d) 318
(e) 324

Q78. 75% of 450 + 25% of 850 = ?


(a) 540
(b) 580
(c) 550
(d) 560
(e) 555

Q79. √𝟕𝟑𝟗𝟔 + √? = 𝟏𝟎𝟒


(a) 256
(b) 400
(c) 361
(d) 289
(e) 324
Q80. Present average age of A, B and C is 22 years. Three years ago, Average age of B and C is 18
years, then find A’s age 9 years hence?
(a) 24 years
(b) 27 years
(c) 30 years
(d) 33 years
(e) 36 years

Q81. Ratio between speed of boat in still water to speed of stream is 8 : 1. If 67.5km is travelled
downstream in 2.5 hours then find the difference between speed of boat in still water to speed of
stream(in kmph)?
(a) 15
(b) 3
(c) 24
(d) 21
(e) 17.5

Q82. The perimeter of a rectangle whose length is 6 m more than its breadth is 84 m. What will be
the area of the rectangle? (in m²)
(a) 446
(b) 340
(c) 432
(d) 468
(e) 348

Q83. Interest earned on an amount after 2years at 20%p.a compounded yearly is Rs.1716. Find the
interest earned on same amount after 3 years at 15%p.a at Simple interest.
(a) Rs. 1620
(b) Rs.1755
(c) Rs.1665
(d) Rs.1710
(e) Rs.1750

Q84. In place of 18% profit an article is sold at 42% profit and seller gets Rs. 110.40 more. Find the
selling price of article if it were sold at 25% profit?
(a) Rs. 440
(b) Rs.460
(c) Rs.575
(d) Rs.550
(e) Rs.525
Q85. A and B working alone can do a work in 20 days and 15 days respectively. They started the
work together but B left after sometime and A finished remaining work in 6 days. Find after how
many days from start B left the work ?
(a) 5 days
(b) 4 days
(c) 6 days
(d) 3 days
(e) 7 days

Directions (86-90): In each of these questions, two equations are given. You have to solve both the
equations and give answer
(a) if x>y
(b) if x≥y
(c) if x<y
(d) if x ≤y
(e) if x = y or no relation can be established between x and y.

Q86. (i) 𝑥 2 = 196


(ii) y² + 2y − 48 = 0

Q87. (i) x² – 11x + 24 =0


(ii) y² – 14y + 45 = 0

Q88. (i) 2x² – 4x + 2 =0


(ii) 2y² – y − 1 = 0

Q89. (i) x² – 15x + 56 = 0


(ii) 𝑦 = √64

Q90. (i) x² – x – 6 = 0
(ii) y² – 6y + 8 = 0

Directions (91-94): What should come in place of question mark


(?) in the following questions?

Q91. √𝟒𝟒𝟏 − √𝟏𝟒𝟒 = √?


(a) 81
(b) 9
(c) 100
(d) 10
(e) 121
𝟐 𝟏 𝟏
Q92. 𝟏𝟖 𝟑 − 𝟕 𝟒 =? +𝟏 𝟐
(a) 9
1
(b) 10 12
11
(c) 9 12
5
(d) 9 6
(e) 10

Q93. √𝟒𝟖𝟒 × √𝟏𝟔𝟗 = ? +𝟓𝟎% 𝒐𝒇 𝟑𝟏𝟐


(a) 160
(b) 150
(c) 140
(d) 130
(e) 120

𝟑
Q94. 𝟏𝟓𝟐 + 𝟑𝟔𝟐 = ? × √𝟐𝟏𝟗𝟕
(a) 127
(b) 117
(c) 137
(d) 147
(e) 153

Q95. The profit earned on selling two articles is Rs. 80 less than profit earned on selling three
articles. If 20% profit is earned on selling one article, then find the cost price of the article?
(a) Rs.200
(b) Rs.600
(c) Rs.1200
(d) Rs.800
(e) Rs.400

Q96. Quantity I: ‘x’: Train ‘A’ running at a speed of 25m/sec crosses Train ‘B’ coming from opposite
direction running at a speed of 15m/sec in 12 seconds. Length of train ‘A’ is twice of train ‘B’. Length
of train ‘A’ is ‘x’
Quantity II: 160 meters.
(a) Quantity I > Quantity II
(b) Quantity I < Quantity II
(c) Quantity I ≥ Quantity II
(d) Quantity I ≤ Quantity II
(e) Quantity I = Quantity II or No relation
Q97. Average of three numbers b, c and d is 1 more than average of a, b and c. Average of a and d
is 19.5
Quantity I: Number ‘a’
Quantity II: 21
(a) Quantity I > Quantity II
(b) Quantity I < Quantity II
(c) Quantity I ≥ Quantity II
(d) Quantity I ≤ Quantity II
(e) Quantity I = Quantity II or No relation

Q98. Quantity I: ‘x’: A pipe alone can fill a cistern in 60 minutes. But due to leakage pipe filled only
80% of the cistern in 1 hour. ‘x’ is the capacity of cistern in liters if due to leakage 60liter can be leaked
out in 1 hour.
Quantity II: 250 liters
(a) Quantity I > Quantity II
(b) Quantity I < Quantity II
(c) Quantity I ≥ Quantity II
(d) Quantity I ≤ Quantity II
(e) Quantity I = Quantity II or No relation

Q99. Quantity I: ‘x’: Ratio between speed of boat in still water to speed of stream is 2 : 1. Total time
taken by a man to cover 72km in upstream and come back is 32 hours. ‘x’ is the downstream speed in
kmph
Quantity II: 9 kmph
(a) Quantity I > Quantity II
(b) Quantity I < Quantity II
(c) Quantity I ≥ Quantity II
(d) Quantity I ≤ Quantity II
(e) Quantity I = Quantity II or No relation

Q100. Quantity I: ‘x’: Area of a square is 324𝑐𝑚2 whose perimeter


is equal to perimeter of a rectangle. Length of rectangle is 4cm
more than breadth of rectangle. ‘x’ is the area of rectangle
Quantity II: 320 𝑐𝑚2 .
(a) Quantity I > Quantity II
(b) Quantity I < Quantity II
(c) Quantity I ≥ Quantity II
(d) Quantity I ≤ Quantity II
(e) Quantity I = Quantity II or No relation
Solutions

S1. Ans (c)


Sol. Refer to the first paragraph of the passage where it is clearly mentioned, “Yet despite this
overwhelming evidence, access to flexible work and careers is not widespread. Flexible work is still
regarded as an add-on, something we do for mothers for a few months when they are back from
parental leave.” Hence, option (c) will be the correct answer.

S2. Ans (e)


Sol. Refer to the second paragraph of the passage where it is mentioned, “In today’s workforce, fewer
people identify with the stereotype of the ideal worker – a full-time, fully committed employee without
personal or family commitments that impact on availability”. Hence, option (e) will be the correct
answer.

S3. Ans (b)


Sol. Refer to the third paragraph of the passage where it is mentioned, “Similarly, technology is driving
– and enabling – greater flexibility. It is dramatically reshaping our workplaces, blurring the
boundaries between work and home and diversifying where, when and how employees work”. Hence,
option (b) will be the correct answer.

S4. Ans (d)


Sol. Refer to the last paragraph of the passage where it is mentioned, “For example, a UK bakery sat
down with their bakers and came up with a flexible system of two to three baking shifts a day to
maintain a steady supply of fresh bread. The team agreed to rotate their hours each week so no team
member permanently worked a shift that did not suit. After the change was made, bakery sales
increased by more than 65% in the first year and employee satisfaction in the bakery has risen 10%
since the change to 93%.” Option (III) will not be the answer because it was the result of the steps taken
by the bakery, not the step itself. Hence, option (d) will be the correct answer.

S5. Ans (c)


Sol. Option (c), i.e. “The brightest workforce requires more flexible work” is the most appropriate title
of the passage.

S6. Ans (b)


Sol. Predict means say or estimate that (a specified thing) will
happen in the future or will be a consequence of something.
Forecast has the same meaning as predict. Hence, option (b) will
be the correct answer.
Recount means tell someone about something; give an account of
an event or experience.
Narrate means give a spoken or written account of.
S7. Ans (e)
Sol. Temporary means lasting for only a limited period of time; not permanent. Permanent has the
most opposite meaning to temporary. Hence, option (e) will be the correct answer.
Expedient means (of an action) convenient and practical although possibly improper or immoral
Provisional means arranged or existing for the present, possibly to be changed later.

S8. Ans (b)


Sol. In the second part of the sentence, the word “were” will be replaced by the word “was” because
when two subjects are combined with ‘together with’, the main subject is the one which is written
before together with, and the verb follows the main subject. All other parts of the sentence are correct.
Hence, option (b) will be the correct answer.

S9. Ans (b)


Sol. In the second part of the sentence, the word “lived” will be replaced by “has lived” or “has been
living” because “is watching” used in first part of the sentence and “has never created” used in the
third part of the sentence signify that the sentence is in present tense. Hence, option (b) will be the
correct answer.

S10. Ans (d)


Sol. In the fourth part of the sentence, “are” should be replaced by “is” because the subject of the
statement starts with “Each of” and such sentences always follow singular Verb. Hence, option (d) will
be the correct answer.

S11. Ans (c)


Sol. In the third part of the sentence, “when” will be replaced by “than” because ‘No sooner’ is always
followed by ‘than’ and ‘Hardly/Scarcely’ is followed by ‘when/before’. Hence, option (c) will be the
correct answer.

S12. Ans (d)


Sol. In the fourth part of the sentence, “cares” should be replaced by “care” because if two subjects are
combined using ‘Either…or’ or ‘Neither…nor’ then the verb always follows the subject near to it, which
in this case is ‘the city corporations’. So the verb will be plural. Hence, option (d) will be the correct
answer.

S13. Ans (e)


Sol. All the parts of the given sentence are grammatically correct. Hence, option (e) will be the correct
answer.

S14. Ans (d)


Sol. In the fourth part of the sentence, “greatest” should be replaced by “greater” because here we are
comparing between two types of people, ‘Prime Minister’ and ‘Ministers’, and when we compare two
types of people or things in Comparative Degree then the adjective used should also be of Comparative
Degree. Hence, option (d) will be the correct answer.
S15. Ans (a)
Sol. The error lies in the first part of the sentence where the usage
of the words ‘blunder’ and ‘mistake’ together is superfluous as
‘blunder’ itself means ‘a stupid or careless mistake’. Hence, option
(a) will be the correct answer.

S16. Ans. (c)


Ans. The correct sequence of the other parts to form a
grammatically correct and contextually meaningful sentence is
BACD. Hence, option (c) is the most suitable answer choice.

S17. Ans. (a)


Sol. The correct sequence of the other parts to form a grammatically correct and contextually
meaningful sentence is BADC. Hence, option (a) is the most suitable answer choice.

S18. Ans. (b)


Sol. The correct sequence of the other parts to form a grammatically correct and contextually
meaningful sentence is CABD. Hence, option (b) is the most suitable answer choice.

S19. Ans. (d)


Sol. The correct sequence of the other parts to form a grammatically correct and contextually
meaningful sentence is ADCB. Hence, option (d) is the most suitable answer choice.

S20. Ans. (e)


Sol. The given parts of the sentence are in their precise position forming a coherent sentence. Since,
they do not require any rearrangement option (e) becomes the most suitable answer choice.

S21. Ans. (a)


Sol. The most appropriate word that would fill the blank is ‘mistakes’ which means an act or judgement
that is misguided or wrong. All the other words do not fill the blank appropriately, hence option (a) is
the most suitable answer choice.
Accuracies means the quality or state of being correct or precise.
Obstacles means a thing that blocks one's way or prevents or hinders progress.
Disputes means a disagreement or argument.

S22. Ans. (c)


Sol. The most appropriate word that would fill the blank is ‘travel’ which means journeys, especially
abroad. All the other words do not fill the blank appropriately, hence option (c) is the most suitable
answer choice.
Deportation means the action of deporting a foreigner from a country.
Transfer means move from one place to another.
S23. Ans. (d)
Sol. The most appropriate word that would fill the blank is ‘discretion’ which means the freedom to
decide what should be done in a particular situation. All the other words do not fill the blank
appropriately, hence option (d) is the most suitable answer choice.
Omission means someone or something that has been left out or excluded.
Affairs means an event or sequence of events of a specified kind or that has previously been referred
to.

S24. Ans. (b)


Sol. The most appropriate word that would fill the blank is ‘opportunity’ which means a time or set of
circumstances that makes it possible to do something. All the other words do not fill the blank
appropriately, hence option (b) is the most suitable answer choice.
Negligence means failure to take proper care over something.
Surveillance means close observation, especially of a suspected spy or criminal.

S25. Ans. (e)


Sol. The most appropriate word that would fill the blank is ‘estimation’ which means a judgement of
the worth or character of someone or something. All the other words do not fill the blank appropriately,
hence option (e) is the most suitable answer choice.
Consideration means careful thought, typically over a period of time.
Suggestion means an idea or plan put forward for consideration.
Guidance means advice or information aimed at resolving a problem or difficulty, especially as given
by someone in authority.

S26. Ans. (d)


Sol. The most appropriate phrase to replace the phrase given in bold is “despite being the most
efficient”. It is to be noted that “despite” does not take any preposition with it. It is always followed
with a noun, pronoun or a gerund. Moreover, when we use a superlative adjective (most) before the
noun, we generally use it with 'the'. This is because there's only one (or one group) of the things we are
talking about. Since option (d) is in the precise grammatical syntax, it becomes the most suitable answer
choice.

S27. Ans. (a)


Sol. The most appropriate phrase to replace the incorrect bold phrase in the sentence is “satisfactory
number of contestants”. It is to be noted that ‘satisfactory’ is an adjective which means ‘fulfilling
expectations or needs; acceptable, though not outstanding or perfect’ while ‘satisfactorily’ is an adverb.
Moreover, ‘number of’ reflects the plural nature of the noun, hence “contestant” should be replaced by
“contestants”. Therefore, option (a) becomes the most suitable answer choice.
S28. Ans. (b)
Sol. The most suitable phrase that should replace the phrase given
in bold is “depending upon the power of”. Except for option (b)
none other options are in absolute grammatical syntax. Hence, it
becomes the most viable answer choice.

S29. Ans. (c)


Sol. The most appropriate phrase to replace the given phrase in
bold to make the sentence grammatically correct is “had been
teaching”. It is to be noted past perfect continuous tense is used for
ongoing continuous events over a period of time before another
action or event in the past. These must be continuous verbs, and cannot be actions that are still
happening in the present. Hence, considering the given rationale option (c) becomes the most suitable
answer choice.

S30. Ans. (e)


Sol. The given phrase in bold is accurate and does not require any corrections or replacement. Hence,
option (e) becomes the most suitable answer choice.

Solutions (31-33);
Years Books
1947 56
1956 59
1987 66
1998 61
2002 63

S31. Ans.(a)
S32. Ans.(d)
S33. Ans.(a)

S34. Ans. (a)


Sol. The 1st, 6th , 8th and 9th letter of a word ‘EMANICIPATE’ are E, C, P, A
So, the words formed are PACE and CAPE

S35. Ans.(a).
Sol. The given word is- FIGURES
After the rearrangement- EIUFGRS
So, the letters between U and R will be two.
S36. Ans. (b)
Sol. The given number is- 39682147
After applied operation- 48573256
So, only digit 5 is repeated in the number thus formed.

Solutions (37-38)

S37. Ans. (e)


S38. Ans. (c)

Solutions (39-41):

S39. Ans. (b)


S40. Ans. (d)
S41. Ans. (d)

Solution (42-46):

S42. Ans.(d)
S43. Ans.(a)
S44. Ans.(d)
S45. Ans.(b)
S46. Ans.(c)
S47. Ans.(e)
Floors Persons
5 A/B
4 X/Z
3 X/Z/Y
2 Z/Y
1 B/A

S48. Ans. (c)


Sol. OT21

S49. Ans.(e)
Sol. From both the statements I and II we can find that Diya sits immediate to the left of Ravi.

S50. Ans.(d)
Sol. right can be coded either %47 or *32.

S51. Ans.(d)

S52. Ans.(a)
Sol. 5th to left of 17th from left= 17-5= 12th from left = D

S53. Ans.(c)
Sol. 9

S54. Ans.(e)
Sol. UJ6

S55. Ans.(d)
Sol. 1@, 3%, 8*

S56. Ans.(a)

Solutions (57-58):

S57. Ans(e)
S58. Ans(d)
Solutions (59-60):
Word Code
friends cm
key xo
good pe
law/found og/bt
data tu

S59. Ans.(b)
S60. Ans.(d)

S61. Ans.(a)
Sol. 543 418 932 471 909

S62. Ans.(c)
Sol. 9/2= 4.5

S63. Ans.(d)
Sol. 356 128 237 149 299

S64. Ans.(d)
Sol. 563 218 732 929

S65. Ans.(b)
Sol. 218 732

S66. Ans.(c)
Sol.

S67. Ans.(a)
Sol.
S68. Ans.(d)
Sol.

S69. Ans.(b)
Sol.

S70. Ans.(e)
Sol.

S71. Ans.(c)
Sol. Required difference
(24 + 16) − (18 + 12)
= × 300
100
= (40 − 30) × 3 = 30
= 30

S72. Ans.(e)
Sol. Total number of students who gave exam in August 2017
120
= 300 × 100 = 360

S73. Ans.(a)
Sol. Required central angle = 16 × 3.6 = 57.6°

S74. Ans.(b)
Sol. Required average
1 13+18+24
= 3( 100
) × 300 = 55

S75. Ans.(d)
Sol. Required Ratio
17 + 16 + 18 51 17
= = =
13 + 17 + 24 54 18
S76. Ans.(a)
Sol.
? = √16 × 15 + 24 × 12 + 97
? = √240 + 288 + 97
? = √625
? = 25

S77. Ans.(a)
Sol.
28 36
?= × 420 + × 540
100 100
? = 117.6 +194.4
? = 312

S78. Ans.(c)
Sol. 75% × 450 + 25% × 850 = ?
25 1
?= [3 × 450 + 850] = [2200] = 550
100 4

S79. Ans.(e)
Sol. √? = 104 − √7396
√? = 104 − 86
? = (18)2 = 324

S80. Ans.(d)
Sol. Sum of present ages of A, B and C = 66 years
Sum of present age of B and C = 18 × 2 + 6 = 42
Present age of A = 66 − 42 = 24
A’s age nine years hence = 24 + 9 = 33 years

S81. Ans.(d)
Sol. Let speed of boat in still water and speed of stream be 8x and x respectively.
ATQ,
67.5
= 8x + x
2.5
27
x=
9
x=3
Required difference = 8x − x = 7x = 7 × 3 = 21
S82. Ans.(c)
Sol. Breadth of rectangle = x metre
Length = (x+ 6) metre
∴ 2(x+ 6 +x) = 84
⇒ 2x = 42 – 6 = 36
⇒ x= 18
∴ Length = 18 + 6 = 24 metre
∴ Area of rectangle
= Length × Breadth
= 18 × 24
= 432 sq. metre

S83. Ans.(b)
20×20
Sol. Overall rate for 2 years at 20% p.a compounded yearly is equivalent to 20 + 20 + = 44%
100
ATQ,
44% of sum = 1716
100% of sum = 3900
3900×15×3
Simple interest earned = = Rs. 1755
100

S84. Ans.(c)
Sol. Let cost price of article = 100x
ATQ,
42x − 18x = 110.4
24x = 110.4
x = 4.6
Cost price of article = 4.6 × 100 = 460
125
Selling price to earn 25% profit = 460 × 100 = Rs 575
S85. Ans.(c)
Sol.

Work done by A in last 6 days = 6 × 3 = 18 work.


Remaining work done by A + B = 60 – 18 = 42 work
42
B left the work after = = 6 days.
7

S86. Ans.(e)
Sol.
(i) x 2 = 196
x = ±14
(ii) y² + 2y − 48 = 0
y² + 8y – 6y − 48 = 0
y (y + 8) –6 (y + 8) = 0
(y – 6) (y + 8) = 0
y = 6 , −8
No relation can be established between x and y

S87. Ans.(e)
Sol.
(i) x² – 11x + 24 =0
x² – 8x – 3x + 24 =0
x (x – 8) – 3 (x – 8) =0
(x – 3) (x –8) = 0
x = 8, 3
(ii) y² – 14y + 45 = 0
y² – 9y – 5y + 45 = 0
y(y –9) – 5 (y – 9)=0
(y – 5) (y – 9) = 0
y = 5, 9
No relation can be established between x and y

S88. Ans.(b)
Sol.
(i) 2x² – 4x + 2 =0
2x² – 2x – 2x + 2 =0
2x (x – 1) – 2 (x – 1) =0
(2x –2) (x –1) = 0
x = 1, 1
(ii) 2y² – y − 1 = 0 S92. Ans.(c)
2y² – 2y + y − 1 = 0 Sol.
2 1 1
2y (y –1) +1(y – 1)=0 18 3 − 7 4 =? +1 2
2 1 1
(2y + 1) (y –1) = 0 18 − 7 + 3 − 4 =? +1 + 2
1 2 1 1
y=− ,1 10 + − − =?
2 3 4 2
8−3−6
x≥y 10 + =?
12
1
10 − 12 =?
S89. Ans.(d) 11
9 12 =?
Sol.
(i) x² – 15x +56 = 0 S93. Ans.(d)
x² – 7x − 8x +56 = 0 Sol.
x(x – 7) − 8 (x – 7) =0 √484 × √169 = ? +50% of 312
50
(x −8) (x – 7) = 0 22 × 13 = ? + 100 × 312
x = 8, 7 286 = ? +156
? = 130
(i) y = √64
y=8 S94. Ans.(b)
y≥x Sol.
3
152 + 362 = ? × √2197
S90. Ans.(e) 225 + 1296 = ? × 13
1521
Sol. =?
13
(i) x² – x – 6 = 0 117 = ?
x² – 3x + 2x – 6 = 0
S95. Ans.(e)
x (x – 3) +2 (x – 3) = 0
Sol. Let cost price of article = 100x
(x – 3) (x + 2) = 0 Selling price of one article = 120x
x = 3, –2 ATQ,
(ii) y² – 6y + 8 = 0 3 × 20x − 2 × 20x = 80
y² – 2y – 4y + 8 = 0 20x = 80
x=4
y (y – 2) – 4 (y – 2) = 0
Cost price of article = Rs 400
(y – 2) (y – 4) = 0
y = 2, 4 S96. Ans.(a)
No relation can be established between x and y Sol.
Quantity I:
Length of train ‘A’ = x
S91. Ans.(a)
Length of train ‘B’ = 0.5x
Sol. ATQ,
√441 − √144 = √? x + 0.5x = 12 × (25 + 15)
21 − 12 = √? 1.5x = 480
x = 320 meters
9 = √?
Quantity II: 160 meters
? = 81 𝐐𝐮𝐚𝐧𝐭𝐢𝐭𝐲 𝐈 > 𝐐𝐮𝐚𝐧𝐭𝐢𝐭𝐲 𝐈𝐈
S97. Ans.(b)
Sol. Let average of a, b and c be x
a + b + c = 3x
And, b + c + d = 3x + 3
⇒d−a= 3
And, d + a = 39
d = 21 and a= 18
Quantity I:
a = 18
Quantity II: 21
𝐐𝐮𝐚𝐧𝐭𝐢𝐭𝐲 𝐈𝐈 > 𝐐𝐮𝐚𝐧𝐭𝐢𝐭𝐲 𝐈

S98. Ans.(a)
Sol. Quantity I: Due to leakage only 80% of the cistern is filled this means 20% of tank is leaked out
by leakage which is equal to 60 liters
20% = 60
100% = 300liters
Capacity of tank = 300 liters
Quantity II: 250 liters
𝐐𝐮𝐚𝐧𝐭𝐢𝐭𝐲 𝐈 > 𝐐𝐮𝐚𝐧𝐭𝐢𝐭𝐲 𝐈𝐈

S99. Ans.(e)
Sol.
Quantity I:
Let speed of boat in still water and speed of stream be 2x and x respectively
ATQ,
72 72
⇒ 32 = +
3x x
96
⇒x= =3
32
Downstream speed = 2x + x = 3x = 9kmph
Quantity II: 9kmph
𝐐𝐮𝐚𝐧𝐭𝐢𝐭𝐲 𝐈 = 𝐐𝐮𝐚𝐧𝐭𝐢𝐭𝐲 𝐈𝐈

S100. Ans.(e)
Sol.
Quantity I:
Side of square = √324 = 18cm
Let length of rectangle be x and breadth of rectangle be (x−4) cm
ATQ,
4 × 18
x+x−4 = = 36
2
x = 20
Area of rectangle = 20 × 16 = 320cm2
Quantity II: 320cm2
Quantity I = Quantity II
Part - 6

REASONING ABILITY

Directions (Q. 1 – 5): In these questions, a (c) 0


relationship between different elements is shown in (d) 1
the statements. The statements are followed by two (e) 4
conclusions. Given answer 9. If 1 is added to the first digit and 2 is added to the last
(a) if only conclusion I is true. digit of each of the numbers then which of the
(b) if only conclusion II is true. following numbers will be the second highest
(c) if either conclusion I or II is true. number?
(d) if neither conclusion I nor II is true. (a) 385
(e) if both conclusions I and II are true. (b) 684
1. Statement: C  D > E = M < J = L (c) 437
Conclusion : I. L > E (d) 296
II. C  J (e) 512
2. Statement: P = N  Q > R > T = S 10. If in each number the first and the second digits are
Conclusion : I. N  S interchanged then which will be the highest number?
II. P  Q (a) 296
3. Statement: J  P = I  M < T  V > H (b) 512
Conclusion : I. M  J (c) 437
II. H  M (d) 684
4. Statement: Q  X  E > F = D < O < K = G (e) 385
Conclusion : I. D > Q Directions (Q. 11-15): Study the flowing information
carefully to answer arrangement of symbols, digits and
II. K  E
letters.
5. Conclusion : I. Q  E
W%93 G6H#7K$L2BMJ©45E8@Z
II. G > F
11. If all the numbers are deleted from the above
Directions (Q. 6 – 10): The following questions are
arrangement then which of the following will be
based on the five three – digit numbers given below:
seventh to the left of sixth from the right?
684 512 437 385 296
(a) H (b) J (c) M
6. If 2 is added to the first digit of each of the numbers
(d) $ (e) None of these
how many numbers thus formed will be divisible by
12. How many such numbers are there in the above
three?
arrangement each of which is immediately preceded
(a) None
by a symbol?
(b) One
(a) One (b) Two (c) Three
(c) Two
(d) Four (e) None of these
(d) Three
13. ‘9W’ is to ‘GH#’ and ‘$7’ is to ‘2BM’ in the same way
(e) None of these
as ‘4J’ is to ______ in the arrangement.
7. If all the digits in each of the numbers are arranged in
(a) E@8 (b) 58® (c) B2L
descending order within the number, which of the
(d) 58Z (e) None of these
following will be the highest number in the new
14. How many such symbols are there in the above
arrangement of numbers ?
arrangement each of which is immediately followed
(a) 684
by a letter?
(b) 385
(a) None (b) One (c) Two (d)
(c) 296
Three (e) None of these
(d) 437
15. If all the symbols are deleted from the above
(e) None of these
arrangement then which of the following will be the
8. What will be the resultant number if the second digit
fourth to the left of twelfth from the right?
of the second lowest number is divided by the third
(a) 9 (b) 3 (c) W
digit of the highest number ?
(d) M (e) None of these
(a) 2
(b) 3

www.bankersadda.com | www.sscadda.com | www.careerpower.in | www.careeradda.co.in


Directions (Q. 16-20): Study the following information Directions (Q. 26- 27): Study the following
carefully to answer these questions. information carefully to answer these questions.
Eight friends A, B, C, D E, F G and H are sitting around a A vehicle starts from point P and run 10 km towards
circle facing the center. A sits third to the left of B, while North. It takes a right turn and runs 15 km. Now It
second to the right of F. D does not sit next to A or B. C runs 6 km after taking a left turn. Finally, It takes a
and G always sit next to each other. H never sits next to D left turn, runs 15 km and stops at point Q.
and C does not sit next to B. 26. How far is point Q with respect to point P?
16. Which of the following pairs sits between H and E? (a) 16 km (b) 25 km
(a) F, D (b) H, B (c) C, G (c) 4 km (d) 0 km
(d) E, G (e) None of these (e) None of these
17. Starting from A’s position, if all the eight were 27. Towards which direction was the vehicle moving
arranged in alphabetical order in clockwise direction before it stopped at point Q?
the seating position of how many members (a) North (b) East
(excluding A) not change? (c) South (d) West
(a) None (b) one (c) Two (d) (e) North - West
Three (e) None of these 28. The position of how many digits in the number
18. Which of the following pairs has only one person 54327618 will remain unchanged if the digits within
sitting between them, if the counting is done in the number are written in ascending order from left
clockwise direction? to right?
(a) A, B (b) C,D (c) F, E (a) None
(d) G,H (e) None of these (b) One
19. Who sits to the immediate right of E? (c) Two
(a) A (b) D (c) F (d) Three
(d) H (e) None of these (e) None of these
20. What is the position of B with respect to C? 29. In a row of 34 students, W is fifth after X from the
(a) Second to the left (b) Third to the front and X is 20th from the back. What is the position
right of W from the front?
(c) Third to the left (d) Can’t be (a) 20
determined (b) 25
(e) None of these (c) 30
21. In a certain code language SERIES is written as (d) 22
QCGTGU. How is EXPERT written in that code (e) None of these
language? 30. What will come in place of question mark (?) in the
(a) VTGRZG (b) RPCRZG (c) following series?
GZRCPR TG HU VI JW ?
(d) RPCGZR (e) None of these (a) KY
22. How many such pairs of letters are there in the word (b) KX
COMPOSE each of which has many letters between (c) YK
them in the word as they have between them in the (d) XK
English alphabetical series? (e) None of these
(a) None (b) One (c) Two Directions (Q. 31-35) Study the following information
(d) Three (e) None of these carefully and answer the question given below.
Directions (Q. 23-25): Study the following information Six friends A ,B, C, D, E and F are going for entrance exam
carefully and answer the questions given below. during a period of starting from Monday to Saturday.
In a certain code ‘all Aspirants must qualify’ is written as Only one exam will be held on each day.
‘na li ja pa’, ‘qualify in all subjects’ in written as ‘ta ja li ra’  C gives exam atleast before three exams.
and ‘Aspirants read all subjects’ is written as ‘sa li na ra’.  F gives exam on Tuesday.
23. What does ‘li’ stand for?  Both B and E does not give exam on even day.
(a) subject (b) qualify (c) must  D is going for exam immediately after the exam on
(d) All (e) None of these which C given.
24. What will be the code for ‘ja na’?  Atleast four persons are going for exam after E.
(a) Must qualify (b) Aspirants subjects
(c) qualify subjects 31. On which day E gives entrance exam?
(d) Can’t be determined (e) None of these (a) Wednesday
25. What will be the code for ‘qualify’? (b) Monday
(a) li (b) sa (c) ja (c) Friday
(d) pa (e) Can’t be determined. (d) Tuesday
www.bankersadda.com | www.sscadda.com | www.careerpower.in | www.careeradda.co.in
(e) None of these.
32. Who gives exam immediately after the exam of ‘B’?
(a) E
(b) D
(c) A
(d) B
(e) None of these.
33. How many persons give exam before on which C
gives?
(a) One
(b) Two
(c) Three
(d) Four
(e) None.
34. ‘A’ going for the exam on which day?
(a) Monday
(b) Wednesday
(c) Saturday
(d) Tuesday
(e) None of these.
35. How many persons give exam between B and the
person who’s exam is on Monday?
(a) One
(b) Two
(c) Three
(d) Four
(e) None.

www.bankersadda.com | www.sscadda.com | www.careerpower.in | www.careeradda.co.in


MATHS

Directions (36-40): What should come in the place of (a) 105 (b) 98 (c) 145
question mark (?) in the following questions? (d) 139 (e) 104
36. Directions (46-50): What should come in the place of
question mark (?) in the following questions?
(a) 133.4 (b) 137.2 (c) 127.8
46. 4376 + 3209 – 1784 + 97 = 3125 + ?
(d) 131.6 (e) None of these
(a) 2713 (b) 2743 (c) 2773
37.
(d) 2793 (e) 2737
(a) 166.5 (b) 167.5 (c) 168.5
(d) 169.5 (e) None of these 47. √ + 14 = √
38. (a) 1521 (b) 1369 (c) 1225
(a) 61481 (b) 62921 (c) 63861 (d) 961 (e) 1296
(d) 64241 (e) None of these 48. 85% of 420 + ?% of 1080 = 735
39. (a) 25 (b) 30 (c) 35
(a) 2.8 (b) 3 (c) 3.2 (d) 40 (e) 45
(d) 4 (e) 6 49. of of of 3024 = ?
40. (a) 920 (b) 940 (c) 960
(a) 404 (b) 408 (c) 410 (d) 980 (e) 840
(d) 414 (e) 416 50. 30% of 1225 – 64% of 555 = ?
Directions (41-45): Study the following table carefully and (a) 10.7 (b) 12.3 (c) 13.4
answer the given questions: (d) 17.5 (e) None of these
The number of various crimes, as supplied by national 51. How many litres of water should be added to a 30 litre
crime record, reported in different states in the year mixture of milk and water containing milk and water in
2012-13. the ratio of 7 : 3 such that the resultant mixture has 40%
Dist. Stalking Assault Theft Murder Criminal water in it?
Trespass (a)5 (b)2 (c)3
Bihar 352 496 265 132 332 (d)8 (e) 7
MP 376 225 216 125 115 52. The S.I on certain sum of money for 15 months at rate of
UP 85 125 53 56 57 7.5% per annum exceed the S.I on same sum at 12.5%
HP 10545 3652 12224 354 10128 per annum for 8 months by Rs 3250 find sum?
AP 445 225 252 173 154 (a) 160000 (b)20000 (c) 170000
Delhi 473 576 675 764 852 (d)18000 (e)312000
Haryana 245 256 257 261 263 53. 4 men and 3 women finish a job in 6 days, and 5 men and
Rajasthan 273 276 278 252 353 7 women can do the same job in 4 days. How long will 1
41. The total number of various crimes in HP is man and 1 woman take to do the work?
(a) 37803 (b) 38903 (c) 37903
(a) 22( ) days (b) 25( ) days (c) 5( ) days
(d) 36903 (e) 37003
42. Find the ratio of Stalking and Assault in UP to Theft and (d) 12 days (e) None of these
Criminal Trespass in Haryana. 54. A and B started a business with initial investments in the
(a) 28 : 51 (b) 21 : 52 (c) 52 : 21 ratio 5 : 7. If after one year their profits were in the ratio
(d) 14 : 55 (e) 55 : 14 1 : 2 and the period for A’s investment was 7 months, B
43. Find the approximate average of Murder and Theft in all invested the money for
the eight states together. (a) 6 months (b) 2 ½ months (c) 10 months
(a) 1141 (b) 1132 (c) 1311 (d) 4 months (e) 7 months
(d) 941 (e) 1021 55. An army lost 10% its men in war, 10% of the remaining
44. The total number of Assaults and Murders together in due to diseases and 10% of the rest were disabled. Thus,
Bihar is what per cent of the total number of crimes in the strength was reduced to 729000 active men. Find the
that state? original strength.
(a) 29.82% (b) 39.82% (c) 25% (a) 1000000 (b) 1200000 (c) 1500000
(d) 21.82% (e) 25.5% (d) 1800000 (e) none of these
45. Find the difference between the number of various
crimes committed in Bihar and that in Rajasthan.

www.bankersadda.com | www.sscadda.com | www.careerpower.in | www.careeradda.co.in


56. What is the difference between the compound interests (a) (b) (c)
on Rs. 5000 for 1 years at 4% per annum compounded
yearly and half-yearly? (d) (e)
(a)2 (b)3 (c)4 63. What would be the area of a rectangle whose area is
(d)8 (e)none of these equal to the area of a circle of radius 7 cm?
57. The speeds of John and Max are 30 km/h and 40 km/h. (a) 77 cm2 (b) 154 cm2 (c) 184 cm2
Initially Max is at a place L and John is at a place M. The (d) 180 cm2 (e) 150 cm2
distance between L and M is 650 km. John started his 64. In a village three people contested for the post of village
journey 3 hours earlier than Max to meet each other. If Pradhan. Due to their own interest, all the voters voted
they meet each other at a place P somewhere between L and no one vote was invalid. The losing candidate got
and M, then the distance between P and M is : 30% votes. What could be the minimum absolute margin
(a) 220 km (b) 250 km (c) 330 km of votes by which the winning candidate led by the
(d) 320 km (e) None of these nearest rival, if each candidate got an integral per cent of
58. The average weight of boys in a class is 30 kg and the votes?
average weight of girls in the same class is 20 kg. If the (a) 4 (b) 2 (c) 1
average weight of the whole class is 23.25 kg, what could (d) 3 (e) None of these
be the possible strength of boys and girls respectively in 65. The price of an article is first increased by 20% and later
the same class? on the price were decreased by 25% due to reduction in
(a) 14 and 26 (b) 13 and 27 (c) 17 and 27 sales. Find the net percentage change in final price of
(d) 19 and 21 (e) None of these Article.
59. A profit of 8% is made by selling a shirt after offering a (a) 20% (b) 18% (c) 38%
discount of 12%. If the marked price of the shirt is (d) 10% (e) None of these
Rs.1080, find its cost price Directions (66–70): What will come in the place of the
(a) 890 (b) 780 (c) 880 question mark (?) in the following number series?
(d) 900 (e) none of these 66. 48, 23, ?, 4.25, 1.125
60. The difference between of a number and 45% of the (a) 10.5 (b) 10 (c) 2.5
(d) 11 (e) None of the above
number is 56. What is 65% of the number?
67. 2, 15, 41, 80, 132, ?
(a) 96 (b) 104 (c) 112
(a) 197 (b) 150 (c) 178
(d) 120 (e) None of these
(d) 180 (e) None of the above
61. A man can row 24 km upstream and 54 km downstream
68. ?, 15, 75, 525, 4725, 51975
in 6 hours. He can also row 36 km upstream and 48 km
(a) 5 (b) 10 (c) 8
downstream in 8 hours. What is the speed of the man in
(d) 6 (e) None of the above
still water?
69. 4, 19, 49, ?, 229
(a) 18.75 kmph (b) 19.25 kmph
(a) 75 (b) 109 (c) 65
(c) 17.65 kmph (d) 15.55 kmph (e) 22.75 kmph
(d) 169 (e) None of the above
62. The numerator of a fraction is decreased by 25% and the
70. 840, ?, 420, 140, 35, 7
denominator is increased by 250%. If the resultant
(a) 408 (b) 840 (c) 480
fraction is , what is the original fraction? (d) 804 (e) None of the above

www.bankersadda.com | www.sscadda.com | www.careerpower.in | www.careeradda.co.in


ENGLISH LANGUAGE

Directions (Q. 71-80): Read the passage carefully and d) Your relationship with your friend is the most important
answer the questions given below it. Certain words/ one that can influence your happiness.
phrases have been given in bold to help you locate e) The most important relationship you can have is the one
them while answering some of the questions. you have with yourself.
One day all the employees reached the office and they saw 73. What was inside the coffin?
a big advice on the door on which it was written: a) The corpse of the employee who died.
“Yesterday the person who has been hindering your b) The coffin was empty.
growth in this company passed away. We invite you to c) A note on which there were instructions.
join the funeral in the room that has been prepared in the d) There was a mirror inside the coffin.
gym”. In the beginning, they all got sad for the death of e) Photos of every employee.
one of their colleagues, but after a while they started 74. What was the reason of employee’s excitement?
getting curious to know who was that man who hindered a) As they were expecting a bonus or promotion.
the growth of his colleagues and the company itself. b) Because the employee who hindered their growth died.
The excitement in the gym was such that security agents c) As they were happy they will succeed now since no one
were ordered to control the crowd within the room. The will hinder their growth now.
more people reached the coffin, the more the excitement d) They were excited because what they found inside the box
heated up. Everyone thought: “Who is this guy who was was completely opposite of what they expected.
hindering my progress? Well, at least he died!” One by one e) They were curious to know the identity of the man who
the thrilled employees got closer to the coffin, and when hindered the growth of his colleagues.
they looked inside it they suddenly became speechless. 75. Why everyone was speechless and shocked?
They stood nearby the coffin, shocked and in silence, as if a) They were shocked as the employee who died was the
someone had touched the deepest part of their soul. There most hardworking one.
was a mirror inside the coffin: everyone who looked b) As they never expected that their growth was actually
inside it could see himself. hindered by one of their own.l
There was also a sign next to the mirror that said: “There c) As they found a mirror inside the coffin.
is only one person who is capable to set limits to your d) As one of the employees died.
growth: it is YOU.” You are the only person who can e) They were shocked to find that coffin was empty.
revolutionize your life. You are the only person who can Directions (Q. 76-78): Choose the word/group of words
influence your happiness, your realization and your which is MOST SIMILAR in meaning to the word / group
success. You are the only person who can help yourself. of words printed in bold as used in the passage.
Your life does not change when your boss changes, when 76. THRILLED
your friends change, when your partner changes, when a) Tedious
your company changes. Your life changes when YOU b) Monotonous
change, when you go beyond your limiting beliefs, when c) Delight
you realize that you are the only one responsible for your d) Dull
life. “The most important relationship you can have is the e) Boring
one you have with yourself.’’ 77. INFLUENCE
a) Insignificance
71. Which of the following is true in context of the b) Domination
passage? c) Underwhelming
a) The crowd gathered in the office for the funeral. d) Triviality
b) The coffin was empty. e) Weakness
c) The employee who died was hindering everybody’ 78. REVOLUTIONIZE
growth. a) Harmony
d) Only a person is responsible for his own success. b) Calm
e) The crowd loved the employee who died. c) Uprising
d) Stagnation
72. What can we learn from this passage? e) Obedience
a) Never trust anyone without confirming it yourself. Directions (Q. 79-80): Choose the word/group of words
b) One who is not hard working is not loved by the which is MOST OPPOSITE in meaning of the word/ group
employees. of words printed in bold as used in the passage.
c) Good relation with your employees is most important in 79. FUNERAL
your life. a) Nativity

www.bankersadda.com | www.sscadda.com | www.careerpower.in | www.careeradda.co.in


b) Burial 90. To the men (a)/ who worked so hard (b)/ in the
c) Cremation project the news was(c)/ profound disappointing. (d)/No
d) Entombment error (e).
e) Inhumation
80. HINDERED Directions (91-95): In each sentence below four
a) Hamper words have been printed in bold which are numbered
b) Inhibit (4), (2), (3) and (4) One of these words may be
c) Impede misspelt or inappropriate in the context of the
d) Retard sentence. Find out the wrongly spelt or inappropriate
e) Expedite word. The number of that word is the answer. If all
the words are correctly spelt and are appropriate the
Directions (Q. 81-85): Rearrange the following six answer is (5) i.e. ‘All correct.'
sentence (A), (B), (C), (D), (E) and (F) in the proper
sequence to form a meaningful paragraph and then 91. Rising (1)/prices (2)/of food grains will have an adverse
answer the questions given below. impac (3)/on developing (4)/countries. All correct (5)
A. The man who bought it immediately put it up for auction. 92. To deal effectively (1)/with a crisis (2)/quick decisions
B. And he went on to list the many qualities of the animal. At (3)/are requited. (4)/All correct (5)
the end of his sales talk a man said he would give 40 93. The IT Company has succeeded (1)/in achieving high
dinars for it. growth rate despite (2)/facing (3)/several
C. "Look at this fine animal!" he shouted to passersby. "Have (4)/problems. All correct (5)
you ever seen a better specimen of a donkey? See how 94. On an average (1)/there are very fern persons willingly
clean and strong it is!" (2)/to take on (3)/such responsibility. (4)/All correct
D. Nasruddin Hodja took his donkey to the market place and (5)
sold it for 30 dinars. 95. More than halve (1)/the budget (2)/has been spent
E. Another man offered 50. A third offered 55. (3)/on modernizing (4)/the factory/All correct (5)
F. Hodja who was watching was amazed at the interest
everyone was showing in the donkey. Directions (Q.96-100): In the following passage, some
81. Which of the following should be the FIRST sentence after of the words have been left out, each of which is
rearrangement? indicated by a number. Find the suitable word from
(a)F (b)E (c)B (d) A the options given against each number and fill up the
(e)D blanks with appropriate words to make the
82. Which of the following should be the SECOND sentence paragraph meaningfully complete.
after rearrangement?
(a)A (b)B (c)C (d)E Jack went to work for a Farmer and (96) a penny. But
(e) D while returning home, he dropped it in a brook. At home,
83. Which of the following should be the THIRD sentence his mother (97) him and told him to put his earnings in
after rearrangement? his pocket the next time.The next day, he worked for a
(a)E (b)F (c)D (d)C cow keeper, who (98) him a jar of milk. Jack tried to put
(e)B the jar of milk in his pocket and spilled it everywhere.
84. Which of the following should be the FOURTH sentence Once again, his mother rebuked him. She told him that he
after rearrangement? should have carried it on his shoulders. The next day, Jack
(a)B (b)G (c)F (d)E was given a donkey. He carried the donkey on
(e)C his shoulders. Now, the king had a daughter who never
85. Which of the following should be the LAST (SIXTH) laughed. (99) Jack carrying the donkey on his shoulders,
sentence after rearrangement? she (100) laughing for the first time.
(a)C (b)A (c)B (d)D
(e)F 96. (a) Collected (b)Earned (c)Had (d)
Directions (Q.86-90): Read each sentence to find out Accumulated (e)Make
whether there is any grammatical or idiomatic error 97. (a) Praised
in it. The error, if any, will be in one part of the (b)complimented(c)Scolded (d)lambaste
sentence. The number of that part is the answer. If (e)Criticize
there is ‘No error’, the answer is 5). (Ignore errors of 98. (a) Sold (b)Provide (c) Made
punctuation, if any.) (d) Gave (e)Issued
86. Suppose, if you (a)/ were left alone to (b)/ live on a 99. (a) Detecting (b) Seeing
desert island(c)/ what would you do? (d)/ No error (e). (c)Watching (d)Recognizing (e) Sawing
87. He wondered that what (a)/would be the next move 100. (a) Start (b)Stopped
(b)/of his opponents who had (c)/ vowed to see him (c)commence (d)Initiated (e) Began
dislodged from power? (d)/No error (e).
88. The nation should (a)/ be grateful to (b) the armed
forces for (c)/ protecting them. (d)/No error (e).
89. For so many years(a)/it is almost his habit (b)/ to go
to the bed (c)/ at 10 pm daily. (d)/No error (e).

www.bankersadda.com | www.sscadda.com | www.careerpower.in | www.careeradda.co.in


REASONING SOLUTIONS

Direction (1-5) Direction (23-25)


1. (a) L>E (True) allli
2. (b) aspirantsna
3. (a) mustpa
4. (d) qualifyja
5. (e) inta
Direction (6-10) subjectra
6. (b) Only 385 will be divisible by 3 when added readsa
2 on first digit of each number. 23. (d)
7. (c) 864 521 743 853 962 24. (e)
8. (a) 8÷4=2 25. (c)
9. (e) 786 614 539 487 398 Direction (26-27)
10. (a) 864 152 347 845 926
Direction (11-15)
11. (a) L=7th
R=6th
--------------
R=13th
12. (c) %9,#7,©4
13. (e) E@Z
14. (d)$L ,  B, @Z
15. (b) L=4th 26. (a)
R=12th 27. (d)
------------------ 28. (d) 5 4 3 2 7 6 1 8
R=16th
Direction (16-20) 12345678
Hence, there are three numbers
29. (a) x is 20th from the back.
The position of w from the
back is (20-5) = 15th
Hence the position of W from the front is
(34-15+1) = 20th

30. (d)
16. (a)
Direction (31-35)
17. (d)
18. (c) PERSONS DAYS
19. (b) A Saturday
20. (e)
21. (b) B Friday
S E R I E S
C Wednesday
Q C G T G U
D Thursday
E X P E R T E Monday

R P C R Z G F Tuesday
22. (d)
31. (b)
32. (c)
33. (b)
34. (c)
35. (c)
www.bankersadda.com | www.sscadda.com | www.careerpower.in | www.careeradda.co.in
MATHS

36. (a); 54. (c); Let B invested money for months.


37. (a);
38. (c);
39. (b);
months
40. (d); 55. (a); Let initial men = 100
Lost in war
41. (d); Total No. of crimes in HP = 36903
42. (b); Ratio = 210 : 520 Lost in diseases
= 21 : 52 Disables
43. (e); = Remaining men = 72.9
= When 72.9 remaining total men = 100
When 729000 remaining total men = 1000000
1021
56. (a); When compounded yearly,
44. (b); Required = Student = 200
= 39.82% When compounded half – yearly
45. (c); Required difference = 1577 – 1432 = 145 r = 2%, n = 2
46. (c); ?= 7682 – 4909 = 2773  interest = 202
47. (b); √ √  difference = 202 – 200
? = 1369 57. (d); speed of john = 30 km/hr
48. (c); Speed of max = 40 km/hr
Let distance b/w p and m = x km
x = 35
49. (d); 980 =
50. (b); ? = 367.5 – 355.2 7x = 2240
= 12.3 x = 320 km
51. (a); Let Required quantity 58. (b); Let Boys
Girls
( )

52. (e); Let sum

59. (c); Cost Price


60. (b);
(80 – 45) = 35% of the no. = 56
65% of the no. =
53. (a); Let men’s 1 day work 61. (b) ( )
Let women’s 1 day work
( )
………………(i)
eqn (1) × 3 – eqn (2) × 2
……………..(ii)
By solving eqn. (i) and (ii) —

Required days
Put in the eqn (1)

www.bankersadda.com | www.sscadda.com | www.careerpower.in | www.careeradda.co.in


Required minimum margin
Speed of the man in still water 65. (d); Net Change = 20 – 25 –
=0–5–5
= – 10%
19.25 kmph
66. (a); ÷ 2 – 1 = 23, ÷ 2 – 1 = 10.5, ÷ 2 – 1 = 4.25……
62. (d) 67. (a); 2 + 13 = 15, 15 + 26 = 41, 41 + 39 = 80, 80 +
52 = 132
132 + 65 = 197
75x = 420y 68. (a); 51975 ÷ 11 = 4725, 4725 ÷ 9 = 525,
525 ÷ 7 = 75, 75 ÷ 5 = 15,
15 ÷ 3 = 5
69. (b); 4 + 15 = 19, 19 + 30 = 49, 49 + 60 = 109,
109 + 120 = 229
63. (b) Required area = 70. (b); 840 ÷ 1 = 840, 840 ÷ 2 = 420, 420 ÷ 3 = 140,
= 154 cm2 140 ÷ 4 = 35, 35 ÷ 5 = 7
64. (b); Since winning candidate and his rival got
70% of total votes.

ENGLISH LANGUAGE SOLUTIONS

71.(d);Refer to the last paragraph, ''you are the only person who can influence your happiness, your
realization and your success.''
72.(e);Refer to the last paragraph, “the most important relationship you can have is the one you have
with yourself.''
73.(d);Refer to second paragraph, ''there was a mirror inside the coffin: everyone who looked inside it
could see himself''.
74.(e);they were excited about the identity.
75.(c);Refer to the second paragraph it is explained there that they were shocked to see the mirror.
76.(c);thrill means cause (someone) to have a sudden feeling of excitement and pleasure hence delight is
most similar in meaning.
77.(b);influence means the capacity to have an effect on the character, development, or behaviour of
someone or something, or the effect itself hence domination is the word most similar in meaning.
78.(c);Revolutionize means a forcible overthrow of a government or social order, in favour of a new
system hence uprising is the word most similar in meaning.
79.(a);Funeral means a ceremony or service held shortly after a person's death, usually including the
person's burial or cremation hence nativity is the word most opposite in meaning.
80.(e);hindered means make it difficult for (someone) to do something or for (something) to happen
hence expedite is the word most opposite in meaning.
FOR questions(81-85);The correct sequence is DACBEF
81.(e);D 82.(a);A
83.(d);C 84.(a);B
85.(e);F 86.(a);Remove 'suppose'.
87.(a);Remove 'that'. 88.(e);No error.
89.(c);Remove 'the' 90.(d);Use ‘profoundly’ in place of ‘profound’.
91.(3); Change ‘impac’ into ‘impact’ 92.(4); Change ‘requited’ into ‘required’
93.(5); 94.(2); Change ‘willingly’ into ‘willing’
95.(1); Change ‘halve’ into ‘half’ 96.(b);
97.(c); 98.(d);
99.(b); 100.(e);

www.bankersadda.com | www.sscadda.com | www.careerpower.in | www.careeradda.co.in


Part - 7

Directions (1-8): Read the given passage and answer the following questions based on that.

Home is where the heart is, or so goes the adage. But the human heart did not always crave a still hearth.
In fact, before the emergence of agrarian societies, the early humans journeyed along the roads taken by
their prey — hunter-gatherer societies were mobile and the road was their life. But with the rise of
agrarianism, humans learnt to put down their roots and a life of journeys became something to be frowned
upon. Yet, vestiges of the wandering spirit persist. In the year, 2000, Herman and Candelaria Zapp set out
in a car from Buenos Aires to travel the world. The Zapps’ home is the road — literally and proverbially.
They have had four children in different countries, and in an era of strict visa regimes and rising insularity,
managed to treat borders as what they truly are — arbitrary and artificial lines on a map. The world was a
different place when the Zapps set out — there were no smartphones, the internet was in its infancy, a site
of knowledge and hope rather than disinformation and division, the twin towers towered over New York,
and Russia was a diminutive shell stripped of its Soviet grandeur. It is tempting to imagine that the Zapps
__________________ Kishore Kumar’s immortal song, “Musafir hoon yaaron, na ghar hai na thikana”, as they
covered great distances across a changing world.
The Zapps, interestingly, are not an anomaly. Latest estimates show that over one million Americans live
permanently in recreational vehicles with no permanent address. Many are retired, but many others work
part of the year, just long enough to keep them going for a few months or so. It is easy to romanticize the
lifestyle and the idea of freedom that it brings. But in most countries, people without a stationary home are
subjected to intense legal scrutiny. Law enforcement institutions are even prejudiced against specific
communities. For example, in the United States of America, there are legislations that make it illegal to sit
on the sidewalk, to camp or sleep anywhere in a downtown area or to hang around in a public place without
an apparent purpose. The intent seems to encourage sanitization of space of some kinds of human
presences. These laws are not unique to the US. Anti-vagrancy laws have existed for hundreds of years —
think of the persecution of gypsies across Europe. In the old United Kingdom, vagrants could be put in jail,
sold as slaves, or sometimes even killed for no other reason than wandering. Colonial India, too, had
witnessed the purge of nomadic communities, most notably under the guise of the campaign against
thuggees.

Q1. According to the passage, what changes has human society seen from the pre-agrarian era to the
present?
(a) Modern humans have diverged their dependency from
agriculture, unlike their primitive ancestor whose livelihood was
solely based on that
(b) From being a hunter-gatherer nomad society, it evolved into a
settled agrarian society
(c) The immense technological advancement in recent years makes it
possible to envisage services that were seen as utopian in pre-
agrarian era
(d) All of these
(e) None of these

127 adda247.com/teachers | www.sscadda.com | www.bankersadda.com | www.adda247.com


Q2. What was NOT the global situation when the Zapps embarked on their journey?
(a) The world was witnessing the inception of the internet
(b) The twin towers were standing tall in New York
(c) Post dissolution of Soviet Union, Russia was still in its timid state.
(d) The world was going through an acute economic crisis due to surge in oil prices.
(e) None of these

Q3. How do the itinerant Americans sustain their lives?


(a) Few of them are retired and thus continued their rest of the lives by travelling on their recreational
vehicles
(b) Some of them are engaged in work for a few months of a year and use the rest for traveling
(c) Prominent travel companies and independent organisations sponsor those people who actively travel
(d) Only (a) and (b)
(e) Only (b) and (c)

Q4. How are the countries treating the new age wanderer?
(a) They are subject to legal scrutiny in many countries because of their lack of possession of land.
(b) Many countries assess their livelihood and put their earnings on the tax slab.
(c) Many law enforcement agencies are biased towards these travelers
(d) Only (a) and (c)
(e) All of these

Q5. Which of the following is FALSE according to the data given in the passage?
(a) Thuggees are predominantly nomadic people of colonial India
(b) Germany has a prohibition law for sitting on the sidewalks.
(c) UK has criminalized wandering, to the extent of giving out death sentences.
(d) The Zapps couple had started off from Buenos Aires in the year 2000.
(e) none of these

Q6. Which of the following words can fit into the blank given in the passage?
(a) hummed
(b) iterated
(c) cherished
(d) longed
(e) enunciated

Q7. Which word from among the following words most closely represents the antonym of the highlighted
word “infancy” keeping in consideration its intended meaning in the passage?
(a) service
(b) apprehension
(c) perseverance
(d) maturity
(e) intelligence

128 adda247.com/teachers | www.sscadda.com | www.bankersadda.com | www.adda247.com


Q8. Choose the word from among the following which is most similar in meaning to the highlighted word
“insularity” in accordance with its meaning in the given text.
(a) irritation
(b) inflexibility
(c) disbelief
(d) freedom
(e) awareness

Directions (9-14): In the following passage there are blanks, each of which has been numbered. These
numbers are printed below the passage and against each, five options are given. Find out the appropriate
word which fits the blank appropriately.
Filmmaking research is a form of practice research that ______________ (9) into production practices,
techniques, modes and genres used in cinema, television and online. The outputs are films that may
include__________________ (10), documentary and hybrid forms. Filmmaking research pushes at the
boundaries of traditional filmmaking and traditional research methods by adopting distinct ____________
(11) to professional and critical practices. This is in contrast to typical industry research where commercial
pressures dominate. There have been _____________ (12) success stories that demonstrate this potential.
Filmmaking research is a developing area and films produced within the academy are growing in number.
In both the UK and Australia, this research has tended to _______________(13) in one of two disciplines,
creative arts (fine art, experimental, video art) or media, communication and film studies (fiction film,
documentary). This has made it fragmented, sometimes difficult to identify and subject to different forms
of support and____________________ (14). Researchers come from a range of backgrounds, many moving from
industry into academia seeking to reframe their work within Higher Education.

Q9. (a) probes


(b) enquires
(c) inquests
(d) surveys
(e) investigate

Q10. (a) friction


(b) segregation
(c) accede
(d) fiction
(e) purport

Q11. (a) regulations


(b) adversaries
(c) approaches
(d) assistances
(e) intervals

129 adda247.com/teachers | www.sscadda.com | www.bankersadda.com | www.adda247.com


Q12. (a) expressed
(b) threatening
(c) overlooked
(d) mocking
(e) spectacular

Q13. (a) reside


(b) accommodate
(c) occupy
(d) fuller
(e) lament

Q14. (a) ample


(b) wither
(c) engagement
(d) enrollments
(e) alliance

Directions (15): A word has been given below, and following it are provided three sentences which use
the word. Mark the correct option from the options given below which indicates the sentence or
combination of sentences with correct usage of the word.

Q15. pore
(i) After I was given the manuscript for my first title role in the play, I began to pore over the script for
hours.
(ii) It is pore that people show identification before being allowed to board a commercial aircraft.
(iii) Before I sign the contract, I decided it would be best for me to pore through it so that the job would
run smoothly.
(a) Both (i) and (ii)
(b) Both (i) and (iii)
(c) Only (ii)
(d) Both (ii) and (iii)
(e) All of these

Directions (16-20): Given below are five sentences, each of which has been jumbled up in an order which
makes the text incoherent. Mark the option which gives the meaningful rearrangement of the parts of the
sentence as the answer.

Q16. The property, they (A) / had to undergo a (B) / in order to purchase (C) / lengthy application process
(D).
(a) BDAC
(b) DACB
(c) ADBC
(d) CABD
(e) No rearrangement required

130 adda247.com/teachers | www.sscadda.com | www.bankersadda.com | www.adda247.com


Q17. Constituted the battlefields (A) / of World War Two, encompassing (B) / much of the globe (C) / many
conflicts and fronts (D).
(a) DABC
(b) BACD
(c) ADCB
(d) BDCA
(e) No rearrangement required

Q18. We received a package (A) / not notice the mail (B) / yesterday but did (C) /until early this morning
(D).
(a) DCBA
(b) BDAC
(c) ACBD
(d) CADB
(e) No rearrangement required

Q19. The venomous black widow (A) / are often killed (B) / because they resemble (C) / perfectly harmless
spiders (D).
(a) BCDA
(b) DBCA
(c) ADCB
(d) CABD
(e) No rearrangement required

Q20. Look for new jobs (A) / were rigid and uncompromising, (B) / leaving the workers to (C)/ many of
the company’s rules (D).
(a) CDBA
(b) ACBD
(c) DBCA
(d) CABD
(e) No rearrangement required

Directions (21-25): Given below are five sentences, with four words highlighted in each sentence. One of
these words may be the misspelt form of a word, making the sentence contextually incoherent. Mark the
option which depicts the corresponding letter to the incorrectly spelled word, or mark option (e) if there
is none.

Q21. Pat didn’t mean to loose (A) her mother’s favourite (B) bracelet but hasn’t been able to locate (C) it
since she wore (D) it last week.
(a) A
(b) B
(c) C
(d) D
(e) No spelling errors

131 adda247.com/teachers | www.sscadda.com | www.bankersadda.com | www.adda247.com


Q22. The unweary (A) gamblers lost all their (B) money when they
carelessly (C) put all their chips on the wrong (D) tile.
(a) A
(b) B
(c) C
(d) D
(e) No spelling errors

Q23. Taking a position as curator (A) at the museum (B), the woman
hoped to spread (C) her love for sports memorablia (D) to others.
(a) A
(b) B
(c) C
(d) D
(e) No spelling errors

Q24. As my father’s only surviving (A) sister, my aunt took great (B) care of my aging (C) grandparents
until they both passed away recently (D).
(a) A
(b) B
(c) C
(d) D
(e) No spelling errors

Q25. Because of technical (A) difficulties (B), the sound of the music floctuated (C) during the concert (D).
(a) A
(b) B
(c) C
(d) D
(e) No spelling errors

Directions (26-30): Which of the phrase/ word from the options (a), (b), (c) and (d) given below each
sentence should replace the phrase printed in bold letters to make the sentence grammatically correct? If
the sentence is correct as it is, mark (e) i.e., “No replacement required” as the answer.

Q26. Jeff was given a medal of honor in recognition by the twenty-five years he spent fighting for his
country.
(a) of recognition by
(b) in recognition of
(c) by recognition of
(d) in recognition at
(e) No correction required

132 adda247.com/teachers | www.sscadda.com | www.bankersadda.com | www.adda247.com


Q27. INS Vikrant is a unique reflection of India becoming self-reliance.
(a) becomes self-reliant
(b) became self-reliant
(c) become self-reliance
(d) becoming self-reliant
(e) No replacement required

Q28. There have been a steady spike in cases of cybercrime in the last five years.
(a) has been a stead
(b) has been a steady
(c) has been a steadily
(d) has a steady
(e) No replacement required

Q29. It is vital that every Indian takes a pledge to eliminate discrimination and enable equity.
(a) take a pledge to eliminate
(b) took a pledge to eliminate
(c) takes a pledges to eliminate
(d) take a pledge to elimination
(e) No replacement required

Q30. He was worried that the anger would erupt and led to a law-and-order situation.
(a) would erupts and lead
(b) would erupt and lead
(c) would erupted and led
(d) would erupt and leads
(e) No replacement required

Direction (31 – 35): What will come in the place of question (?) mark in following number series.

Q31. 50, 75, 175, 400, 800, ?


(a) 1435
(b) 1425
(c) 1415
(d) 1405
(e) 1375

Q32. 8, 18, 38, 78, ?, 318


(a) 178
(b) 168
(c) 154
(d) 148
(e) 158

133 adda247.com/teachers | www.sscadda.com | www.bankersadda.com | www.adda247.com


Q33. 41, ?, 53, 64, 77, 94
(a) 48
(b) 46
(c) 43
(d) 49
(e) 41

Q34. 215, ?, 89, 47, 19, 5


(a) 136
(b) 139
(c) 143
(d) 145
(e) 141

Q35. 1, 9, 36, 100, 225, ?


(a) 441
(b) 443
(c) 447
(d) 431
(e) 451

Direction (36 – 40); Line graph given below shows total number of boats and ships manufactured by four
different companies in 2020 and it also show total number of ships manufactured by these four companies.
Read the line graph carefully and answer the questions.
250

200

150

100

50

0
A B C D
total number of (boats + Ships) manufactured
total number of ships manufactured

134 adda247.com/teachers | www.sscadda.com | www.bankersadda.com | www.adda247.com


2
Q36. If 3rd of total boats manufactured by B are used by general people and rest used by fishermen, then
find total boats used by fishermen are what percent of total ships manufactured by A?
(a) 30%
(b) 33.33%
(c) 40%
(d) 37.5%
(e) 50%

Q37. If total number of boats and ships manufactured by A in 2021 is 40% more than that of in year 2020
and total number of ships manufactured by A in 2021 is equal to that of in 2020, then find the total number
of boats manufactured by A in 2021?
(a) 115
(b) 105
(c) 75
(d) 85
(e) 120

Q38. Find the ratio of total boats manufactured by C & D together to total ships manufactured by B & C
together?
(a) 6 : 7
(b) 6 : 5
(c) 5 : 4
(d) 3 : 4
(e) 3 : 5

Q39. If manufacturing cost of each boat for company D is Rs. 4600, then find the total cost (in Rs.) of all
boats manufactured by D?
(a) 240000
(b) 236000
(c) 233000
(d) 220000
(e) 230000

Q40. Total boats and ships manufactured by A & B together is what percent more than total boats
manufactured by A & D together?
(a) 300%
(b) 175%
(c) 250%
(d) 400%
(e) 350%

135 adda247.com/teachers | www.sscadda.com | www.bankersadda.com | www.adda247.com


Q41. Six years ago, sum of age of Amit and Den is 20 years. Mohit is six years younger than Amit and the
average of present age of Mohit and Den is 13 years. Find the present age of Den?
(a) 16 years
(b) Can’t determined
(c) 20 years
(d) 14 years
(e) 24 years

Q42. The perimeter of semicircle is 36 cm and the radius of the semicircle is equal to the breath of a
rectangle. If area of rectangle is 147 cm2, then find the length of rectangle?
(a) 10.5 cm
(b) 28 cm
(c) 14 cm
(d) 21 cm
(e) 35 cm

Q43. Sum of six consecutive odd numbers is 150 more than average of these six numbers. Find the sum of
highest and lowest number?
(a) 60
(b) 50
(c) 70
(d) 45
(e) 55

Q44. P and Q started a business by investing Rs. X and Rs. (X + 100) respectively. After three months, P
withdrew 40% of his initial investment. If at the end of year, P got Rs. 210 as profit share out of total profit
of Rs. 530, then find the value of X?
(a) 1750
(b) 1250
(c) 2500
(d) 1500
(e) 2000

Q45. A man invested Rs. P on simple interest at 18% p.a. for three years and received Rs. (0.5P + 100) as
interest. If man invested Rs. (P + 3500) on compound interest at 10% p.a. for two years, then find amount
gets by man?
(a) 7240 Rs.
(b) 7260 Rs.
(c) 7280 Rs.
(d) 7460 Rs.
(e) 7040 Rs.

136 adda247.com/teachers | www.sscadda.com | www.bankersadda.com | www.adda247.com


Q46. Pipe A can fill a tank in 4 hours and pipe B can fill the same tank in 6 hours. If A, B & pipe C can together
fill the same tank in 3 hours, then find time taken by pipe C to empty the tank?
(a) 24 hours
(b) 12 hours
(c) 16 hours
(d) 8 hours
(e) 18 hours

Q47. The ratio of marked price to selling price of an article is 5 : 3 and shopkeeper gets a profit of 40%. If
shopkeeper sells the article at its marked price, then the difference between cost price and marked price
of article is Rs. 400. Find the cost price of the article?
(a) 540 Rs.
(b) 480 Rs.
(c) 600 Rs.
(d) 300 Rs.
(e) 360 Rs.

Q48. A vessel contains mixture of milk and water in the ratio of 7 : 3. If 50 liter of milk is added into the
mixture, then the quantity of milk becomes 80% of the resulting mixture. Find the initial quantity of milk
in the mixture?
(a) 112 liters
(b) 105 liters
(c) 84 liters
(d) 70 liters
(e) 56 liters

Q49. Ram alone can complete a work in 40 days. If Ram and Shayam together take 37.5% less time than
Ram to complete the same work, then find in how many days Shayam alone can complete the work?
2
(a) 56 3 days
(b) 58 days
(c) 50 days
2
(d) 66 3 days
1
(e) 33 3 days

Q50. Arun spent 60% of his monthly salary on accommodation, 15%


of remaining salary on entertainment and remaining salary he saved.
If the difference of amount spent on accommodation and his saving is
Rs 3510, then find his monthly salary?
(a) Rs 12,500
(b) Rs 13,500
(c) Rs 12,800
(d) Rs 14,000
(e) Rs 15,000

137 adda247.com/teachers | www.sscadda.com | www.bankersadda.com | www.adda247.com


Q51. The length of train A is half of the length of train B and the ratio of speed of train A to B is 4 : 3. If train
A & B can cross 100 meters long platform in 5 sec. and 10 sec. respectively, then find the length (in meters)
of train B?
(a) 200
(b) 180
(c) 140
(d) 240
(e) 120

Q52. The ratio of speed of boat in downstream to that of in upstream is 4 : 1 and total time taken by boat
to cover a certain distance in downstream and upstream together is 10 hours. Find the time taken by boat
to cover the same distance in still water?
(a) 3 hours and 24 minutes
(b) 3 hours and 12 minutes
(c) 3 hours and 10 minutes
(d) 3 hours and 15 minutes
(e) 3 hours and 20 minutes

Direction (53 – 65): What will come in the place of question (?) mark in following questions.

Q53. 64% 𝑜𝑓 2500 + 75% 𝑜𝑓 1600 = ? × 112


(a) 28
(b) 21
(c) 24
(d) 32
(e) 25

Q54. 55% of 1400 + ?2 +282= (12)3


(a) 28
(b) 24
(c) 26
(d) 36
(e) 16

7.5% 𝑜𝑓 7200
Q55. + 450= 15 % of 3200
?
(a) 15
(b) 24
(c) 20
(d) 18
(e) 12

138 adda247.com/teachers | www.sscadda.com | www.bankersadda.com | www.adda247.com


Q56. ?2 + 224 – 96 = 85% 𝑜𝑓 280 + 34
(a) 18
(b) 12
(c) 16
(d) 14
(e) 8

Q57. √1521 + (21)2 − 18 × 5 = 5 × ?


(a) 80
(b) 78
(c) 64
(d) 70
(e) 96

Q58. ? × 7 + 337 = (5)3 + 436


(a) 36
(b) 44
(c) 48
(d) 45
(e) 32

Q59. ?÷ 6 + (12)2 − √2304 = (14)2


(a) 1100
(b) 400
(c) 600
(d) 900
(e) 1500

Q60. (?× 48) ÷ 54 + 82 = 96


(a) 45
(b) 36
(c) 72
(d) 54
(e) 27

1 2 3 3
Q61. 2 + 5 − 3 = ? − 8
3 3 4 4
(a) 14
(b) 13
(c) 12
(d) 15
(e) 16

139 adda247.com/teachers | www.sscadda.com | www.bankersadda.com | www.adda247.com


25 ×36
Q62. 92 ×42 + 7 = (? )2
(a) 4
(b) 5
(c) 7
(d) 6
(e) 3

Q63. 9097÷ 11 ÷ 10 = ? +15.7


(a) 87
(b) 77
(c) 67
(d) 57
(e) 47

Q64. 15×12-10×11 = -15×17 + ?


(a) 330
(b) 345
(c) 335
(d) 325
(e) 365

Q65. 35×5 + 19 × 7 + ? = (20)²


(a) 95
(b) 103
(c) 97
(d) 92
(e) 90

Q66. Find the odd one out?


(a) DGJE
(b) KNQL
(c) UXZU
(d) CFID
(e) MPSN

Direction (67-70): Study the following information carefully and answer the questions given below:

A certain number of persons sit in a row and all of them face towards the north. Q sits second to the right
of M. Two persons sit between J and Q. W sits second to the right of J and sits third from the extreme right
end. Five persons sit between W and S. The number of persons sits to the left of S is same as the number of
persons sits to the right of W. K is the only neighbour of P. K sits to the right of J.

140 adda247.com/teachers | www.sscadda.com | www.bankersadda.com | www.adda247.com


Q67. How many persons sit in the row?
(a) 13
(b) 11
(c) 10
(d) 15
(e) 17

Q68. What is the position of Q with respect to W?


(a) Fourth to the left
(b) Fifth to the right
(c) Second to the right
(d) Fifth to the left
(e) Third to the left

Q69. Which among the following statement(s) is/are true?


I. P sits at the extreme right end
II. More than three persons sit to the left of Q
III. M sits adjacent to S
(a) Both I and II
(b) Both II and III
(c) Only I
(d) Both I and III
(e) Only III

Q70. Four of the following five are alike in a certain way and thus form a group. Who among the following
does not belong to the group?
(a) M
(b) W
(c) P
(d) Q
(e) K

Directions (71-73): In these questions, relationship between different elements is shown in the
statements. The statements are followed by two conclusions. Give answer.
(a) If only conclusion I is true
(b) If only conclusion II is true
(c) If either conclusion I or II is true
(d) If neither conclusion I nor II is true
(e) If both conclusions I and II are true

Q71. Statements: Q≥W>Y=E≤U<P>S≤T


Conclusions I: Y<P II: W>S

141 adda247.com/teachers | www.sscadda.com | www.bankersadda.com | www.adda247.com


Q72. Statements: M≤B>H=A≥D<L=C
Conclusions I: H>L II: A≤C

Q73. Statements: K>M≤Z<E≥V=X<C


Conclusions I: E≥C II: V<M

Directions (74-78): Study the following information carefully and answer the questions given below:

Eight persons A, B, C, D, E, F, G and H sit around a square table such that four persons sit at corner and the
remaining four sit at the middle of the sides of the table but not necessarily in the same order. The persons
sit at the corner face towards the center of the table and the persons sit at the middle of the side face away
from the center of the table.
E sits second to the right of B. Two persons sit between B and H. F sits to the immediate right of H. The
number of persons sits between H and D is same as the number of persons sits between D and G. D sits at
the corner of the table. A sits adjacent to D.

Q74. Who among the following sits third to the left of C?


(a) A
(b) B
(c) D
(d) E
(e) H

Q75. How many persons sit between E and H when counts to the left of H?
(a) Two
(b) Three
(c) One
(d) None
(e) More than three

Q76. What is the position of C with respect to G?


(a) Second to the right
(b) Second to the left
(c) Immediate left
(d) Third to the right
(e) None of these

Q77. Which among the following statement(s) is/are not true?


(a) A sits to the immediate left of H
(b) Three persons sit between D and C
(c) E sits opposite to A
(d) B sits at the corner of the table
(e) All are true

142 adda247.com/teachers | www.sscadda.com | www.bankersadda.com | www.adda247.com


Q78. Four of the following five are alike in a certain way and thus form a group. Who among the following
does not belong to the group?
(a) E
(b) F
(c) D
(d) A
(e) B

Directions (79-82): Study the following number series carefully and answer the questions given below:
984637 294152591436918294 5761

Q79. How many prime numbers are in the above series each of which is immediately preceded by an even
number and immediately followed by odd number?
(a) Three
(b) Five
(c) Four
(d) Six
(e) More than six

Q80. What is the sum of the odd numbers which are immediately followed by the number which is multiple
of 3?
(a) 11
(b) 15
(c) 17
(d) 13
(e) 19

Q81. Which among the following number is 9th to the left of the number which is 7th from the right end?
(a) 2
(b) 1
(c) 9
(d) 5
(e) 8

Q82. How many even numbers are between the numbers which are 9 th from the right end and 3rd from the
left end?
(a) Six
(b) Five
(c) Four
(d) Three
(e) Two

143 adda247.com/teachers | www.sscadda.com | www.bankersadda.com | www.adda247.com


Q83. If we form a four-letter meaningful word with 1st, 5th, 6th and 8thletter from the left end of the word
‘’CRITERIA” (Using each letter once), then what would be the third letter of that meaningful word? If no
meaningful word is formed, then mark the answer as X. If more than one meaningful word is formed then
mark the answer as Z.
(a) C
(b) R
(c) A
(d) Z
(e) X

Directions (84-88): Study the following information carefully and answer the questions given below:

Seven persons have different designations i.e. Vice Chancellor (VC), Director (DR), Director of nursing (DN),
Head of Department (HOD), Principal, Professor and Assistant professor in a company. The order of
seniority is the same as given above i.e. Vice Chancellor (VC) is the senior-most designation and Assistant
professor is the junior-most designation.
At most two persons are senior to R. Two designations gap between R and V who is not a professor. One
designation gap between P and T and both of them are senior to V. There are as many persons junior to P
as senior to U. S is junior to Q.

Q84. How many persons are senior to Q?


(a) Three
(b) One
(c) Four
(d) Two
(e) None

Q85.Who among the following is designated as Principal?


(a) P
(b) Q
(c) T
(d) V
(e) S

Q86. The number of persons designated between R and U is same as the number of persons designated
before ____.
(a) S
(b) V
(c) Q
(d) T
(e) None of these

144 adda247.com/teachers | www.sscadda.com | www.bankersadda.com | www.adda247.com


Q87. Who among the following is three post junior to the one who is just senior to R?
(a) Q
(b) Director (DR)
(c) S
(d) Director of nursing (DN)
(e) Both Q and Director of nursing (DN)

Q88. If all the persons are arranged according to the alphabetical order from top to bottom then how many
persons remain unchanged except (P)?
(a) One
(b) Two
(c) None
(d) Three
(e) None of these

Directions (89-91): In each of the questions below, some statements are given followed by some
conclusions. You have to take the given statements to be true even if they seem to be at variance with
commonly known facts. Read all the conclusions and then decide which of the given conclusions logically
follow from the given statements, disregarding commonly known facts. Give answer
(a) If only conclusion I follows.
(b) If only conclusion II follows.
(c) If either conclusion I or II follows.
(d) If neither conclusion I nor II follows.
(e) If both conclusions I and II follow.

Q89. Statements: Some juice are mango.


Only a few mango are orange.
Conclusions: I. Some Juice are not mango
II. All Orange being mango is a possibility

Q90. Statements: No Bell is Clock.


All Bell is Ring.
Conclusions: I. Some Clock is Ring
II. No Ring is Clock

Q91. Statements: Only Car is Train.


Some Ship are Car
Conclusions: I. Some Ship are not Train
II. All Car being Ship is a possibility

145 adda247.com/teachers | www.sscadda.com | www.bankersadda.com | www.adda247.com


Direction (92-96): Study the following information carefully and answer the questions given below:

Seven persons H, J, K, L, M, N and O buy a product on different day one after another but not necessarily in
the same order.
One person buys product between H and O. Two persons buy product between O and N. N buys product
neither just before nor just after H. K buys product just before J and after O. L buys product two days before
M.

Q92. How many persons buy product between L and K?


(a) More than four
(b) Three
(c) None
(d) Two
(e) One

Q93. The number of persons buy product between J and N is same as the number of persons buy product
before ____.
(a) L
(b) M
(c) O
(d) H
(e) None of these

Q94. Which of the following statement is true?


I. More than two persons buy product after K
II. O buys product just after M
III. Only one person buys product before H
(a) Both I and III are true
(b) Both II and III are true
(c) Only I is true
(d) Only II is true
(e) None is true

Q95. How many persons buy product after N?


(a) One
(b) Two
(c) More than four
(d) Four
(e) None of these

146 adda247.com/teachers | www.sscadda.com | www.bankersadda.com | www.adda247.com


Q96.Four of the following five are alike in a certain way and hence form a group. Which of the following
does not belong to that group?
(a) M-L
(b) L-H
(c) O-M
(d) J-N
(e) K-O

Direction (97-99): Study the following information carefully and answer the questions given below:

L has two daughters. C is the unmarried sister-in-law of H. K is the father of G. K is the only sibling of C. P is
the spouse of L. W is the maternal uncle of H. L is the maternal grandmother of M who is sibling of G. S is
the sister-in-law of K. G is the only grandson of J who is not female.

Q97. How many married couple are live in the family?


(a) Four
(b) One
(c) Three
(d) Two
(e) None of these

Q98. How is L related to K?


(a) Sister
(b) Niece
(c) Mother-in-law
(d) Sister-in-law
(e) Mother

Q99. Who is the sister-in-law of H?


(a) C
(b) M
(c) L
(d) Either C or M
(e) None of these

Q100. In the word ‘PAGINATE’, how many pairs of the letters have the same number of letters between them
(both forward and backward direction) in the word as in the alphabetical series?
(a) Four
(b) Two
(c) One
(d) Three
(e) More than four

147 adda247.com/teachers | www.sscadda.com | www.bankersadda.com | www.adda247.com


Solutions

S1. Ans.(b)
Sol. On referring to the first paragraph of he given passage, we can conclude that only option (b) is true
here. Refer to the section, “In fact, before the emergence of agrarian societies, the early humans journeyed
along the roads taken by their prey — hunter-gatherer societies were mobile and the road was their life.
But with the rise of agrarianism, humans learnt to put down their roots and a life of journeys became
something to be frowned upon”

S2. Ans.(d)
Sol. Refer to the first paragraph to answer the given question, “The world was a different place when the
Zapps set out — there were no smartphones, the internet was in its infancy, a site of knowledge and hope
rather than disinformation and division, the twin towers towered over New York, and Russia was a
diminutive shell stripped of its Soviet grandeur.”

S3. Ans.(d)
Sol. The first few lines of the second paragraph of the passage can be used to deduce the answer. Refer,
“Latest estimates show that over one million Americans live permanently in recreational vehicles with no
permanent address. Many are retired, but many others work part of the year, just long enough to keep
them going for a few months or so.”

S4. Ans.(d)
Sol. On referring to the second paragraph, we can conclude that only option (a) and (c) are correct. Hence
the right answer choice is option (d). Refer, “But in most countries, people without a stationary home are
subjected to intense legal scrutiny. Law enforcement institutions are even prejudiced against specific
communities.”

S5. Ans.(b)
Sol. Only sentence (b) is false.
For (a), (b) and (c): Refer to the second paragraph, “For example, in the United States of America, there are
legislations that make it illegal to sit on the sidewalk, to camp or sleep anywhere in a downtown area or to
hang around in a public place without an apparent purpose. The intent seems to encourage sanitization of
space of some kinds of human presences. These laws are not unique to the US. Anti-vagrancy laws have
existed for hundreds of years — think of the persecution of gypsies across Europe. In the old United
Kingdom, vagrants could be put in jail, sold as slaves, or sometimes even killed for no other reason than
wandering. Colonial India, too, had witnessed the purge of nomadic communities, most notably under the
guise of the campaign against thuggees”
For sentence (d): Refer to the first paragraph, “In the year, 2000, Herman and Candelaria Zapp set out in a
car from Buenos Aires to travel the world”

148 adda247.com/teachers | www.sscadda.com | www.bankersadda.com | www.adda247.com


S6. Ans.(a)
Sol. The appropriate word for the given blank is “hummed”
Hummed: make a low, steady continuous sound like that of a bee
Iterated: perform or utter repeatedly.
Cherished: protect and care for (someone) lovingly
Longed: have a strong wish or desire.
Enunciated: say or pronounce clearly.

S7. Ans.(d)
Sol. infancy – the early stage in the development or growth of
something
service – the action of helping or doing work for someone
apprehension – anxiety or fear that something bad or unpleasant will happen
perseverance – persistence in doing something despite difficulty or delay in achieving success
maturity – the state, fact, or period of having reached the most advanced stage in a process
intelligence – the ability to acquire and apply knowledge and skills

S8. Ans.(b)
Sol. insularity – ignorance of or lack of interest in cultures, ideas, or peoples outside one's own experience
irritation – the state of feeling annoyed, impatient, or slightly angry
inflexibility – inability to be changed or adapted to particular circumstances
disbelief – inability or refusal to accept that something is true or real
freedom – the power or right to act, speak, or think as one wants
awareness – knowledge or perception of a situation or fact

S9. Ans.(b)
Sol. The given statement is mentioning filmmaking research that enquires into production practices,
techniques, modes and genres used in cinema, television and online. Therefore, the correct word for the
given blank is ‘enquires’.
(a) probe means a thorough investigation into a crime or other matter.
(b) enquire means an act of asking for information.
(c) inquest means a judicial inquiry to ascertain the facts relating to an incident.
(d) survey means look closely at or examine (someone or something).
(e) investigate means make inquiries as to the character, activities, or background of (someone).

S10. Ans.(d)
Sol. The given statement is mentioning that the films are the output of the research and are based on fiction,
documentary and hybrid forms. Therefore, the correct word for the given blank is ‘fiction’.
(a) friction means the action of one surface or object rubbing against another.
(b) segregation means the action or state of setting someone or something apart from others.
(c) accede means agree to a demand, request, or treaty.
(d) fiction means literature in the form of prose, especially novels, that describes imaginary events and
people.
(e) purport means appear to be or do something, especially falsely.

149 adda247.com/teachers | www.sscadda.com | www.bankersadda.com | www.adda247.com


S11. Ans.(c)
Sol. The given statement is mentioning that by embracing different approaches, filmmaking research
pushes at the boundaries of traditional filmmaking and traditional research methods. Therefore, the
correct word for the given blank is ‘approaches’.
(a) regulation means a rule or directive made and maintained by an authority.
(b) adversary means one's opponent in a contest, conflict, or dispute.
(c) approach means a way of dealing with a situation or problem.
(d) assistance means the action of helping someone by sharing work.
(e) interval means an intervening time.

S12. Ans.(e)
Sol. The given statement is mentioning that there are many splendid success stories that demonstrate the
potential of pushing at the boundaries of traditional filmmaking and traditional research methods by
adopting. Therefore, the correct word for the given blank is ‘spectacular’
(a) express means convey (a thought or feeling) in words or by gestures and conduct.
(b) threatening means having a hostile or deliberately frightening quality or manner.
(c) overlook means fail to notice.
(d) mock means making fun of someone or something in a cruel way; derisive.
(e) spectacular means beautiful in a dramatic and eye-catching way.

S13. Ans.(a)
Sol. The given statement is mentioning two disciplines in which research tends to indulge. Therefore the
correct word for the given blank is ‘reside’
(a) reside means be present or inherent in something.
(b) accommodate means provide lodging or sufficient space for.
(c) occupy means fill or preoccupy (the mind).
(d) fuller means a person whose occupation is to clean, shrink, and felt cloth by heat, pressure, and
moisture. cloth.
(e) lament means a passionate expression of grief or sorrow.

S14. Ans.(c)
Sol. The given statement is mentioning that the indulgence of research in two disciplines has made it
fragmented, sometimes difficult to identify and subject to different forms of support and engagements.
Therefore the correct word for the given blank is ‘engagement’
(a) ample means enough or more than enough; plentiful.
(b) wither means become dry and shrivelled.
(c) engagement means the action of engaging or being engaged.
(d) enrollment means the action of enrolling or being enrolled.
(e) alliance means a relationship based on similarity of interests, nature, or qualities.

S15. Ans.(b)
Sol. Only the sentences (i) and (iii) use the word “pore” in the correct context. ”pore” is defined as “to scan
or examine intently”.

150 adda247.com/teachers | www.sscadda.com | www.bankersadda.com | www.adda247.com


S16. Ans.(d)
Sol. Option (d) gives the only rearrangement required for making the sentence grammatically and
contextually correct.
“In order to purchase the property, they had to undergo a lengthy application process.”

S17. Ans.(a)
Sol. Option (a) gives the only rearrangement required for making the sentence grammatically and
contextually correct.
“Many conflicts and fronts constituted the battlefields of World War Two, encompassing much of the
globe.”

S18. Ans.(c)
Sol. Option (c) gives the only rearrangement required for making the sentence grammatically and
contextually correct.
“We received a package yesterday but did not notice the mail until early this morning.”

S19. Ans.(b)
Sol. Option (b) gives the only rearrangement required for making the sentence grammatically and
contextually correct.
“Perfectly harmless spiders are often killed because they resemble the venomous black widow”

S20. Ans.(c)
Sol. Option (c) gives the only rearrangement required for making the sentence grammatically and
contextually correct.
“Many of the company’s rules were rigid and uncompromising, leaving the workers to look for new jobs.”

S21. Ans.(a)
Sol. The correct spelling of the given misspelt word will be “lose”. Even though “loose” is a word, the context
requires “lose” and thus it becomes apparent that the word “loose”, here is just a misspelled form of “lose”.
lose – become unable to find (something or someone)

S22. Ans.(a)
Sol. The correct spelling of the given misspelt word will be “unwary”.
unwary – not cautious of possible dangers or problems

S23. Ans.(d)
Sol. The correct spelling of the given misspelt word will be “memorabilia”.
memorabilia – objects kept or collected because of their associations with memorable people or events

S24. Ans.(e)
Sol. All the given highlighted words are spelt correctly.

151 adda247.com/teachers | www.sscadda.com | www.bankersadda.com | www.adda247.com


S25. Ans.(c)
Sol. The correct spelling of the given misspelt word will be “fluctuated”.
fluctuate – rise and fall irregularly in number or amount

S26. Ans.(b)
Sol. The collocation of “in recognition” is most appropriate with “of”, as it sufficiently attributes recognition
to someone to their achievements. Thus, option (b) is the most appropriate answer.

S27. Ans.(d)
Sol. The given sentence is in present continuous tense hence verb ‘become’ should be in gerund form. ‘Self-
reliance’ is a noun while we need adjective for ‘India’ so we should replace it with ‘self-reliant’.

S28. Ans.(b)
Sol. Since the given sentence is in present perfect continuous tense, the subject ‘spike’ is singular and we
need the adjective ‘steady’ to modify the noun, the correct replacement will be, “has been a steady”.

S29. Ans.(e)
Sol. The given highlighted phrase is correct hence no improvement required.

S30. Ans.(b)
Sol. The highlighted phrase is incorrect because modal verb ‘would’ always takes base form of verb with
it. Therefore use of ‘erupt’ and ‘lead’ is correct and suitable replacement will be, ‘would erupt and lead’.

S31. Ans.(b)
Sol. Pattern of series –
50 + 52 = 75
75 + 102 = 175
175 + 152= 400
400 + 202 = 800
? = 800 + 252= 1425

S32. Ans.(e)
Sol. Pattern of series –
8 + 10 = 18
18 + 20 = 38
38 + 40 = 78
? = 78 + 80 = 158
158 + 160 = 318

S33. Ans.(b)
Sol. Pattern of series –
? = 41 + 5 = 46
46 + 7 = 53
53 + 11 = 64
64 + 13 = 77
77 + 17 = 94

152 adda247.com/teachers | www.sscadda.com | www.bankersadda.com | www.adda247.com


S34. Ans.(d)
Sol. Pattern of series –
? = 215 − (14 × 5) = 145
145 −(14 × 4) = 89
89 −(14 × 3) = 47
47 − (14 × 2) = 19
19 −(14 × 1) = 5

S35. Ans.(a)
Sol. Pattern of series –
1 + 23 = 9
9 + 33 = 36
36 + 43 = 100
100 + 53 = 225
? = 225 + 63 = 441

Solutions. (36-40):
Companies total number of boats manufactured total number of ships manufactured
A (175 – 125) = 50 125
B (225 – 75) = 150 75
C (150 – 50) = 100 50
D (200 – 150) = 50 150

S36. Ans.(c)
1
Sol. Total boats used by fishermen = 150 × 3 = 50
1
Required percentage = 50 × 125 × 100 = 40%

S37. Ans.(e)
140
Sol. Total number of boats and ships manufactured by A in 2021 = 175 × 100 = 245
Total number of boats manufactured by A in 2021 = 245 − 125 = 120

S38. Ans.(b)
(100+50)
Sol. Required ratio = =6:5
(75+50)

S39. Ans.(e)
Sol. Total cost of all boats manufactured by D = 50 × 4600 = 230000 Rs.

S40. Ans.(a)
Sol. Total boats and ships manufactured by A & B = (175 + 225) = 400
Total boats manufactured by A & D = (50 + 50) = 100
400−100
Required percentage = × 100 = 300%
100

153 adda247.com/teachers | www.sscadda.com | www.bankersadda.com | www.adda247.com


S41. Ans.(b)
Sol. Let present age of Amit, Den & Mohit be ‘a’, ‘d’ & ‘m’ respectively
ATQ -
a + d = 20 + 2 × 6 = 32 − − − −(𝑖)
a −𝑚 = 6 ----- (ii)
And, m + d =26 ------ (iii)
From (i), (ii) (iii) we get
32 − 𝑎 + 𝑎 − 6 = 26
26 = 26
We cannot determine the age of Den

S42. Ans.(d)
Sol. 𝜋r + 2r = 36
22
× 𝑟 + 2𝑟 = 36
7
r = 7 cm
147
Length of rectangle = = 21 cm
7

S43. Ans.(a)
Sol. Let six consecutive odd numbers = a, a+2, a+4, a+6, a+8, a+10
ATQ –
6a + 30 − (𝑎 + 5) = 150
5a + 25 = 150
a = 25
Required sum = 25 + 35 = 60

S44. Ans.(d)
3
Sol. Profit sharing ratio of P to Q = (X × 3 + 5 𝑋 × 9) ∶ (𝑋 + 100) × 12
14𝑋
∶ 4(𝑋 + 100)
5
14𝑋 210
=
5×4(𝑋+100) 530−210
16𝑋 − 15𝑋 = 1500
𝑋 = 1500

S45. Ans.(b)
Sol. ATQ –
18×3
P× = 0.5P + 100
100
0.54P −0.50𝑃 = 100
P = 2500
121
Required amount = (2500 + 3500) × 100 = 7260 𝑅𝑠.

154 adda247.com/teachers | www.sscadda.com | www.bankersadda.com | www.adda247.com


S46. Ans.(b)
Sol. Total capacity of tank = 12 unit (LCM of 4, 6 & 3)
12
Efficiency of pipe A = 4 = 3 units/hour
12
Efficiency of pipe B = = 2 units/hour
6
12
Efficiency of pipe (A + B + C) = = 4 units/hour
3
So, efficiency of pipe C = 4 −(3 + 2) = −1 unit/hour (pipe C is an
outlet pipe)
Required time = 12 hours

S47. Ans.(d)
Sol. Let marked price and selling price of the article is 35x and 21x
respectively
100
Cost price of the article = 21x × 140 = 15𝑥
ATQ –
35x −15𝑥 = 400
x = 20 Rs.
Cost price of article = 20 × 15 = 300 𝑅𝑠.

S48. Ans.(d)
Sol. Let milk and water in the vessel initially be 7x and 3x respectively
ATQ –
7𝑥+50 80
= 20
3𝑥
12x – 7x = 50
x = 10 liters
initial quantity of milk in the mixture = 70 liters

S49. Ans.(d)
5
Sol. Time taken by Ram and Shayam together = 40 × 8 = 25 days
Total work = 200 unit (LCM of 40 & 25)
200 200
Efficiency of Shayam = 25 − 40 = 3 units/day
200 2
Required days = = 66 3 days
3

S50. Ans.(b)
Sol. Let his monthly salary be Rs 100x
Amount spent on accommodation=Rs 60x
15
Amount spent on entertainment=Rs 40𝑥 × 100 = 𝑅𝑠 6𝑥
Saving=Rs 34x
ATQ
26𝑥 = 3510
𝑥 = 135
His monthly salary= Rs 13,500

155 adda247.com/teachers | www.sscadda.com | www.bankersadda.com | www.adda247.com


S51. Ans.(a)
Sol. Let length of train B = 2l meters
So, length of train A = l meters
And let speed of train A = 4s meter/sec
And speed of train B = 3s meters/sec
ATQ –
𝑙+100
= 4s -------- (i)
5
2𝑙+100
Also, = 3s -------- (ii)
10
From (i) & (ii) we get –
l = 100 meters
So, length of train B = 2l = 200 meters

S52. Ans.(b)
Sol. Let the speed of boat in upstream be ′𝑥’ km/hr and speed of boat in downstream be ‘4x’ km/hr.
And, distance be ‘D’ km
ATQ,
𝐷 𝐷
+ = 10
4𝑥 𝑥
5𝐷
= 10
4𝑥
𝐷 = 8𝑥
8𝑥
So, required time = 𝑥+4𝑥 = 3 ℎ𝑜𝑢𝑟𝑠 and 12 minutes
2

S53. Ans.(e)
64 75
Sol. 100 × 2500 + 100 × 1600 =?× 112
?× 112 = 1600 + 1200
2800
?= 112
? = 25

S54. Ans.(c)
55
Sol. 100 × 1400 + ?2 = 1728-282
?2 = 676
? = 26

S55. Ans.(d)
540 15
Sol. + 450 = 100 × 3200
?
540
= 480 − 450
?
? = 18

156 adda247.com/teachers | www.sscadda.com | www.bankersadda.com | www.adda247.com


S56. Ans.(b)
85
Sol. ?2 = 100 × 280 + 34 − 224 + 96
?2 = 144
? = 12

S57. Ans.(b)
Sol. 39 + 441 − 90 = 5 × ?
5 × ? = 390
? = 78

S58. Ans.(e)
Sol. ? × 7 = 125 + 436 − 337
224
?= 7
? = 32

S59. Ans.(c)
?
Sol. 6 = 196 + 48 − 144
? = 100 × 6
? = 600

S60. Ans.(b)
1
Sol. (?× 48) × = 32
54
? = 36

S61. Ans.(b)
1 2 3 3
Sol. ? = 2 + + 5 + − 3 − + 8 +
3 3 4 4
? = 13

S62. Ans.(b)
32×729
Sol. 81×16 + 7 = (? )2
(?)² = 18 + 7 = 25
?=5

S63. Ans.(c)
1 1
Sol. 9097 × 11 × 10 = ? +15.7
? = 82.7 – 15.7
? = 67

S64. Ans.(d)
Sol. 180 – 110 = −255 + ?
? = 70 + 255
? = 325

157 adda247.com/teachers | www.sscadda.com | www.bankersadda.com | www.adda247.com


S65. Ans.(d)
Sol. 175 + 133 + ? = 400
? = 92

S66. Ans.(c)
Sol. Same logic except option (c)

Solutions (67-70):

S67. Ans.(b)

S68. Ans.(d)

S69. Ans.(d)

S70. Ans.(c)

Solutions (71-73):

S71. Ans.(a)

S72. Ans.(c)

S73. Ans.(d)

Solutions (74-78):

158 adda247.com/teachers | www.sscadda.com | www.bankersadda.com | www.adda247.com


S74. Ans.(a)

S75. Ans.(e)

S76. Ans.(b)

S77. Ans.(d)

S78. Ans.(c)

Solutions (79-82):

S79. Ans.(c)

S80. Ans.(b)

S81. Ans.(d)

S82. Ans.(a)

S83. Ans.(d)

Solutions (84-88):

Designations Person
Vice Chancellor (VC) P
Director (DR) R
Director of nursing (DN) T
Head of Department Q
(HOD)
Principal V
Professor S
Assistant professor U

S84. Ans.(a)

S85. Ans.(d)

S86. Ans.(b)

S87. Ans.(a)

159 adda247.com/teachers | www.sscadda.com | www.bankersadda.com | www.adda247.com


S88. Ans.(c)

Solutions (89-91):

S89. Ans.(b)
Sol.

S90. Ans.(c)
Sol.

S91. Ans.(a)
Sol.

Solutions (92-96):
Persons
L
H
M
O
K
J
N

160 adda247.com/teachers | www.sscadda.com | www.bankersadda.com | www.adda247.com


S92. Ans.(b)

S93. Ans.(a)

S94. Ans.(b)

S95. Ans.(e)

S96. Ans.(a)

Solutions (97-99):

S97. Ans.(d)

S98. Ans.(c)

S99. Ans.(a)

S100. Ans.(c)
Sol.

161 adda247.com/teachers | www.sscadda.com | www.bankersadda.com | www.adda247.com


Part - 8

Directions (1-6): Each question is divided into four parts in which one part may or may not have error.
Choose the part that has the error as your answer. If all the parts are correct choose (E) i.e., No Error.

Q1. (A) Apart from contributing/(B) to air pollution, a large number/(C) of vehicles on the Indian roads/
(D) also contributes to noise pollution/(E) No Error.
(a) A
(b) B
(c) C
(d) D
(e) No Error

Q2. (A) Farmers who had stayed put at/(B) sites around the national capital/ (C) for a year are now
dispersing, /(D) but not with much clear on the road ahead/(E) No Error.
(a) A
(b) B
(c) C
(d) D
(e) No Error

Q3. (A) If I was a/(B) rich man, I /(C) would make more/(D) charitable donations/(E) No Error.
(a) A
(b) B
(c) C
(d) D
(e) No Error

Q4. (A) After a long /(B) adjournment, the jury have/(C) given the verdict/ (D) on Saturday/(E) No Error.
(a) A
(b) B
(c) C
(d) D
(e) No Error

Q5. (A) You had better/(B) to watch the/(C) way you talk to/ (D) me
in the future/(E) No Error.
(a) A
(b) B
(c) C
(d) D
(e) No Error

162 adda247.com/teachers | www.sscadda.com | www.bankersadda.com | www.adda247.com


Q6. (A) I reached at /(B) 10 am, but he/ (C) was latter than/(D) I expected/ (E) No Error.
(a) A
(b) B
(c) C
(d) D
(e) No Error

Directions (7-11): In the following questions five sentences are given. You have to rearrange these
sentences to make a contextually meaningful paragraph. Later, answer the follow-up questions.
(A) And because no light can get out, people can't see black holes.
(B) The gravity is so strong because matter has been squeezed into a tiny space.
(C) A black hole is a place in space where gravity pulls so much that even light can not get out.
(D) This compression happens when a star is dying.
(E) They are invisible and space telescopes with special tools can help find black holes.

Q7. Which of the following would be the correct order of sentences after rearrangement?
(a) DEBAC
(b) CBDAE
(c) ABCED
(d) BACED
(e) CEDAB

Q8. Which of the following would be the THIRD sentence after rearrangement?
(a) A
(b) B
(c) C
(d) D
(e) E

Q9. Which of the following would be the FIRST sentence after rearrangement?
(a) A
(b) B
(c) C
(d) D
(e) E

Q10. Which of the following would be the LAST sentence after rearrangement?
(a) A
(b) B
(c) E
(d) D
(e) C

163 adda247.com/teachers | www.sscadda.com | www.bankersadda.com | www.adda247.com


Q11. Which of the following would be the FOURTH sentence after rearrangement?
(a) A
(b) B
(c) C
(d) D
(e) E

Directions (12-19): Read the following passage and answer the given questions based on the information
provided in the passage.

New employee orientation programs have been shown to socialize newcomers and increase their
knowledge, skills, and abilities upon completion. These types of programs are perhaps the most influential
piece of an employee’s development. Although many organizations include orientation as part of their new
employee development tactics, few truly utilize its full scope. New employee orientation programs are
deliberate attempts by an organization to introduce a new employee to what working for the organization
will be like. Although orientations can teach some important skills and abilities, they also teach new
employees about the organization’s history, goals, values, people, politics, and language. Orientation
programs start off the socialization process and help the new employee fit into his or her new
surroundings. New employee orientation programs may sometimes feel like a stale period of high anxiety
and stress while training takes place when these potentially harmful feelings have most likely been
resolved. A great deal of time and money is spent every year socializing and technically training employees.
Making sure this investment is well spent should be a central concern. However, this is not always the case.
Because training is expensive, organizations should make sure they are effectively preparing employees to
run for on-the-job success by delivering the proper training content and teaching it the right way.
Assessing the training needs of new employees consumes time and money, but it sends the message that
the organization is committed to the development of its employees, to giving them the right tools to
perform and putting them in a position where they can succeed, all of which are recognized goals of new
employee orientation programs. Helping a new employee fit into his or her job and organization is
important when considering job satisfaction, commitment, and retention. Whether potential candidates
accept the job or not is somewhat influenced by how well they believe they will fit into that company. Thus,
developing an orientation program that will not only properly train new hires, but successfully introduce
them to their new organization is extremely important.
The attributes an employee needs to succeed on the job, including those desired upon entry into the job,
can be found through an analysis of the necessary knowledge, skills, and abilities required to perform the
essential functions of the job. Since technical skills training is meant to enhance those skills and abilities
needed for a job, it makes sense that training should have a close ___________(I) ___________ to whether or not
a new employee fits into his or her environment. A big difference between Japanese and western
organizations is that orientation programs are much more important to Japanese organizations because
they want to cement a lasting relationship with new employees. Thus, the Japanese approach to selection
and recruitment also works hand in hand. Japanese firms hire almost entirely at the entry-level. This is
done so that the new arrival can be properly trained and socialized into the organization to be a good fit.
The length of this process can vary from six months to three years, whereas in many western organizations
a new employee merely completes the required HR paperwork and is then suddenly expected to execute
the role for which he or she was hired.

164 adda247.com/teachers | www.sscadda.com | www.bankersadda.com | www.adda247.com


Q12. Why is it imperative to undertake the socialization process in orientation programs?
(a) To ensure that the expenditure incurred on orientation is well used.
(b) To assist the new employee in acclimating to his or her new environment.
(c) To give the appropriate training content in a timely manner.
(d) To improve the skills and talents required for the completion of the tasks.
(e) None of these

Q13. How does the author differentiate between new employee orientation programs and training?
(a) Orientation programs take place during a time of high anxiety and tension, while training takes place
in a tranquil environment.
(b) Orientation programs take place during a tranquil moment, whereas training takes place during a
period of high anxiety and tension.
(c) When potentially detrimental thoughts of orientation have been addressed, training can continue.
(d) Only (a) and (b)
(e) Both (a) and (c)

Q14. Which of the following statements with reference to the information provided in the given passage
is/are correct?
(i) The viewpoints of potential candidates have an impact on whether or not a position is accepted.
(ii) Time and money are invested in assessing the development needs of new employees.
(iii) Orientation programs offer new employees the necessary tools to enable them to succeed.
(a) Only (ii)
(b) Both (i) and (ii)
(c) All of these
(d) Only (iii)
(e) Both (ii) and (iii)

Q15. Why do most organizations invest in orientation programs, despite the fact that assessing the training
needs of new employees consumes time and money?
(a) It illustrates the organization's commitment to employee development.
(b) It provides employees with the necessary tools to help them succeed.
(c) To assist new employees in overcoming high levels of fear and tension.
(d) Only (a) and (b)
(e) Only (b) and (c)

Q16. Choose the most appropriate filler from the following words to fill in the blank (I), as highlighted in
the given passage.
(a) confiscation
(b) requisition
(c) distraint
(d) analogy
(e) amusement

165 adda247.com/teachers | www.sscadda.com | www.bankersadda.com | www.adda247.com


Q17. Which of the following statements with reference to the Japanese approach to selection and
recruitment is/are incorrect?
(a) The majority of Japanese companies hire primarily at entry-level positions.
(b) Japanese companies take longer to hire and train employees than Western companies.
(c) After the HR paperwork is done, western firms expect new employees to take up the function.
(d) Only (b) and (c)
(e) None of these

Q18. Choose the most appropriate antonym of the word ‘STALE’, as highlighted in the given passage.
(a) noxious
(b) virulent
(c) rancid
(d) chipper
(e) None of these

Q19. Choose the most appropriate synonym of the word ‘RUN’, as highlighted in the given passage.
(a) hasten
(b) execute
(c) cascade
(d) expedite
(e) None of these

Directions (20-25): In each sentence four words are given in bold. In which one word is either misspelt
or grammatically inappropriate, choose it as your answer. If all the words are correct choose option (E),
i.e., No Error.

Q20. Teachers, especially those working in high poverty(A) enviroments and with(B) marginalised
groups, face an inordinate amount of job stress, it is very important to(C) recognise and validate their
stress, bring it out and(D) discuss it openly.
(a) enviroments
(b) marginalised
(c) recognise
(d) discuss
(e) No Error

Q21. During the framing of the Indian(A) Constitution, it was proposed that any petition alleging a (B)
brech of (C) fundamental rights by the state ought to be(D) judicially decided within one month.
(a) Constitution
(b) brech
(c) Fundamental
(d) judicially
(e) No error

166 adda247.com/teachers | www.sscadda.com | www.bankersadda.com | www.adda247.com


Q22. We have a(A) stending invitation to(B) visit them anytime, but we(C) choose not to go(D) anymore.
(a) stending
(b) visit
(c) choose
(d) anymore

Q23. Where all parties(A) consent, the(B) court may make, vary or(C) revoke directions without the need
for(D) attandance at court.
(a) Consent
(b) court
(c) revoke
(d) attandance
(e) No Error

Q24. May be, I(A) delivered my(B) opinion more(C) bluntly than I should have, but I had always been(D)
candid with Ted.
(a) delivered
(b) opinion
(c) bluntly
(d) candid
(e) No Error

Q25. Choose the style that(A) suts your shape and style, and you'll look(B) effortlessly (C) beautiful
during the(D) warmer months.
(a) suts
(b) effortlessly
(c) beautiful
(d) warmer
(e) No Error

Directions (26-30): In the questions given below, a sentence has been broken down into four fragments
labelled (A), (B), (C) and (D) and arranged not necessarily in the correct order. You have to find the correct
order of arrangement from the options given below. If none of the options fail to form a meaningful
sentence, mark ‘None of these’ option as your answer.

Q26. (A) decade ago it seemed


(B) when the Right to Education Act
(C) like a breakthrough
(D) was promulgated over a
(a) BCAD
(b) CBDA
(c) BDAC
(d) ACDB
(e) None of these

167 adda247.com/teachers | www.sscadda.com | www.bankersadda.com | www.adda247.com


Q27. (A) demands even greater efficiency
(B) of agriculture technologies.
(C) thus it relies on the use
(D) agriculture is a sector that today
(a) BCAD
(b) ACBD
(c) CDBA
(d) DACB
(e) None of these

Q28. (A) monitoring of student


(B) beyond schoolwork and
(C) behavior often extends
(D) normal school hours.
(a) BCAD
(b) DBCA
(c) CBDA
(d) ACBD
(e) None of these

Q29. (A) of love and family


(B) the emotional dramas
(C) have explored in great detail
(D) researchers and philosophers
(a) DCBA
(b) DBCA
(c) CBDA
(d) ACDB
(e) None of these

Q30. (A) lack the means to satisfy


(B) poverty is said
(C) their basic needs
(D) to exist when people
(a) BCAD
(b) BDAC
(c) CBDA
(d) ACDB
(e) None of these

Q31. If the average of present age of A and B is 18 years and six years hence A age will be two times of age
of B that time. Then find the difference between present age of A & B?
यदि A और B की वर्तमान आयु का औसर् 18 वर्त र्था 6 वर्त बाि A की आयु, उस समय B की आयु की िो गुनी होगी, र्ो A और
B की वर्तमान आयु के बीच का अंर्र ज्ञार् कीजिए।
(a) 14 years\14 वर्त
(b) 16 years\16 वर्त
(c) 20 years \20 वर्त
(d) 8 years\8 वर्त
(e) 10 years\10 वर्त

168 adda247.com/teachers | www.sscadda.com | www.bankersadda.com | www.adda247.com


Q32. Varun invested Rs. X at the rate of 8 % per annum for five years and obtained total simple interest of
Rs. 5000. Had he invested the same amount at the same rate of interest for two years, then how much
amount would he have obtained as compound interest (compound annually) ?
वरुण पांच वर्ों के जिए प्रजर् वर्त 8% की िर से X रु. का जनवेश करर्ा है और 5000 रु. का कु ि साधारण ब्याि प्राप्त करर्ा है।
यदि उसने समान राजश को िो वर्ों के जिए समान ब्याि िर पर जनवेश दकया होर्ा, र्ो उसे चक्रवृजि ब्याि (वार्र्तक चक्रवृजि) के
रूप में दकर्नी राजश प्राप्त होर्ी?
(a) 2050 Rs. \2050 रु.
(b) 2010 Rs. \2010 रु.
(c) 2060 Rs. \2060 रु.
(d) 2080 Rs. \2080 रु.
(e) 2040 Rs. \2040 रु.

Q33. A man starts from point P to reach point Q, which is 20 kms apart. If speed of man is 5 kmph, then
find what percent should he increase his speed in order to shorten the journey time by 3/5 th?
एक व्यजि बबंिु P से शुरू होकर बबंिु Q र्क पहंचर्ा है, िो 20 दकमी की िूरी पर है। यदि व्यजि की गजर् 5 दकमी प्रजर् घंटा है, र्ो
यात्रा के समय को 3/5 कम करने के जिए उसे अपनी गजर् दकर्ने प्रजर्शर् की वृजि करनी चाजहए?
(a) 165%
(b) 140%
(c) 175%
(d) 125%
(e) 150%

Q34. Pipe A alone & pipe B alone can fill the same tank in 5 hours and 4 hours respectively. If the ratio of
efficiency of pipe A & B together to that of pipe C alone is 9:2, then find time taken by pipe C alone to fill the
same tank?
पाइप A अके िा और पाइप B अके िा समान टैंक को क्रमशः 5 घंटे और 4 घंटे में भर सकर्े है। यदि पाइप A और B की धाररर्ा का
अके िे पाइप C की धाररर्ा से अनुपार् 9:2 है, र्ो उसी टैंक को भरने के जिए अके िे पाइप C द्वारा जिया गया समय ज्ञार् कीजिए।
(a) 8 hours\8 घंटा
(b) 5 hours \5 घंटा
(c) 6 hours \6 घंटा
(d) 10 hours\10 घंटा
(e) 12 hours \12 घंटा

Q35. A is 40% more efficient than C and B is 50% more efficient than A. If A and C together can complete a
piece of work in 45 days, then find days taken by A, B & C together to complete 50% of the work?
A, C से 40% अजधक कु शि है और B, A से 50% अजधक कु शि है। यदि A और C जमिकर दकसी कायत को 45 दिनों में पूरा कर
सकर्े हैं, र्ो A, B और C द्वारा जमिकर कायत का 50% पूरा करने में िगने वािे दिन ज्ञार् कीजिए।
(a) 20 days\20 दिन
(b) 24 days\24 दिन
(c) 26 days \26 दिन
(d) 32 days\32 दिन
(e) 16 days\16 दिन

169 adda247.com/teachers | www.sscadda.com | www.bankersadda.com | www.adda247.com


Q36. If 45% of X is equal to 60% of Y and the average of both the numbers is 20 less than the X, then find
60% of Y?
यदि X का 45%, Y के 60% के बराबर है और िोनों संख्याओं का औसर् X से 20 कम है, र्ो Y का 60% ज्ञार् कीजिए?
(a) 108
(b) 64
(c) 72
(d) 96
(e) 80

Q37. P & Q invested total amount of Rs.6400 in business for 12 months and 9 months respectively. If P &
Q gets total profit of Rs.3600 and profit share of P is Rs.1600, then find investment of Q?
P और Q ने एक व्यवसाय में क्रमशः 12 महीने और 9 महीने के जिए कु ि 6400 रुपये की राजश का जनवेश दकया। यदि P और Q
को कु ि 3600 रुपये का िाभ जमिर्ा है और P का िाभ जहस्सा 1600 रुपये है, र्ो Q का जनवेश ज्ञार् कीजिए।
(a) 3000 Rs. \3000 रु.
(b) 2400 Rs. \2400 रु.
(c) 3600 Rs. \3600 रु.
(d) 4000 Rs. \4000 रु.
(e) 3200 Rs. \3200 रु.

Q38. A vessel contains 64 liters mixture of milk & water in the ratio of 9 :7 respectively. If ‘x’ liters of milk
is added in the vessel, then the new quantity of milk becomes 75% more than water in vessel, then find ‘x’?
एक बर्तन में िूध और पानी का 64 िीटर जमश्रण क्रमशः 9:7 के अनुपार् में है। यदि बर्तन में 'x' िीटर िूध जमिाया िार्ा है, र्ो
िूध की नई मात्रा बर्तन में पानी से 75% अजधक हो िार्ी है, र्ो 'x' ज्ञार् कीजिए।
(a) 7
(b) 9
(c) 10
(d) 13
(e) 11

Q39. Shopkeeper sold an article after allowing two successive discounts of 15% and 20% on it. If
shopkeeper earned 2% profit on selling the article and total discount allowed by him is Rs. 192, then find
cost price of the article?
िुकानिार ने एक वस्र्ु पर 15% और 20% की िो क्रमागर् छू ट िेने के बाि उसे बेचा। यदि िुकानिार वस्र्ु को बेचने पर 2% का
िाभ अर्ितर् करर्ा है और उसके द्वारा अनुमर् कु ि छू ट 192रु., र्ो वस्र्ु का क्रय मूल्य ज्ञार् कीजिए।
(a) Rs.500\500 रु.
(b) Rs.400\400 रु.
(c) Rs.300\300 रु.
(d) Rs.600\600 रु.
(e) Rs.250\250 रु.

170 adda247.com/teachers | www.sscadda.com | www.bankersadda.com | www.adda247.com


Q40. Average weight of a group is 36 kg. If a student having weight of 48 kg left the group, then the average
weight of a group is decreased by one kg., then find the number of people in the group initially?
एक समूह का औसर् भार 36 दकग्रा है। यदि 48 दकग्रा भार वािा एक जवद्याथी समूह छोड़ िेर्ा है, र्ो इस समूह के औसर् भार में
एक दकग्रा की कमी हो िार्ी है, र्ो आरम्भ में समूह में व्यजियों की संख्या ज्ञार् कीजिए।
(a) 10
(b) 13
(c) 15
(d) 16
(e) 20

Q41. Speed of train B is 20% more than speed of train A and length of train A and B is 720 meters and 600
meters respectively. If both trains running in opposite direction cross each other in 24 second, then find
speed of trains B (in km/hr) ?
ट्रेन B की गजर्, ट्रेन A की गजर् से 20% अजधक है र्था ट्रेन A और B की िंबाई क्रमशः 720 मीटर और 600 मीटर है। यदि जवपरीर्
दिशा में चिने वािी िोनों ट्रेनें 24 सेकंड में एक िूसरे को पार करर्ी हैं , र्ो ट्रेन B की गजर् (दकमी/घंटा में) ज्ञार् कीजिए।
(a) 160
(b) 84
(c) 144
(d) 108
(e) 90

Q42. If perimeter of a rectangle is 4.5 times of its breadth and length of the rectangle is 25 meters, then
find the area of rectangle (in meter square) ?
यदि एक आयर् का पररमाप उसकी चौड़ाई का 4.5 गुना है और आयर् की िंबाई 25 मीटर है , र्ो आयर् का क्षेत्रफि (मीटर वगत
में) ज्ञार् कीजिए।
(a) 480
(b) 400
(c) 500
(d) 750
(e) 600

Q43. The ratio of income & savings of a man is in ratio of 16:3 respectively. If his savings increases by 1/3
rd and expenditure increased by 1/2, then find the ratio of new income to earlier income of man?
एक व्यजि की आय और बचर् का अनुपार् क्रमशः 16:3 है। यदि उसकी बचर् में 1/3 की वृजि होर्ी है और व्यय में 1/2 की वृजि
होर्ी है, र्ो व्यजि की नई आय का जपछिी आय से अनुपार् ज्ञार् कीजिये।
(a) 39:32
(b) 23:16
(c) 47:32
(d) 32:19
(e) 47 : 36

171 adda247.com/teachers | www.sscadda.com | www.bankersadda.com | www.adda247.com


Q44. A man invested Rs.25000 on simple interest at R% p.a. If interest received by man is three-twentieth
of the principal after three years, then find ‘R’?
एक व्यजि ने R% वार्र्तक की िर से साधारण ब्याि पर 25000 रु.का जनवेश दकया। यदि र्ीन वर्त बाि व्यजि द्वारा प्राप्त ब्याि
मूिधन का 3/20 जहस्सा है, र्ो 'R' ज्ञार् कीजिए।
(a) 5%
(b) 4%
(c) 6%
(d) 3%
(e) 8%

Q45. Speed of a boat in still water is 6 km/hr and speed of the current is 2 km/hr. If boat takes four hours
more in upstream than in the downstream to cover ‘D’ km, then find time take by boat to cover ‘D’ km in
still water?
शांर् िि में एक नाव की गजर् 6 दकमी/घंटा है और धारा की गजर् 2 दकमी/घंटा है। यदि नाव 'D' दकमी की िूरी र्य करने में धारा
के अनुकूि की र्ुिना में धारा के प्रजर्कू ि चार घंटे अजधक िेर्ी है, र्ो शांर् िि में 'D' दकमी की िूरी र्य करने में नाव द्वारा जिया
गया समय ज्ञार् कीजिए।
(a) 5 hours 30 minutes\5 घंटे 30 जमनट
(b) 5 hours 45 minutes\5 घंटे 45 जमनट
(c) 5 hours 10 minutes\5 घंटे 10 जमनट
(d) 5 hours 20 minutes\5 घंटे 20 जमनट
(e) 5 hours 40 minutes\5 घंटे 40 जमनट

Directions (46-50): Find the wrong number in following number series.


जनम्नजिजिर् संख्या श्रंििा में गिर् संख्या ज्ञार् कीजिए।

Q46. 60, 61, 86, 167, 336, 627, 1066


(a) 336
(b) 86
(c) 627
(d) 1066
(e) 61

Q47. 1, 1.5, 3.5, 11, 44.5, 224, 1338.5


(a) 1.5
(b) 1338.5
(c) 44.5
(d) 224
(e) 11

Q48. 4, 8, 24, 96, 485, 2880, 20160


(a) 4
(b) 96
(c) 8
(d) 20160
(e) 485

172 adda247.com/teachers | www.sscadda.com | www.bankersadda.com | www.adda247.com


Q49. 56, 76, 99, 125, 154, 186, 223
(a) 56
(b) 76
(c) 99
(d) 154
(e) 223

Q50. 15, 90, 30, 180, 60, 480, 120


(a) 480
(b) 30
(c) 90
(d) 120
(e) 180

Directions (51-60): What will come in the place of question (?) mark in following questions.
जनम्नजिजिर् प्रश्नों में प्रश्न जचह्न (?) के स्थान पर क्या आएगा।

Q51. 36% of 250 + 26 ÷2× ? = 207


(a) 8
(b) 5
(c) 6
(d) 9
(e) 7

Q52.
(a) 72
(b) 84
(c) 90
(d) 96
(e) 66

Q53.
(a) 1534
(b) 1554
(c) 1444
(d) 1544
(e) 1564

Q54.
(a) 1062
(b) 1064
(c) 1072
(d) 1096
(e) 1082

173 adda247.com/teachers | www.sscadda.com | www.bankersadda.com | www.adda247.com


Q55.
(a) 11
(b) 9
(c) 12
(d) 8
(e) 14

Q56.
(a) 120
(b) 140
(c) 150
(d) 160
(e) 210

Q57.
(a) 22
(b) 24
(c) 23
(d) 27
(e) 25

Q58. 756 + 432 -361+?= 990


(a) 163
(b) 153
(c) 167
(d) 143
(e) 147

Q59.
(a) 5
(b) 4
(c) 3
(d) 2
(e) 1

Q60.
(a) 9
(b) 8
(c) 7
(d) 6
(e) 11

174 adda247.com/teachers | www.sscadda.com | www.bankersadda.com | www.adda247.com


Directions (61-65): Table given below shows number of functions organized in three halls (A, B & C) in
four different months. Read the data carefully and answer the questions.
नीचे िी गई र्ाजिका चार अिग-अिग महीनों में र्ीन हॉि ((A, B और C) में आयोजिर् समारोहों की संख्या िशातर्ी है। डेटा को
ध्यान से पढें और प्रश्नों के उत्तर िें।

Q61. Total number of functions organized in all three halls in month of June are what percent (approx.)
less than total functions organized in Hall B & C together in month of May?
िून के महीने में र्ीनों हॉिों में आयोजिर् समारोहों की कु ि संख्या, मई के महीने में हॉि B और C में आयोजिर् कु ि समारोहों से
दकर्ने प्रजर्शर् (िगभग) कम है?
(a) 6%
(b) 4%
(c) 2%
(d) 7%
(e) 8%

Q62. Find difference between total number of functions organized by hall A & B in July and average number
of functions organized by A & B in April?
िुिाई में हॉि A और B द्वारा आयोजिर् समारोहों की कु ि संख्या और अप्रैि में A और B द्वारा आयोजिर् समारोहों की औसर्
संख्या के बीच अंर्र ज्ञार् कीजिए।
(a) 24
(b) 12
(c) 18
(d) 20
(e) 16

Q63. In month of August total functions organized by hall A is 20% more than that of functions organized
by same hall in May & total functions organized by hall C in month of August is 40% more than that of in
previous month, then find total functions organized by hall A & C together in month of August?
अगस्र् के महीने में हॉि A द्वारा आयोजिर् कु ि समारोह, मई में उसी हॉि द्वारा आयोजिर् समारोहों की र्ुिना में 20% अजधक
है र्था अगस्र् के महीने में हॉि C द्वारा आयोजिर् कु ि समारोह, जपछिे महीने की र्ुिना में 40% अजधक है, र्ो अगस्र् के महीने
में हॉि A और C द्वारा एक साथ आयोजिर् कु ि समारोह की संख्या ज्ञार् कीजिए।
(a) 196
(b) 188
(c) 186
(d) 184
(e) 178

175 adda247.com/teachers | www.sscadda.com | www.bankersadda.com | www.adda247.com


Q64. Find the ratio of total functions organized by hall A & C together in June to total functions organized
by hall B & C in April?
िून में हॉि A और C द्वारा आयोजिर् कु ि समारोहों का अप्रैि में हॉि B और C द्वारा आयोजिर् कु ि समारोहों से अनुपार् ज्ञार्
कीजिए।
(a) 25 : 47
(b) 25 : 43
(c) 49 : 25
(d) 25 : 48
(e) 25 : 49

Q65. Total functions organized by hall A and B together in May are what percent more than total functions
organized by hall C in July?
मई में हॉि A और B द्वारा आयोजिर् कु ि समारोह, िुिाई में हॉि C द्वारा आयोजिर् कु ि समारोहों से दकर्ने प्रजर्शर् अजधक हैं?
(a) 55%
(b) 40%
(c) 45%
(d) 35%
(e) 30%

Directions (66-69): Study the following information carefully and answer the questions given below.
जनम्नजिजिर् िानकारी का ध्यानपूवतक अध्ययन कीजिये और नीचे दिए गए प्रश्नों के उत्तर िीजिये।

Seven persons sit around a circular table facing the center. F sits 4th to the left of K. Two persons sit between
G and K. B sits immediate left of D. B and G are not an immediate neighbor. L does not sit immediate right
of S.
सार् व्यजि एक वृर्ाकार मेि के चारो ओर कें द्र की ओर मुि करके बैठे हैं। F, K के बायें से चौथे स्थान पर बैठा है। G और K के बीच
िो व्यजि बैठे हैं। B, D के ठीक बायें बैठा है। B और G जनकटर्म पड़ोसी नहीं हैं। L, S ठीक िायें नहीं बैठा है।

Q66. Who sits 3rd to the right of S?


S के िायें से र्ीसरे स्थान पर कौन बैठा है?
(a) K
(b) B
(c) D
(d) G
(e) None of these \इनमें से कोई नहीं

Q67. How many persons sit between F and L when counted to the left of F?
F के बायें से गणना करने पर F और L के मध्य दकर्ने व्यजि बैठे हैं?
(a) One \एक
(b) Four \चार
(c) Two \िो
(d) Three \र्ीन
(e) Either (a) or (b) \या र्ो (a) या (b)

176 adda247.com/teachers | www.sscadda.com | www.bankersadda.com | www.adda247.com


Q68. Who sits immediate right of G?
G के ठीक िायें कौन बैठा है?
(a) S
(b) L
(c) K
(d) F
(e) None of these\इनमें से कोई नहीं

Q69. The number of persons sit between L and F, when counted to the left of L is same as the number of
persons sit between S and ___, when counted to the left of __?
िब L के बायें से गणना की िार्ी है, र्ो L और F के बीच में बैठे व्यजियों की संख्या, ________ के बायें से गणना दकये िाने पर S और
____ के बीच बैठने वािे व्यजियों की संख्या के समान है
(a) B
(b) D
(c) F
(d) G
(e) None of these \इनमें से कोई नहीं

Q70. In the word ‘OBLIQUE’, how many pairs of the letters have the same number of letters between them
in both forward and backward direction in the word as in alphabetical series?
शब्ि 'OBLIQUE' में, अक्षरों के ऐसे दकर्ने युग्म हैं जिनके बीच अक्षरों की संख्या शब्ि में आगे और पीछे िोनों दिशाओं में समान है,
िैसे दक वणतमािा श्रंििा में होर्े है?
(a) None \कोई नहीं
(b) Two \िो
(c) One \एक
(d) Four \चार
(e) Three \र्ीन

Directions (71-73): In each of the questions below are given some statements followed by two
conclusions. You have to take the given statements to be true even if they seem to be at variance with
commonly known facts. Read all the conclusions and then decide which of the given conclusions logically
follows from the given statements disregarding commonly known facts.
नीचे दिए गए प्रत्येक प्रश्न में कु छ कथन और उसके बाि िो जनष्कर्त दिए गए हैं। आपको दिए गए कथनों को सत्य मानना है, भिे ही वे
सवतज्ञार् र्थ्यों से जभन्न प्रर्ीर् होर्े हों। सभी जनष्कर्ों को पदढए और दफर र्य कीजिये दक दिए गए जनष्कर्ों में से कौन सा जनष्कर्त
सामान्य रूप से ज्ञार् र्थ्यों की अवहेिना दकए जबना दिए गए कथनों का र्ार्कत क रूप से अनुसरण करर्ा है।

Q71. Statements:
All Orange is Papaya.
All Papaya are Mango.
कथन:
सभी संर्रे पपीर्ें हैं।
सभी पपीर्ें आम हैं।
177 adda247.com/teachers | www.sscadda.com | www.bankersadda.com | www.adda247.com
Conclusion
जनष्कर्त
I: All Mango are Papaya.
I: सभी आम पपीर्ें हैं।
II: Some Papaya is Orange.
II: कु छ पपीर्ें संर्रे हैं।
(a) If only conclusion I follows. \यदि के वि जनष्कर्त I अनुसरण करर्ा है।
(b) If only conclusion II follows. \यदि के वि जनष्कर्त II अनुसरण करर्ा है।
(c) If either conclusion I or II follows. \यदि या र्ो जनष्कर्त I या II अनुसरण करर्ा है।
(d) If neither conclusion I nor II follows. \यदि न र्ो जनष्कर्त I और न ही II अनुसरण करर्ा है।
(e) If both conclusions I and II follow. \यदि जनष्कर्त I और II िोनों अनुसरण करर्े हैं।

Q72. Statements:
All Driver are Road. No Road is Truck.
कथन: सभी ड्राइवर रोड हैं। कोई रोड ट्रक नहीं है।
Conclusion
जनष्कर्त
I: All Truck are Road.
I: सभी ट्रक रोड हैं।
II: Some Truck being Driver is a probability.
II: कु छ ट्रक के ड्राइवर होने की संभावना है।
(a) If only conclusion I follows. \यदि के वि जनष्कर्त I अनुसरण करर्ा है।
(b) If only conclusion II follows. \यदि के वि जनष्कर्त II अनुसरण करर्ा है।
(c) If either conclusion I or II follows. \यदि या र्ो जनष्कर्त I या II अनुसरण करर्ा है।
(d) If neither conclusion I nor II follows. \यदि न र्ो जनष्कर्त I और न ही II अनुसरण करर्ा है।
(e) If both conclusions I and II follow. \यदि जनष्कर्त I और II िोनों अनुसरण करर्े हैं।

Q73. Statements:
Only a few Rabbit are Duck.
Only a few Duck are Goat.
कथन:
के वि कु छ िरगोश बत्ति हैं।
के वि कु छ बत्ति बकरी हैं।
Conclusion
जनष्कर्त
I: Some Goat is Rabbit.
I: कु छ बकरी िरगोश है।
II: All Rabbit being Duck is a possibility.
II: सभी िरगोश के बत्ति होने की संभावना है।
(a) If only conclusion I follows. \यदि के वि जनष्कर्त I अनुसरण करर्ा है।
(b) If only conclusion II follows. \यदि के वि जनष्कर्त II अनुसरण करर्ा है।
(c) If either conclusion I or II follows. \यदि या र्ो जनष्कर्त I या II अनुसरण करर्ा है।
(d) If neither conclusion I nor II follows. \यदि न र्ो जनष्कर्त I और न ही II अनुसरण करर्ा है।
(e) If both conclusions I and II follow. \यदि जनष्कर्त I और II िोनों अनुसरण करर्े हैं।

178 adda247.com/teachers | www.sscadda.com | www.bankersadda.com | www.adda247.com


Directions (74-76): Study the information carefully and answer the questions given below.
िी गई िानकारी का ध्यानपूवतक अध्ययन कीजिये और नीचे दिए गए प्रश्नों के उत्तर िीजिये।

A certain number of persons are sitting in a row facing north. M sits fourth to the left of O and only two
persons sit left of M. Only two persons sit between M and Q. R sits second to the left of Q. S sits fourth to
the right of O. Not more than five persons sit between P and Q. P sits to the right of S. Eight persons sit
between P and G, who sits at one of the extreme ends.
एक जनजिर् संख्या में व्यजि उत्तर की ओर मुि करके एक पंजि में बैठे हैं। M, O के बायें से चौथे स्थान पर बैठा है और M के बायें के वि
िो व्यजि बैठे हैं। M और Q के मध्य के वि िो व्यजि बैठे हैं। R, Q के बायें से िूसरे स्थान पर बैठा है। S, O के िायें से चौथे स्थान पर बैठा
है। P और Q के बीच में पााँच से अजधक व्यजि नहीं बैठे हैं। P, S के िायें बैठा है। P और G के बीच आठ व्यजि बैठे हैं, िो दकसी एक
अंजर्म छोर पर बैठा है।

Q74. How many persons sit in the row?


पंजि में दकर्ने व्यजि बैठे हैं?
(a) 18
(b) 21
(c) 22
(d) 23
(e) 17

Q75. How many persons are sitting between O and Q?


O और Q के मध्य दकर्ने व्यजि बैठे हैं?
(a) One \एक
(b) Two \िो
(c) None \कोई नहीं
(d) Three \र्ीन
(e) More than three \र्ीन से अजधक

Q76. Who sits 5th to the left of P?


P के बायें से पांचवें स्थान पर कौन बैठा है?
(a) O
(b) R
(c) M
(d) S
(e) More than thirteen \र्ेरह से अजधक

Q77. In a certain code CIRCLE is written as DHSBMD in the same way


how is PUBLIC written in that code?
एक जनजिर् कू ट में CIRCLE को DHSBMD जििा िार्ा है। समान कू ट में PUBLIC
को दकस प्रकार से जििा िाएगा?
(a) QTBKJB
(b) QTCKJB
(c) CQTBKJ
(d) TQCLBK
(e) None of these
इनमें से कोई नहीं

179 adda247.com/teachers | www.sscadda.com | www.bankersadda.com | www.adda247.com


Directions (78-81): Study the following information and answer the questions below:
जनम्नजिजिर् िानकारी का अध्ययन कीजिये और नीचे दिए गए प्रश्नों के उत्तर िीजिये:

Seven persons were born in seven different months, i.e January, March, April, May, June, July and August,
of the same year and on same date. Q was born in the month which has even number of days. Two persons
were born between S and Q. U is just older to R but not born in the month which has even number of days.
More than two persons born between R and V who is younger to S. P is older than T who was not born in
June.
सार् व्यजियों का िन्म एक ही वर्त के सार् अिग-अिग माह- िनवरी, माचत, अप्रैि, मई, िून, िुिाई और अगस्र् में और समान जर्जथ
को हआ था। Q का िन्म उस माह में हआ था जिसमें दिनों की संख्या सम होर्ी है। S और Q के बीच िो व्यजियों का िन्म हआ था। U
ठीक R से बड़ा है, िेदकन उस ठीक में िन्म नहीं िेर्ा है, जिसमें दिनों की संख्या सम है। R और V, िो S से छोटा है, के बीच िो से
अजधक व्यजि िन्म िेर्े हैं। P, T से बड़ा है, जिसका िन्म िून में नहीं होर्ा है।

Q78. Who among the following is the oldest person?


जनम्नजिजिर् में से सबसे अजधक आयु का व्यजि कौन है?
(a) S
(b) P
(c) V
(d) R
(e) None of these \इनमें से कोई नहीं

Q79. How many persons are younger than T?


T से दकर्ने व्यजि छोटे हैं?
(a) Four \चार
(b) One \एक
(c) Three \र्ीन
(d) Two \िो
(e) None \कोई नहीं

Q80. Who among the following was born in April?


जनम्नजिजिर् में से दकसका िन्म अप्रैि में हआ था?
(a) P
(b) T
(c) U
(d) V
(e) None of these \इनमें से कोई नहीं

Q81. Which of the following statement is not true about V? जनम्नजिजिर् में से कौन सा कथन V के सन्िभत में सत्य नहीं
है?
(a) Q is older than S\Q, S से बड़ा है
(b) V was born in April \V अप्रैि में िन्म िेर्ा है
(c) Four persons were born after V\चार व्यजियों का िन्म V के बाि होर्ा है
(d) T is just younger than V\T ठीक V से छोटा है
(e) Both (a) and (d) \(a) और (d) िोनों

180 adda247.com/teachers | www.sscadda.com | www.bankersadda.com | www.adda247.com


Directions (82-86): Answer these questions referring to the letter sequence given below:
नीचे दिए गए अक्षरों के क्रम द्वारा जनम्नजिजिर् प्रश्नों के उत्तर िीजिये:
QRDSTWCKGUVEJZHIXYANOFMPBL

Q82. If the letters of the above given series are written in reverse order, which letter will be the fourth to
the left of eleventh letter from the right end?
यदि ऊपर िी गई श्रृंििा के अक्षरों को उल्टे क्रम में जििा िार्ा है, र्ो िायें छोर से ग्यारहवें अक्षर के बायें से चौथा अक्षर कौन सा
होगा?
(a) O
(b) U
(c) V
(d) N
(e) None of these \इनमें से कोई नहीं

Q83. What will come in place of question mark (?) in the following series?
जनम्नजिजिर् श्रृंििा में प्रश्न जचन्ह (?) के स्थान पर क्या आएगा?
RSD WKC VJE ?
(a) XYA
(b) XAY
(c) IYX
(d) IXY
(e) None of these \इनमें से कोई नहीं

Q84. Which of the following is 6th to the right of the 13th letter from the left end?
जनम्नजिजिर् में से कौन बायें छोर से 13वें अक्षर के िायें से छठा अक्षर है?
(a) Y
(b) A
(c) N
(d) O
(e) None of these \इनमें से कोई नहीं

Q85. How many consonants are there which are immediately preceded by a consonant and immediately
followed by a vowel?
ऐसे दकर्ने व्यंिन हैं जिनके ठीक पहिे एक व्यंिन है और ठीक बाि में एक स्वर है?
(a) Three \र्ीन
(b) Two \िो
(c) One \एक
(d) Four\चार
(e) None \कोई नहीं

181 adda247.com/teachers | www.sscadda.com | www.bankersadda.com | www.adda247.com


Q86. If all vowels are eliminated, then which of the following 10 th to the left of the element which is 8 th
from the right end?
यदि सभी स्वरों को हटा दिया िाए, र्ो जनम्नजिजिर् में से कौन सा र्त्व िायें छोर से आठवें र्त्व के बायें से िसवां है?
(a) T
(b) W
(c) S
(d) C
(e) None of these\इनमें से कोई नहीं

Directions (87-91): Study the information given and answer the questions below:
िी गई िानकारी का अध्ययन कीजिये और नीचे दिए गए प्रश्नों के उत्तर िीजिये:

Eight persons live on different floors of a building where ground floor is numbered as 1 and the topmost
floor is numbered as 4. Each floor has two flats i.e., flat P and flat Q. Flat P is in the west of Flat Q. Flat P of
floor 2 is just above the flat P of floor 1 and just below the flat P of floor 3. Similarly, flat Q of floor 2 is just
above the flat Q of floor 1 and just below the flat Q of floor 3.
Two floors gap between A and C and both live in different flats. A lives above B who lives south-west of D.
A and D does not live on the same floor. H live on an even numbered floor above B but in different flat. F
lives west of D. One floor gap between H and E. E lives north-east of G who lives on an odd numbered floor.
आठ व्यजि एक इमारर् की जवजभन्न मंजििों पर रहर्े हैं िहां भूर्ि की संख्या 1 है और सबसे ऊपरी मंजिि की संख्या 4 है। प्रत्येक
मंजिि में िो फ्िैट- फ्िैट P और फ्िैट Q हैं। फ्िैट P फ्िैट Q के पजिम में है। मंजिि 2 का फ्िैट P, मंजिि 1 के फ्िैट P के ठीक ऊपर
और मंजिि 3 के फ्िैट P के ठीक नीचे है। इसी प्रकार, मंजिि 2 का फ्िैट Q, मंजिि 1 के फ्िैट Q के ठीक ऊपर और मंजिि 3 के फ्िैट
Q के ठीक नीचे है।
A और C के बीच िो मंजििों का अंर्र है और िोनों अिग-अिग फ्िैटों में रहर्े हैं। A, B के ऊपर रहर्ा है िो D के िजक्षण-पजिम में
रहर्ा है। A और D समान मंजिि पर नहीं रहर्े हैं। H, B के ऊपर एक सम संख्या वािी मंजिि पर रहर्ा है िेदकन अिग-अिग फ्िैट
में रहर्ा है। F, D के पजिम में रहर्ा है। H और E के बीच एक मंजिि का अंर्र है। E, G के उत्तर-पूवत में रहर्ा है िो एक जवर्म संख्या
वािी मंजिि पर रहर्ा है।

Q87. Who among the following lives in Flat Q of 4th floor?


जनम्नजिजिर् में से कौन चौथी मंजिि के फ्िैट Q में रहर्ा है?
(a) E
(b) D
(c) A
(d) H
(e) None of these \इनमें से कोई नहीं

Q88. Who among the following lives in the west of C?


जनम्नजिजिर् में से कौन C के पजिम में रहर्ा है?
(a) B
(b) G
(c) E
(d) F
(e) None of these \इनमें से कोई नहीं

182 adda247.com/teachers | www.sscadda.com | www.bankersadda.com | www.adda247.com


Q89. Who among the following lives just below D’s Flat?
जनम्नजिजिर् में से कौन D के फ्िैट के ठीक नीचे रहर्ा है?
(a) B
(b) C
(c) E
(d) H
(e) None of these \इनमें से कोई नहीं

Q90. Four among the following five are alike in a certain way and related to a group, who among the
following does not belong to the group?
जनम्नजिजिर् पांच में से चार एक जनजिर् र्रीके से समान हैं और एक समूह से संबंजधर् हैं, जनम्नजिजिर् में से कौन समूह से संबंजधर्
नहीं है?
(a) E
(b) B
(c) F
(d) G
(e) A

Q91. Which of the following statement is true about F?


जनम्नजिजिर् में से कौन सा कथन F के सन्िभत में सत्य है?
(a) F lives on an odd numbered floor \F एक जवर्म संख्या वािी मंजिि पर रहर्ा है
(b) H lives above F \H, F के ऊपर रहर्ा है
(c) C lives south-east of F \C, F के िजक्षण-पूवत में रहर्ा है
(d) F and G live in the same flat. \F और G समान फ्िैट में रहर्े हैं।
(e) All are true \सभी सत्य हैं

Directions (92-94): Read the following information carefully and answer the given questions:
जनम्नजिजिर् िानकारी को ध्यानपूवतक पदढए और दिए गए प्रश्नों के उत्तर िीजिये:

Arun starts walking from point D in north direction after walking 12km he reached at point G then he takes
a right turn and covers 8km after that he takes a left turn and covers 6km to reach at point F. Finally, he
takes a left turn and walks 10km to reach at point V.
अरुण बबंिु D से उत्तर दिशा में चिना शुरू करर्ा है, 12 दकमी चिने के बाि वह बबंिु G पर पहाँचर्ा है, दफर वह िायें मुड़र्ा है और 8
दकमी चिर्ा है उसके बाि वह बायें मुड़र्ा है और बबंिु F पर पहाँचने के जिए 6 दकमी की िूरी र्य करर्ा है। अंर् में, वह बायें मुड़र्ा है
और बबंिु V पर पहाँचने के जिए 10 दकमी चिर्ा है।

Q92. If point L is 18km south of point V then what is the shortest distance between point D and point L?
यदि बबंिु L, बबंिु V के 18 दकमी िजक्षण में है, र्ो बबंिु D और बबंिु L के मध्य न्यूनर्म िूरी क्या है?
(a) 5km \5 दकमी
(b) 7km \7 दकमी
(c) 2km \2 दकमी
(d) 4km \4 दकमी
(e) None of these \इनमें से कोई नहीं

183 adda247.com/teachers | www.sscadda.com | www.bankersadda.com | www.adda247.com


Q93. Find out the shortest distance between point G and point F?
बबंिु G और बबंिु F के बीच की न्यूनर्म िूरी ज्ञार् कीजिए?
(a) 8km \8 दकमी
(b) 9km \9 दकमी
(c) 7km \7 दकमी
(d) 10km \10 दकमी
(e) None of these \इनमें से कोई नहीं

Q94. What is the direction of point V with respect to point D?


बबंिु D के सन्िभत में बबंिु V की दिशा क्या है?
(a) North-East \उत्तर-पूवत
(b) North-West \उत्तर-पजिम
(c) South-East \िजक्षण-पूवत
(d) South-West \िजक्षण-पजिम
(e) None of these \इनमें से कोई नहीं

Directions (95-99): In these questions, relationship between different elements is shown in the
statements. The statements are followed by conclusions.
जनम्नजिजिर् प्रश्नों में, कथनों में जवजभन्न र्त्वों के बीच संबंध को िशातया गया है। कथनों के बाि जनष्कर्त दिए गए हैं।
Give answer
उत्तर िीजिये

Q95. Statements/कथन:
D ≥ K < H=O; N > J ≥ H
Conclusions /जनष्कर्त
I. N> K
II. N < D
(a) If only conclusion I is true \यदि के वि जनष्कर्त I सत्य है
(b) If only conclusion II is true \यदि के वि जनष्कर्त II सत्य है
(c) If either conclusion I or II is true \यदि या र्ो जनष्कर्त I या II सत्य है
(d) If neither conclusion I nor II is true \यदि न र्ो जनष्कर्त I और न ही II सत्य है
(e) If both conclusions I and II are true \यदि जनष्कर्त I और II िोनों सत्य हैं

Q96. Statements/कथन:
S≥L≥U>B=G
Conclusions /जनष्कर्त
I. L > G
II. L = G
(a) If only conclusion I is true \यदि के वि जनष्कर्त I सत्य है
(b) If only conclusion II is true \यदि के वि जनष्कर्त II सत्य है
(c) If either conclusion I or II is true \यदि या र्ो जनष्कर्त I या II सत्य है
(d) If neither conclusion I nor II is true \यदि न र्ो जनष्कर्त I और न ही II सत्य है
(e) If both conclusions I and II are true \यदि जनष्कर्त I और II िोनों सत्य हैं

184 adda247.com/teachers | www.sscadda.com | www.bankersadda.com | www.adda247.com


Q97. Statements/कथन:
E>D≤J>V≥R≥S=F
Conclusions /जनष्कर्त
I. D > F
II. J ≥ D
(a) If only conclusion I is true \यदि के वि जनष्कर्त I सत्य है
(b) If only conclusion II is true \यदि के वि जनष्कर्त II सत्य है
(c) If either conclusion I or II is true \यदि या र्ो जनष्कर्त I या II सत्य है
(d) If neither conclusion I nor II is true \यदि न र्ो जनष्कर्त I और न ही II सत्य है
(e) If both conclusions I and II are true \यदि जनष्कर्त I और II िोनों सत्य हैं

Q98. Statements/कथन
D=E>K>X=B≤S≥C
Conclusions /जनष्कर्त
I. C < K
II. D ≤ S
(a) If only conclusion I is true \यदि के वि जनष्कर्त I सत्य है
(b) If only conclusion II is true \यदि के वि जनष्कर्त II सत्य है
(c) If either conclusion I or II is true \यदि या र्ो जनष्कर्त I या II सत्य है
(d) If neither conclusion I nor II is true \यदि न र्ो जनष्कर्त I और न ही II सत्य है
(e) If both conclusions I and II are true \यदि जनष्कर्त I और II िोनों सत्य हैं

Q99. Statements/कथन
N ≥ G > H ≥ I; M > H ≥ T
Conclusions /जनष्कर्त
I. N > T
II. M > I
(a) If only conclusion I is true \यदि के वि जनष्कर्त I सत्य है
(b) If only conclusion II is true \यदि के वि जनष्कर्त II सत्य है
(c) If either conclusion I or II is true \यदि या र्ो जनष्कर्त I या II सत्य है
(d) If neither conclusion I nor II is true \यदि न र्ो जनष्कर्त I और न ही II सत्य है
(e) If both conclusions I and II are true \यदि जनष्कर्त I और II िोनों सत्य हैं

Q100. In the given word “EXPRESSED” if all the consonants replaced with its next letter and all the vowels
replaced with its previous letter after that how many letters repeated in the new arrangement?
यदि दिए गए शब्ि “EXPRESSED” में सभी व्यंिनों को उसके अगिे अक्षर से बिि दिया िार्ा है और सभी स्वरों को उसके जपछिे
अक्षर से बिि दिया िार्ा है, र्ो उसके बाि नई व्यवस्था में दकर्ने अक्षरों की पुनरावृजत्त होर्ी है?
(a) Three \र्ीन
(b) Four \चार
(c) Two \िो
(d) None \कोई नहीं
(e) One \एक

185 adda247.com/teachers | www.sscadda.com | www.bankersadda.com | www.adda247.com


Solution

S1. Ans.(d)
Sol. “A large number of” is a plural noun, thus, it follows plural verb. Hence, “contributes” must be replaced
with “contribute”. Note, “the number of” is a singular noun.

S2. Ans.(d)
Sol. In part D, “clear” must be replaced with “clarity”. Here noun form of the word should be used thus
clarity is the right choice.

S3. Ans.(a)
Sol. Here in “was” must be replaced with “were”. In unreal conditions, i.e., the sentence that doesn’t reflect
reality if clause should be used, and if the verb in the if clause is “to be,” use “were,” even if the subject of
the clause is a third person singular subject (i.e., he, she, it). This will be followed by auxiliary verb such as
“would” in the second part of the hypothetical portion.

S4. Ans.(b)
Sol. “jury” is a collective noun, thus followed by a singular verb. Hence, “have” must be replaced with “has”.

S5. Ans.(b)
Sol. “to” should not be used in this case as had better, had rather, had as soon ... as .....had sooner etc. are
followed by Bare Infinitive.

S6. Ans.(c)
Sol. “latter” should be replaced with “later”. Later, latest refer to time, latter and last refer to position.

S7. Ans.(b)
Sol. Sentence (C) will be the introductory sentence as it best presents the theme of the paragraph which is
a black hole. The next sentence is (B) as it mentions the feature of a black hole. Further, sentence (B) will
be followed by sentence (D) as it further describes when this compression happens which is mentioned in
the sentence (B) as well. Now, sentence (D) will be followed by sentence (A) as it further describes the
second feature of the black hole that no light can come out of it. Further, sentence (A) will be followed by
sentence (E) as it describes the usage of space telescope with special tools to see these black holes.
Therefore, the correct rearrangement of the given sentences would be “CBDAE”.

S8. Ans.(d)
Sol. Sentence (C) will be the introductory sentence as it best presents
the theme of the paragraph which is a black hole. The next sentence
is (B) as it mentions the feature of a black hole. Further, sentence (B)
will be followed by sentence (D) as it further describes when this
compression happens which is mentioned in the sentence (B) as well.
Now, sentence (D) will be followed by sentence (A) as it further
describes the second feature of the black hole that no light can come
out of it. Further, sentence (A) will be followed by sentence (E) as it
describes the usage of space telescope with special tools to see these
black holes. Therefore, the correct rearrangement of the given
sentences would be “CBDAE”.

186 adda247.com/teachers | www.sscadda.com | www.bankersadda.com | www.adda247.com


S9. Ans.(c)
Sol. Sentence (C) will be the introductory sentence as it best presents the theme of the paragraph which is
a black hole. The next sentence is (B) as it mentions the feature of a black hole. Further, sentence (B) will
be followed by sentence (D) as it further describes when this compression happens which is mentioned in
the sentence (B) as well. Now, sentence (D) will be followed by sentence (A) as it further describes the
second feature of the black hole that no light can come out of it. Further, sentence (A) will be followed by
sentence (E) as it describes the usage of space telescope with special tools to see these black holes.
Therefore, the correct rearrangement of the given sentences would be “CBDAE”.

S10. Ans.(c)
Sol. Sentence (C) will be the introductory sentence as it best presents the theme of the paragraph which is
a black hole. The next sentence is (B) as it mentions the feature of a black hole. Further, sentence (B) will
be followed by sentence (D) as it further describes when this compression happens which is mentioned in
the sentence (B) as well. Now, sentence (D) will be followed by sentence (A) as it further describes the
second feature of the black hole that no light can come out of it. Further, sentence (A) will be followed by
sentence (E) as it describes the usage of space telescope with special tools to see these black holes.
Therefore, the correct rearrangement of the given sentences would be “CBDAE”.

S11. Ans.(a)
Sol. Sentence (C) will be the introductory sentence as it best presents the theme of the paragraph which is
a black hole. The next sentence is (B) as it mentions the feature of a black hole. Further, sentence (B) will
be followed by sentence (D) as it further describes when this compression happens which is mentioned in
the sentence (B) as well. Now, sentence (D) will be followed by sentence (A) as it further describes the
second feature of the black hole that no light can come out of it. Further, sentence (A) will be followed by
sentence (E) as it describes the usage of space telescope with special tools to see these black holes.
Therefore, the correct rearrangement of the given sentences would be “CBDAE”.

S12. Ans.(b)
Sol. Among the given statements, only option (b) is correct with reference to the context of the given
question. While the rest of the statements are incorrect as the major objective to undertake the
socialization process in orientation programs is to help the new employee adjust to his or her new
surroundings. Hence, option (b) is the right answer choice.
Refer to the eighth line of the first paragraph, “Orientation programs start off the socialization process and
help the new employee fit into his or her new surroundings.”

S13. Ans.(e)
Sol. Among the given statements, both (a) and (c) are correct with reference to the context of the given
question. While statement (b) is completely incorrect. Hence, option (e) is the right answer choice.
Refer to the last lines of the first paragraph, “New employee orientation programs occur during a period of
high anxiety and stress while training takes place when these potentially harmful feelings have most likely
been resolved. A great deal of time and money is spent every year socializing and technically training
employees.”

187 adda247.com/teachers | www.sscadda.com | www.bankersadda.com | www.adda247.com


S14. Ans.(c)
Sol. On reading the second paragraph thoroughly, we can infer that all of the given statements are correct
based on the information provided. Hence, option (c) is the right answer choice.

S15. Ans.(d)
Sol. Among the given statements, only (a) and (b) are correct with reference to the context of the given
question. While statement (c) is incoherent as it is not the primary objective behind the investment of time
and money by most of the organizations. Hence, option (d) is the right answer choice.
Refer to the mid-lines of the second paragraph, “Assessing the training needs of new employees consumes
time and money, but it sends the message that the organization is committed to the development of its
employees, to giving them the right tools to perform and putting them in a position where they can succeed,
all of which are recognized goals of new employee orientation programs.”

S16. Ans.(d)
Sol. The concerned sentence states that since technical skills training is intended to improve job-related
skills and abilities, it seems reasonable that it should be intrinsically correlated to whether or not a new
employee fits into his or her surroundings. Thus, we can infer that ‘analogy’ which means ‘a
correspondence or partial similarity’ is the most appropriate word to fill in the given blank. Hence, option
(d) is the right answer choice.
(a) confiscation- the action of taking or seizing someone's property with authority; seizure.
(b) requisition- demand the use or supply of (something) by official order.
(c) distraint- the seizure of someone's property in order to obtain payment of money owed.
(e) amusement- the state or experience of finding something funny.

S17. Ans.(e)
Sol. All of the given statements are completely correct based on the information provided in the given
passage. Thus, there is no incorrect statement given in the options. Hence, option (e) is the right answer
choice.
Refer to the last lines of the third paragraph, “Japanese firms hire almost entirely at the entry-level. This is
done so that the new arrival can be properly trained and socialized into the organization to be a good fit. The
length of this process can vary from six months to three years, whereas in many western organizations a new
employee merely completes the required HR paperwork and is then suddenly expected to execute the role for
which he or she was hired.”

S18. Ans.(d)
Sol. Among the given words, ‘chipper’ is the most appropriate antonym of the highlighted word. The word
‘stale’ means ‘no longer new and interesting’. While ‘chipper’ means ‘cheerful and lively’. Hence, option (d)
is the right answer choice.
(a) noxious- harmful, poisonous, or very unpleasant.
(b) virulent- (of a disease or poison) extremely severe or harmful in its effects.
(c) rancid- highly unpleasant; repugnant.

188 adda247.com/teachers | www.sscadda.com | www.bankersadda.com | www.adda247.com


S19. Ans.(b)
Sol. Among the given options, ‘execute’ is the most appropriate synonym of the highlighted word and is
closely relevant to the meaning of the concerned sentence. The word ‘run’ in the given sentence means ‘be
in or cause to be in operation; function or cause to function’. Hence, option (b) is the right answer choice.
(a) hasten- be quick to do something.
(b) execute- put (a plan, order, or course of action) into effect.
(c) cascade- pass (something) on to a succession of others.
(d) expedite- make (an action or process) happen sooner or be accomplished more quickly.

S20. Ans.(a)
Sol. Here, “enviroments” should be replaced with “environment”.

S21. Ans.(b)
Sol. Here, “brech” must be replaced with “breach”. “Breach” means “an act that breaks an agreement, a law,
etc.”

S22. Ans.(a)
Sol. “stending” must be replaced with “standing”

S23. Ans.(d)
Sol. “attandance” must be replaced with “attendance”.

S24. Ans.(e)
Sol. All words are correct, hence required no change.

S25. Ans.(a)
Sol. “suts” must be replaced with “suits”.

S26. Ans.(c)
Sol. The correct arrangement for the given sentence will be BDAC. Therefore the sentence will be” When
the Right to Education Act was promulgated over a decade ago, it seemed like a breakthrough.”

S27. Ans.(d)
Sol. The correct arrangement for the given sentence will be DACB. Therefore the sentence will be”
Agriculture is a sector that today demands even greater efficiency thus it relies on the use of agriculture
technologies.”

S28. Ans.(d)
Sol. The correct arrangement for the given sentence will be ACBD. Therefore the sentence will be”
Monitoring of student behavior often extends beyond schoolwork and normal school hours.

S29. Ans.(a)
Sol. The correct arrangement for the given sentence will be DCBA. Therefore the sentence will be”
Researchers and philosophers have explored in great detail the emotional dramas of love and family.”

189 adda247.com/teachers | www.sscadda.com | www.bankersadda.com | www.adda247.com


S30. Ans.(b)
Sol. The correct arrangement for the given sentence will be BDAC. Therefore the sentence will be” Poverty
is said to exist when people lack the means to satisfy their basic needs.”

S31. Ans.(b)
Sol.

S32. Ans.(d)
Sol.

S33. Ans.(e)
Sol.

S34. Ans.(d)
Sol.

190 adda247.com/teachers | www.sscadda.com | www.bankersadda.com | www.adda247.com


S35. Ans.(b)
Sol.

S36. Ans.(c)
Sol.

S37. Ans.(d)
Sol.

S38. Ans.(d)
Sol.

191 adda247.com/teachers | www.sscadda.com | www.bankersadda.com | www.adda247.com


S39. Ans.(b)
Sol.

S40. Ans.(b)
Sol.

S41. Ans.(d)
Sol.

S42. Ans.(c)
Sol.

S43. Ans.(c)
Sol.

192 adda247.com/teachers | www.sscadda.com | www.bankersadda.com | www.adda247.com


S44. Ans.(a)
Sol.

S45. Ans.(d)
Sol.

S46. Ans.(c)
Sol.

S47. Ans.(d)
Sol.

193 adda247.com/teachers | www.sscadda.com | www.bankersadda.com | www.adda247.com


S48. Ans.(e)
Sol.

S49. Ans.(e)
Sol.

S50. Ans.(a)
Sol.

S51. Ans.(d)
Sol.

S52. Ans.(d)
Sol.

194 adda247.com/teachers | www.sscadda.com | www.bankersadda.com | www.adda247.com


S53. Ans.(d)
Sol.

S54. Ans.(a)
Sol.

S55. Ans.(c)
Sol.

S56. Ans.(d)
Sol.

S57. Ans.(e)
Sol.

S58. Ans.(a)
Sol.

S59. Ans.(d)
Sol.

S60. Ans.(b)
Sol.

195 adda247.com/teachers | www.sscadda.com | www.bankersadda.com | www.adda247.com


S61. Ans.(b)
Sol.

S62. Ans.(c)
Sol.

S63. Ans.(d)
Sol.

S64. Ans.(e)
Sol.

S65. Ans.(c)
Sol.

S66. Ans.(c)
Sol.

196 adda247.com/teachers | www.sscadda.com | www.bankersadda.com | www.adda247.com


S67. Ans.(b)
Sol.

S68. Ans.(b)
Sol.

S69. Ans.(a)
Sol.

S70. Ans.(a)

S71. Ans.(b)
Sol.

197 adda247.com/teachers | www.sscadda.com | www.bankersadda.com | www.adda247.com


S72. Ans.(d)
Sol.

S73. Ans.(d)
Sol.

S74. Ans.(b)
Sol.

S75. Ans.(c)
Sol.

S76. Ans.(a)
Sol.

S77. Ans.(b)

S78. Ans.(b)
Sol.

198 adda247.com/teachers | www.sscadda.com | www.bankersadda.com | www.adda247.com


S79. Ans.(c)
Sol.

S80. Ans.(d)
Sol.

S81. Ans.(a)
Sol.

S82. Ans.(e)

S83. Ans.(b)

S84. Ans.(b)

S85. Ans.(a)

S86. Ans.(c)

199 adda247.com/teachers | www.sscadda.com | www.bankersadda.com | www.adda247.com


S87. Ans.(d)
Sol.

S88. Ans.(b)
Sol.

S89. Ans.(c)
Sol.

S90. Ans.(a)
Sol.

S91. Ans.(e)
Sol.

200 adda247.com/teachers | www.sscadda.com | www.bankersadda.com | www.adda247.com


S92. Ans.(c) S94. Ans.(b)
Sol. Sol.

S93. Ans.(d) S95. Ans.(a)


Sol.
S96. Ans.(a)

S97. Ans.(b)

S98. Ans.(d)

S99. Ans.(e)

S100. Ans.(c)
Sol. EXPRESSED
DYQSDTTDE

201 adda247.com/teachers | www.sscadda.com | www.bankersadda.com | www.adda247.com


Part - 9

Directions (1-8): Read the following passage and answer the following questions based on the given
passage. Some of the words are highlighted which would help you to answer some of the questions given.

A good night’s sleep is utmost essential for overall good health. It is the time when the body rests,
rejuvenates and recovers. An adult requires 7 to 8 hours of peaceful and quality sleep per day. Sleep
deprivation can be defined as inability to complete the sufficient sleep-time required by the person.
Research has shown that even one night of sleep deprivation is equivalent to being intoxicated. Intentional
sleep deprivation is mostly seen in young people and teenagers who prefer entertainment over sleep. Many
workaholics also consider sleeping as a waste of precious time which is not true. Many a time because of
work commitments such as working in night shifts or long hours may also interfere with quality sleep that
an individual requires. Medical issues such as chronic illnesses, depression and sleep disorders such as
obstructive sleep apnea can also be the reason for sleep deprivation.
You probably already have some understanding of the benefits of rest—and the costs of not getting it. Sleep
allows us to consolidate and store memories, process emotional experiences, replenish glucose (the
molecule that fuels the brain), and clear out beta-amyloid (the waste product that builds up in Alzheimer’s
patients and disrupts cognitive activity).
The most common sign and symptom of the fact that one is sleep deprived is fatigue, lethargy and feeling
sleepy throughout the day. Other symptoms include mood disorders; sleep deprivation may lead to
increasing irritability, desire to stay alone, rapid mood swings and more. Lack of sleep will also cause
psychomotor instability meaning the person will find it difficult to focus and stand still at a place. Sleep
deprivation will also cause issues with sight and hearing. An affected person may experience burning
sensation in eyes, tingling and redness of eyes, light flashes and even hallucinations. He or she may also
find it difficult to gauge distance at which a sound is originating. Other signs and symptoms of sleep
deprivation include tingling sensations on the body, disorganization of thought and much more.
Negative effects of sleep deprivation are many. It affects both physical and mental health of a person
negatively. The most common effect of sleep deprivation is drowsiness, tiredness, mood swings, irritability
and reduced alertness. Although scientific knowledge of the physiological effects of sleep deprivation is
relatively recent but researchers now believe that sleep deprivation can lead to disorders such as
depression. Both short term sleep deprivation and chronic long term sleep deprivation can be very
dangerous for the health as it has a direct impact on functioning of both heart and brain of an individual.
Sleep loss also blunted activity in brain regions that normally induce social engagement. During sleep,
regeneration of neurons happens in the cerebral cortex. Thus in a sleep deprived individual the brain fails
to function optimally.
The simplest and easiest way to treat sleep deprivation is sleep more. Be it acute or chronic condition, a
quality good night’s sleep will help an individual ________(A)_________ effectively. Fighting stress, eating a
healthy and a balanced diet, avoiding alcohol are some other dos for a good night’s sleep. Certain
medications can also interfere with one’s sleep thus consult your medical professional about the same.
Exercising or indulging in an activity such as jogging, walk or swimming can also help one sleep better.
Avoid usage of electronic gadgets before bedtime as they can interfere with one’ sleep. Spending time in
natural sunlight, Yoga, meditation and breathing exercises can also help one sleep better at night. Treating
underlying medical cause if any will also help one sleep better and avoid sleep deprivation.

202 adda247.com/teachers | www.sscadda.com | www.bankersadda.com | www.adda247.com


Q1. According to the passage, which among the following statements is true?
(a) Sleep allows us to enhance our ability of acquiring knowledge and process emotional experiences.
(b) An adult can function properly only with a quality sleep of 5-6 hours a day.
(c) Sleep deprivation can be cured only with a help of a medical professional.
(d) Both (A) and (B)
(e) All are true

Q2. What is/are the cause(s) of incompetency in a sleep deprived individual’s brain?
(i) Clearing out of beta-amyloid that disrupts cognitive activity.
(ii) Sleep deprivation leads to drowsiness, tiredness and reduced alertness.
(iii) Reduction in the regrowth or repair of nervous tissues, cells or cell products in the cerebral cortex.
(a) Only (i)
(b) Only (iii)
(c) Both (i) and (iii)
(d) Both (i) and (ii)
(e) All (i) (ii) and (iii)

Q3. Choose the most suitable phrasal verb to fill the given blank (A) to make the sentence grammatically
and contextually correct.
(a) drop back
(b) break out
(c) come forward
(d) bounce back
(e) fall out

Q4. Which among the followings is true according to the context of the passage?
(a) Fighting stress, eating a healthy and a balanced diet, avoiding alcohol cures depression
(b) Sleep deprivation means a state caused by inadequate quantity or quality of sleep.
(c) Sleep deprivation has negative effects on all the five senses of human body.
(d) Treating prolong and underlying medical cause can cause sleep deprivation.
(e) All are true

Q5. How can one safeguard oneself from sleep deprivation?


(i) Getting regular exercise during the day.
(ii) Refraining from using electronic devices right before bed
(iii) Limiting the consumption of alcohol
(a) Only (i)
(b) Only (iii)
(c) Both (i) and (iii)
(d) Both (i) and (ii)
(e) All (i) (ii) and (iii)

203 adda247.com/teachers | www.sscadda.com | www.bankersadda.com | www.adda247.com


Directions (6-7): Choose the most suitable word that depicts the meaning of the highlighted word.

Q6. SUFFICIENT
(a) Many
(b) Adequate
(c) Restricted
(d) Limited
(e) Wanting

Q7. INDUCE
(a) Hasten
(b) Inspire
(c) Encourage
(d) Advocate
(e) Evaluate

Q8. Which of the following word depicts the opposite meaning of the given word REPLENISH?
(a) Restore
(b) Exhaust
(c) Recharge
(d) Accelerate
(e) Modify

Directions (9-13): In the following questions, a sentence is divided into four parts. Choose the part of the
sentence which may contain a grammatical error in it. If all the parts of the sentence are grammatically
correct and contextually meaningful, choose option (E) i.e. “no error” as your answer choice.

Q9. Slicing a cake between pieces (A)/ is a fun tradition that many (B)/ newlyweds like to (C)/ incorporate
into their ceremonies. (D)/ No Error (E)
(a) A
(b) B
(c) C
(d) D
(e) E

Q10. She had stayed up (A)/ all night because (B)/ she had received (C)/ bad news. (D)/ No error. (E)
(a) A
(b) B
(c) C
(d) D
(e) E

204 adda247.com/teachers | www.sscadda.com | www.bankersadda.com | www.adda247.com


Q11. He as well as (A)/ his friends (B)/ were going (C)/ for picking.(D)/ No Error (E)
(a) A
(b) B
(c) C
(d) D
(e) E

Q12. The coaching industry in India (A)/ plays a pivotal role, (B)/ as it contributes significant (C)/ revenue
to the education sector. (D)/ No Error (E)
(a) A
(b) B
(c) C
(d) D
(e) E

Q13. If I don’t (A)/ turn on my (B)/ air conditioner, my (C)/ house was hot. (D)/ No error (E)
(a) A
(b) B
(c) C
(d) D
(e) E

Directions (14-17): In the following question, sentences are given with a part in bold. The given phrase in
the bold may or may not contain an error. The options following can replace the incorrect phrase. The
correct phrase that is to be replaced will be your answer. If the sentences are correct then select ‘No
improvement required’ as your answer.

Q14. Everything is funny, as long as it is happening too somebody else.


(a) has been happening for
(b) is happening to
(c) was happening too
(d) could have happened to
(e) No improvement required

Q15. They recommend this book even though they himself had never read it.
(a) they themselves had
(b) they themself has
(c) it itself had
(d) one oneself has
(e) No improvement required

Q16. This is the house that Jack built it.


(a) that Jack is building it
(b) Which Jack built it
(c) whom Jack built
(d) that Jack built
(e) No improvement required

205 adda247.com/teachers | www.sscadda.com | www.bankersadda.com | www.adda247.com


Q17. In no way do I agree with what you're saying.
(a) In no way I do
(b) In no way I don’t
(c) No way I don’t
(d) Do I in no way
(e) No improvement required

Directions (18-23): In the following passage some words have been deleted. Fill in the blanks with the
help of the alternatives given. Select the most appropriate option for each blank.

Q18. According to some researchers, musical instruments came about ____(18)___, depending on the
available materials, and, in some cases, by the stimulus of the clamor of battle. The definition of a musical
instrument is quite simple: it is an instrument that was created or ____(19)____ to make musical sounds. It
is important to remember that the history of musical instruments dates back to the beginning of human
culture. This tells us that man has always been drawn to music. The earliest musical instruments were used
for rituals. For example, the trumpet-like ones were used to signal success in a hunt, or drum-like
instruments were used in religious ceremonies.
Over time, cultures have developed the composition and performance of musical pieces for entertainment.
Musical instruments have also ___(20)__ with the ever-changing applications. It appears that no one knows
exactly where music came from. We are not talking here about who Elvis’ singing predecessors were, not
even about when the first musical instrument was invented. Researchers who have been interested in the
physics of instruments believe that musical devices have always ____(21)___ pretty much on the materials
that were available to each civilization. In other words, people worked with what they had. In time,
traditional poetry was ___(22)___ alongside human noise-makers and got rhythms to them. Again, there is
no exact data to back this up, so no one knows exactly when the first song was born. However, adding
stories to melodies became a tradition that was carried on to our days and it does not seem like it will ever
go out of fashion.
(a) deliberate
(b) systematic
(c) cautiously
(d) randomly
(e) anxiously

Q19. According to some researchers, musical instruments came about ____(18)___, depending on the
available materials, and, in some cases, by the stimulus of the clamor of battle. The definition of a musical
instrument is quite simple: it is an instrument that was created or ____(19)____ to make musical sounds. It
is important to remember that the history of musical instruments dates back to the beginning of human
culture. This tells us that man has always been drawn to music. The earliest musical instruments were used
for rituals. For example, the trumpet-like ones were used to signal success in a hunt, or drum-like
instruments were used in religious ceremonies.
Over time, cultures have developed the composition and performance of musical pieces for entertainment.
Musical instruments have also ___(20)__ with the ever-changing applications. It appears that no one knows
exactly where music came from. We are not talking here about who Elvis’ singing predecessors were, not
even about when the first musical instrument was invented. Researchers who have been interested in the
physics of instruments believe that musical devices have always ____(21)___ pretty much on the materials
that were available to each civilization. In other words, people worked with what they had. In time,
traditional poetry was ___(22)___ alongside human noise-makers and got rhythms to them. Again, there is
no exact data to back this up, so no one knows exactly when the first song was born. However, adding
stories to melodies became a tradition that was carried on to our days and it does not seem like it will ever
go out of fashion.
206 adda247.com/teachers | www.sscadda.com | www.bankersadda.com | www.adda247.com
(a) concise
(b) adapted
(c) prohibited
(d) distributed
(e) immense

Q20. According to some researchers, musical instruments came about ____(18)___, depending on the
available materials, and, in some cases, by the stimulus of the clamor of battle. The definition of a musical
instrument is quite simple: it is an instrument that was created or ____(19)____ to make musical sounds. It
is important to remember that the history of musical instruments dates back to the beginning of human
culture. This tells us that man has always been drawn to music. The earliest musical instruments were used
for rituals. For example, the trumpet-like ones were used to signal success in a hunt, or drum-like
instruments were used in religious ceremonies.
Over time, cultures have developed the composition and performance of musical pieces for entertainment.
Musical instruments have also ___(20)__ with the ever-changing applications. It appears that no one knows
exactly where music came from. We are not talking here about who Elvis’ singing predecessors were, not
even about when the first musical instrument was invented. Researchers who have been interested in the
physics of instruments believe that musical devices have always ____(21)___ pretty much on the materials
that were available to each civilization. In other words, people worked with what they had. In time,
traditional poetry was ___(22)___ alongside human noise-makers and got rhythms to them. Again, there is
no exact data to back this up, so no one knows exactly when the first song was born. However, adding
stories to melodies became a tradition that was carried on to our days and it does not seem like it will ever
go out of fashion.
(a) subdued
(b) evolved
(c) Refine
(d) Surge
(e) diminished

Q21. According to some researchers, musical instruments came about ____(18)___, depending on the
available materials, and, in some cases, by the stimulus of the clamor of battle. The definition of a musical
instrument is quite simple: it is an instrument that was created or ____(19)____ to make musical sounds. It
is important to remember that the history of musical instruments dates back to the beginning of human
culture. This tells us that man has always been drawn to music. The earliest musical instruments were used
for rituals. For example, the trumpet-like ones were used to signal success in a hunt, or drum-like
instruments were used in religious ceremonies.
Over time, cultures have developed the composition and performance of musical pieces for entertainment.
Musical instruments have also ___(20)__ with the ever-changing applications. It appears that no one knows
exactly where music came from. We are not talking here about who Elvis’ singing predecessors were, not
even about when the first musical instrument was invented. Researchers who have been interested in the
physics of instruments believe that musical devices have always ____(21)___ pretty much on the materials
that were available to each civilization. In other words, people worked with what they had. In time,
traditional poetry was ___(22)___ alongside human noise-makers and got rhythms to them. Again, there is
no exact data to back this up, so no one knows exactly when the first song was born. However, adding
stories to melodies became a tradition that was carried on to our days and it does not seem like it will ever
go out of fashion.
207 adda247.com/teachers | www.sscadda.com | www.bankersadda.com | www.adda247.com
(a) relatively
(b) suspected
(c) depended
(d) aligned
(e) sustained

Q22. According to some researchers, musical instruments came about ____(18)___, depending on the
available materials, and, in some cases, by the stimulus of the clamor of battle. The definition of a musical
instrument is quite simple: it is an instrument that was created or ____(19)____ to make musical sounds. It
is important to remember that the history of musical instruments dates back to the beginning of human
culture. This tells us that man has always been drawn to music. The earliest musical instruments were used
for rituals. For example, the trumpet-like ones were used to signal success in a hunt, or drum-like
instruments were used in religious ceremonies.
Over time, cultures have developed the composition and performance of musical pieces for entertainment.
Musical instruments have also ___(20)__ with the ever-changing applications. It appears that no one knows
exactly where music came from. We are not talking here about who Elvis’ singing predecessors were, not
even about when the first musical instrument was invented. Researchers who have been interested in the
physics of instruments believe that musical devices have always ____(21)___ pretty much on the materials
that were available to each civilization. In other words, people worked with what they had. In time,
traditional poetry was ___(22)___ alongside human noise-makers and got rhythms to them. Again, there is
no exact data to back this up, so no one knows exactly when the first song was born. However, adding
stories to melodies became a tradition that was carried on to our days and it does not seem like it will ever
go out of fashion.
(a) displayed
(b) recited
(c) elaborate
(d) arranged
(e) Remembered

Directions (23-26): In each of the questions given below a sentence is given with three words in bold.
Choose the option which gives the correct sequence of these words to make the sentence grammatically
and contextually correct.

Q23. The U.P. ordinance not only conflict(A) guaranteed fundamental rights but is also in violates(B) with
existing(C) personal laws
(a) ABC
(b) BAC
(c) CAB
(d) ACB
(e) None of these

208 adda247.com/teachers | www.sscadda.com | www.bankersadda.com | www.adda247.com


Q24. The deficit (A) changes in the sector brought in through the three laws have aggravated (B) the
trust abrupt (C) of the government.
(a) ABC
(b) CAB
(c) BAC
(d) CBA
(e) None of these

Q25. The Chinese Communist Party initially aftermath (A) nationalism as a co-option embraced (B) in
the strategy (C) of the Tiananmen Square massacre
(a) None of these
(b) CAB
(c) ABC
(d) BCA
(e) CBA

Q26. The Supreme Court has been asking States to reservation (A)quantifiable data to justify (B) their
levels of produce (C).
(a) None of these
(b) CAB
(c) ACB
(d) BCA
(e) CBA

Directions (27-30): In the question given below, some sentences/phrases are given which have to be
arranged in a proper sequence. Select the option which best defines the proper sequence and arranges the
sentence in an appropriate way.

Q27. (A) Sidestepping debate


(B) And discussion in Parliament
(C) Discontent over the
(D) Is a result of
(E) New farm laws
(a) DEABC
(b) EDABC
(c) No rearrangement possible
(d) AEBDC
(e) CEDAB

Q28. (A) Population-level vaccination plans


(B) For COVID-19
(C) Expecting from
(D) The outcomes we are
(E) It is crucial to define
(a)BCAED
(b)CABED
(c)EDCAB
(d)DCABE
(e) No rearrangement possible

209 adda247.com/teachers | www.sscadda.com | www.bankersadda.com | www.adda247.com


Q29. (A) WHO must
(B) Work alongside
(C) China in quickly
(D) Origins of the virus
(E) Uncovering the
(a)EBCDA
(b)CAEBD
(c)EBACD
(d)ACDBE
(e)ABCED

Q30. (A) Public activity should


(B) Be available
(C) To the public
(D) Only information
(E) Bearing a nexus to
(a) DEABC
(b) EDABC
(c) No rearrangement possible
(d) AEBDC
(e) CEDAB

Q31. Train A running at a speed of 36 km/hr crosses train B in 20 seconds. Find the speed of train B (in
km/hr), if the sum of the length of train A & B is 600 meters and both trains running in the opposite
direction.
(a) 72 km/hr
(b) 54 km/hr
(c) 48 km/hr
(d) 108 km/hr
(e) 81 km/hr

Q32. A invests Rs. X in a business. After four months B joined him with Rs. 2X and A double his investment.
If at the end of the years total profit is Rs. 13950, then find the profit share of A?
(a) 7250 Rs.
(b) 7750 Rs.
(c) 8750 Rs.
(d) 6750 Rs.
(e) 7050 Rs.

Q33. A man invested an amount in two schemes in the ratio of 2 : 3 at the rate of 20% p.a. and 10% p.a. on
compound interest respectively. If the man gets a total interest of Rs. 1208 after two years from both the
schemes, the find amount invested by man?
(a) 6000 Rs.
(b) 4800 Rs.
(c) 5000 Rs.
(d) 4500 Rs.
(e) 4000 Rs.

210 adda247.com/teachers | www.sscadda.com | www.bankersadda.com | www.adda247.com


Q34. The sum of the length of a rectangle and the side of a square is 72 meters. If the perimeter of the
rectangle is 84 meters and the breadth of the rectangle is 18 meters, then find the measurement of the side
of the square (in meters).
(a) 48
(b) 36
(c) 54
(d) 60
(e) 30

Q35. The ratio of time taken by A, B & C to complete work alone is 4: 6 : 3 respectively. If all three together
complete the work in 8 days, then find in how many days A alone complete three – fourth of the same work?
(a) 12
(b) 30
(c) 15
(d) 24
(e) 18

Directions (36-40): In the following questions, there are two equations in x and y. You have to solve both
the equations and give answer

Q36. I. 2x² – 3x + 1 = 0
II. 2y² – 5y + 3 = 0
(a) if x > y
(b) if x < y
(c) if x ≥ y
(d) if x ≤ y
(e) if x = y or there is no relation between x and y

Q37. I. x² + 21x + 110 = 0


II. y² + 17y + 72 = 0
(a) if x > y
(b) if x < y
(c) if x ≥ y
(d) if x ≤ y
(e) if x = y or there is no relation between x and y

Q38. I. x² = 4
II. y² – 6y + 8 = 0
(a) if x > y
(b) if x < y
(c) if x ≥ y
(d) if x ≤ y
(e) if x = y or there is no relation between x and y

211 adda247.com/teachers | www.sscadda.com | www.bankersadda.com | www.adda247.com


Q39. I. x² + 9x -22 = 0
II. 2y² - 7y + 6 = 0
(a) if x > y
(b) if x < y
(c) if x ≥ y
(d) if x ≤ y
(e) if x = y or there is no relation between x and y

Q40. I. 6x² + 5x + 1 = 0
II. 15y² + 11y + 2 = 0
(a) if x > y
(b) if x < y
(c) if x ≥ y
(d) if x ≤ y
(e) if x = y or there is no relation between x and y

Q41. The population of a village is decreased by 10% in the first year and then increased by 20% in the
second year. Find the population of the village at the end of the second year if two years ago it was 15,000?
(a) 16180
(b) 16200
(c) 16320
(d) 16360
(e) 16480

Q42. A shopkeeper marked up a shirt 50% above the cost price and allows successive discounts of
and 10% on it. If the difference between profit earned and discount given is Rs. 200 then find the marked
price of the shirt?
(a) Rs. 2100
(b) Rs. 3000
(c) Rs. 2400
(d) Rs. 3600
(e) Rs. 3300

Q43. The downstream speed of a boat is 5 km/hr more than its upstream speed and the ratio of the speed
of the boat in still water to the speed of the stream is 19: 5. Find the total time taken by boat to travel 42
km downstream and 31.5 km upstream?
(a) 7 ½ hr
(b) 8 hr
(c) 9 hr
(d) 9 ½ hr
(e) 10 hr

212 adda247.com/teachers | www.sscadda.com | www.bankersadda.com | www.adda247.com


Q44. A container contains a mixture of two liquids P and Q in the ratio 5 : 3. If 16 liter of the mixture is
taken out and replaced with liquid Q, then the new ratio of liquid P to liquid Q becomes 1: 1. Find the initial
quantity of mixture in the container.
(a) 80 L
(b) 60 L
(c) 70 L
(d) 48 L
(e) 96 L

Q45. The ratio of the present age of A to B is 8: 5 and the average of the present age of B and C is 35 years.
If five years ago, the sum of ages of A and B is 55 years, then find the difference between the present age of
A and C.
(a) 12 years
(b) 5 years
(c) 9 years
(d) 8 years
(e) 4 years

Directions (46-50): The line graph given below shows the total number of posts (Photos + Videos) shared
by six (A, B, C, D, E & F) people in December 2019. Read the data carefully and answer the questions.

Q46. The total post shared by C is what percent less than the total post shared by D?
(a) 20%
(b) 25%
(c) 15%
(d) 10%
(e) 30%

213 adda247.com/teachers | www.sscadda.com | www.bankersadda.com | www.adda247.com


Q47. In January 2020 total posts shared by B & F is 12 and 15 more than previous month respectively, then
find the total number of the post shared by B & F in January 2020?
(a) 95
(b) 91
(c) 93
(d) 97
(e) 99

Q48. Find the average number of posts shared by A, C & F?


(a) 42
(b) 48
(c) 40
(d) 36
(e) 44

Q49. Total photos shared by E is four more than total videos shared by him, then find total videos shared
by E?
(a) 24
(b) 20
(c) 28
(d) 22
(e) 30

Q50. If the ratio of total photos to total videos shared by B is 5: 9, then find total photos shared by B?
(a) 10
(b) 18
(c) 12
(d) 14
(e) 16

Directions (51-65): What will come in the place of question mark in following questions.

Q51. (? ÷5 ÷7) ×14+112=420


(a) 710
(b) 720
(c) 660
(d) 770
(e) 750

Q52.
(a) 4
(b) 6
(c) 8
(d) 2
(e) 3

214 adda247.com/teachers | www.sscadda.com | www.bankersadda.com | www.adda247.com


Q53.
(a) 64
(b) 62
(c) 60
(d) 56
(e) 82

Q54.
(a) 19
(b) 18
(c) 17
(d) 16
(e) 15

Q55.
(a) 10
(b) 20
(c) 30
(d) 40
(e) 22

Q56.
(a) 51
(b) 47
(c) 43
(d) 34
(e) 37

Q57. 47×27+15600÷8+181 = ?
(a) 3320
(b) 3420
(c) 3370
(d) 3400
(e) 3460

Q58. 112.5×5+4560÷6-175×7 = ?
(a)103.5
(b)91.5
(c) 97.5
(d) 110.5
(e) 115.5

215 adda247.com/teachers | www.sscadda.com | www.bankersadda.com | www.adda247.com


Q59.
(a) 80
(b) 120
(c) 150
(d) 180
(e) 240

Q60.
(a) 1
(b) 2
(c) 3
(d) 4
(e) 5

Q61. ? + 13× 50 = 420 + 45% of 800 + 220


(a) 300
(b) 350
(c) 400
(d) 450
(e) 250

Q62. 12 % of ? + 12.5% of 960 = 16 ×12


(a) 840
(b) 960
(c) 800
(d) 600
(e) 400

Q63. 15 × ? +20% of 450 = 360


(a) 12
(b) 20
(c) 24
(d) 16
(e) 18

Q64.
(a) 300
(b) 400
(c) 250
(d) 480
(e) 540

216 adda247.com/teachers | www.sscadda.com | www.bankersadda.com | www.adda247.com


Q65. ? % of 900 + 500 =4×197
(a) 28
(b) 25
(c) 36
(d) 40
(e) 32

Directions (66-70): Study the following information carefully and answer the question given below.

Eight boxes are placed one above the another. Only two boxes are placed between box U and T. Only one
box is placed between box T and R. Only two boxes are placed between box R and V. Box V is placed below
box R. Three boxes are placed between S and Q, both S and Q are placed above box T. Box W is placed below
box V. Not more than one box is placed between box S and box P.

Q66. How many boxes are there between box T and box S?
(a) None
(b) One
(c) Two
(d) Three
(e) More than three

Q67. Which of the following box is placed immediately above box Q?


(a) W
(b) U
(c) R
(d) T
(e) None of these

Q68. Which of the following box is placed at the bottommost position?


(a) W
(b) U
(c) R
(d) T
(e) None of these

Q69. Which of the following box is placed immediately below box P?


(a) W
(b) U
(c) R
(d) T
(e) None of these

217 adda247.com/teachers | www.sscadda.com | www.bankersadda.com | www.adda247.com


Q70. How many boxes are there between box W and box R?
(a) None
(b) One
(c) Two
(d) Three
(e) More than three

Q71. In the word ‘BACKGROUND’ all consonants are written as their preceding letter and all vowels are
written as their following letters. Then, how many letters repeated in the new arrangement?
(a) One
(b) Two
(c) Four
(d) Three
(e) More than four

Directions (72-76): Study the following information carefully and answer the question given below-

Six persons are arranged according to their weight in descending order from left to right. E is not the
heaviest among all. A is heavier than F and C. F is heavier than B and D but not heavier than E. B and C are
not the lightest among all. A is not just heavier than F. E is heavier than C, who is not the 3rd heaviest among
all. Weight of the 2nd lightest person is 17kg. Total weight of B and C is 44kg. 2 nd heaviest person is twice
heavy to the one who is 2nd lightest. C is just lighter than F. Total weight of all the persons are 150kg.

Q72. What may be the possible weight of F?


(a) 30kg
(b) 38kg
(c) 24kg
(d) 25kg
(e) None of these

Q73. The number of persons is heavier than F is same as the number of persons lighter than ___?
(a) A
(b) C
(c) D
(d) B
(e) None of these

Q74. If D is 10kg lighter than B and F is 5kg heavier than C then what is the weight of A?
(a) 23kg
(b) 43kg
(c) 30kg
(d) 33kg
(e) None of these

218 adda247.com/teachers | www.sscadda.com | www.bankersadda.com | www.adda247.com


Q75. Four of the following five are alike in a certain way and so form a group. Find the one who does not
belong to that group?
(a) E-C
(b) F-A
(c) D-C
(d) B-D
(e) F-B

Q76. Which of the following statement is true?


(a) Only one person is lighter to C
(b) D is the lightest among all
(c) A is lighter to D
(d) Weight of A is 50 kg
(e) None is true

Directions (77-80): In these questions, relationship between different elements is shown in the
statements. The statements are followed by conclusions. Give answer

Q77. Statements: F > R ≥ T = E > W ≤ Q


Conclusions: I. E < F
II. Q ≥ T
(a) If only conclusion I is true
(b) If only conclusion II is true
(c) If either conclusion I or II is true
(d) If neither conclusion I nor II is true
(e) If both conclusions I and II are true

Q78. Statements: W = G ≥ H ≥ T = C ≤ V ≤ B
Conclusions: I. T < G
II. C = G
(a) If only conclusion I is true
(b) If only conclusion II is true
(c) If either conclusion I or II is true
(d) If neither conclusion I nor II is true
(e) If both conclusions I and II are true

Q79. Statements: K > I ≥ G > F ≤ T < R


Conclusions: I. K > T
II. R > F
(a) If only conclusion I is true
(b) If only conclusion II is true
(c) If either conclusion I or II is true
(d) If neither conclusion I nor II is true
(e) If both conclusions I and II are true

219 adda247.com/teachers | www.sscadda.com | www.bankersadda.com | www.adda247.com


Q80. Statements: K < G ≤ D < C > R ≤ Y
Conclusions: I. C > G
II. K < D
(a) If only conclusion I is true
(b) If only conclusion II is true
(c) If either conclusion I or II is true
(d) If neither conclusion I nor II is true
(e) If both conclusions I and II are true

Q81. In the Given number ‘843576529’ How many such numerals will remain at the same position, when
arranged in ascending order from left to right?
(a) More than Four
(b) Four
(c) Two
(d) Three
(e) One

Directions (82-86): Study the following information carefully and answer the question given below-

Eight persons A, B, C, D, E, F, G and H were born in the month of December of 4 different years i.e. 1956,
1957, 1958 and 1959 either on 14 th or 23rd but not necessarily in the same order. E was born in the even
numbered year but not the eldest among all. More than two persons were born between E and D. H was
born just after D. There are as many persons born before H as same as born after B. G was born before F
and after A and they all were born on the same date.

Q82. How many persons born before C?


(a) One
(b) None
(c) Two
(d) More than four
(e) Four

Q83. In which of the following year B was born?


(a) 1956
(b) 1959
(c) Either 1956 or 1959
(d) 1957
(e) 1958

Q84. How many persons were born between A and F?


(a) Three
(b) Five
(c) Two
(d) One
(e) None of these

220 adda247.com/teachers | www.sscadda.com | www.bankersadda.com | www.adda247.com


Q85. Four of the following five are alike in a certain way and so form a group. Find the one who does not
belong to that group?
(a) D
(b) B
(c) E
(d) C
(e) H

Q86. The number of persons born between G and H is same as between ___ and ___?
(a) E, F
(b) B, H
(c) C, B
(d) D, G
(e) None of these

Directions (87-90): In each question below are given some statements followed by two conclusions
numbered I and II. You have to take the given statements to be true even if they seem to be at variance with
commonly known facts and then decide which of the given conclusions logically follows from the given
statements, disregarding commonly known facts. Give answer:

Q87. Statements: Only a few Photo are Video. Only a few Frame are Video. All Frame are Camera.
Conclusions: I. All Photo being Frame is a possibility.
II. Some Photo are not Camera.
(a) If both conclusions I and II follow.
(b) If only conclusion II follows.
(c) If only conclusion I follows.
(d) If neither conclusion I nor II follows.
(e) If either conclusion I or II follows.

Q88. Statements: Only a few City are Town. No Town is Village. All Village are Block.
Conclusions: I. All City can be Village.
II. Some Block are not Town.
(a) If both conclusions I and II follow.
(b) If either conclusion I or II follows.
(c) If only conclusion I follows.
(d) If neither conclusion I nor II follows.
(e) If only conclusion II follows.

Q89. Statements: Some Bird are not Eagle. Some Eagle are Parrot.
Conclusions: I. Some Parrot are Bird.
II. No Parrot is Bird.
(a) If only conclusion I follows.
(b) If either conclusion I or II follows.
(c) If both conclusions I and II follow.
(d) If neither conclusion I nor II follows.
(e) If only conclusion II follows.

221 adda247.com/teachers | www.sscadda.com | www.bankersadda.com | www.adda247.com


Q90. Statements: Only Banana is Apple. Some Banana are Papaya.
Conclusions: I. Some Apple being Papaya is a possibility.
II. All Banana can be Apple.
(a) If both conclusions I and II follow.
(b) If only conclusion I follows.
(c) If either conclusion I or II follows.
(d) If neither conclusion I nor II follows.
(e) If only conclusion II follows.

Directions (91-95): Study the following information carefully and answer the questions given below:

Eight persons D, E, F, G, M, O, P and S purchase some products one after another but not necessarily in the
same order. At most two persons purchase before F. Only one-person purchase between D and F. P
purchase just before S. One-person purchase between P and D. M purchase just before E. O purchase before
G and after E.

Q91. How many persons purchase after D?


(a) None
(b) Two
(c) More than three
(d) One
(e) None of these

Q92. Who among the following purchase just after G?


(a) E
(b) F
(c) O
(d) P
(e) None of these

Q93. If all the persons are arranged in alphabetical order from top to bottom starting from D, then find
how many persons remains at the same position (excluding D)?
(a) One
(b) None
(c) Two
(d) Four
(e) More than Four

Q94. Who among the following purchase exactly between D and F?


(a) E
(b) O
(c) M
(d) S
(e) None of these

222 adda247.com/teachers | www.sscadda.com | www.bankersadda.com | www.adda247.com


Q95. How many persons purchase between E and P?
(a) Five
(b) Four
(c) Three
(d) None
(e) Two

Q96. How many pairs of letters are there in the word “CHRISTMAS” which has as many letters between
them as we have in the English alphabetical series (from both forward and backward direction)?
(a) Three
(b) One
(c) Two
(d) None
(e) More than three

Directions (97-100): Study the following arrangement carefully and answer the questions given below:
39181258636425285264514947327259

Q97. How many 2’s is there in the above arrangement, each of which is immediately preceded by a digit
which has a numerical value of more than 5?
(a) None
(b) One
(c) Two
(d) Three
(e) More than three

Q98. Which of the following number is seventh to the left of the twentieth from the left end of the above
arrangement?
(a) 3
(b) 9
(c) 2
(d) 7
(e) 1

Q99. If all the even digits are removed from the above arrangements then, which of the following digit will
be tenth from the right end of the arrangement?
(a) 9
(b) 5
(c) 1
(d) 3
(e) 7

Q100. How many such 1’s is there in the above arrangement, each of which is immediately followed by a
perfect square?
(a) None
(b) One
(c) Two
(d) Three
(e) More than three

223 adda247.com/teachers | www.sscadda.com | www.bankersadda.com | www.adda247.com


Solution

S1. Ans.(a)
Sol. Only statement (a) is correct.
For statement (a) Refer to the 2nd paragraph, “. Sleep allows us to consolidate and store memories, process
emotional experiences, replenish glucose (the molecule that fuels the brain), and clear out beta-amyloid
(the waste product that builds up in Alzheimer’s patients and disrupts cognitive activity).”
For statement (b) refer to the 1st paragraph, “An adult requires 7 to 8 hours of peaceful and quality sleep
per day.”
For statement (c) refer to the last paragraph, “The simplest and easiest way to treat sleep deprivation is
sleep more.”

S2. Ans.(b)
Sol. Refer to the last sentence of the 3rd paragraph “During sleep, regeneration of neurons happens in the
cerebral cortex. Thus, in a sleep deprived individual the brain fails to function optimally.” Hence, option (b)
is correct.

S3. Ans.(d)
Sol. Bounce back means to return quickly to a normal condition after a difficult situation.
Drop back means fall back or get left behind.
Break out means escape.
Come forward means volunteer oneself for a task or post or to give evidence about a crime.
Fall out means have an argument.

S4. Ans.(b)
Sol. Among the given statements only statement (b) is correct.
For statement (b) refer to the 1st paragraph “Sleep deprivation can be defined as inability to complete the
sufficient sleep-time required by the person.”
All the other statements are factually incorrect.

S5. Ans.(e)
Sol. Refer to the last paragraph, “Fighting stress, eating a healthy and a balanced diet, avoiding alcohol
are some other dos for a good night’s sleep. Certain medications can also interfere with one’s sleep thus
consult your medical professional about the same. Exercising or indulging in an activity such as jogging,
walk or swimming can also help one sleep better. Avoid usage of
electronic gadgets before bedtime as they can interfere with one’
sleep. Spending time in natural sunlight, Yoga, meditation and
breathing exercises can also help one sleep better at night.”

S6. Ans.(b)
Sol. Sufficient means enough; adequate.

S7. Ans.(c)
Sol. Induce means bring about or give rise to.
Encourage means stimulate the development of (an activity, state, or
belief).

224 adda247.com/teachers | www.sscadda.com | www.bankersadda.com | www.adda247.com


S8. Ans.(b)
Sol. Replenish means restore (a stock or supply) to a former level or condition.

S9. Ans.(a)
Sol. Replace ‘between’ by ‘into’. “Into” is used to show when a person or thing is changing from one form
or condition to another:

S10. Ans.(a)
Sol. The past perfect tense in a sentence or conversation describes an event that happened in the past
before another event in the simple past tense was completed in the past. Thus the correct phrase should
be “She stayed up”.

S11. Ans.(c)
Sol. ‘were; should be replaced by ‘was’. Words joined to a singular subject by ‘with’, ‘together with’, ‘in
addition to’, ‘or’, ‘as well as’, ‘besides’ etc. The verb must take place with the first subject of the sentence.

S12. Ans.(e)
Sol. The sentence is grammatically correct.

S13. Ans.(d)
Sol. ‘was’ should be replaced by ‘is’. Zero conditional sentences refer to the general truth about a situation.
These sentences state that one condition always results in the same outcome. Note that the both clauses
should be in the present tense.

S14. Ans.(b)
Sol. Replace “too” with “to”
To is a preposition with several meanings, including “toward” and “until.”
Too is an adverb that can mean “excessively” or “also.”

S15. Ans.(a)
Sol. reflexive pronouns can also act as intensive pronouns, but the function and usage are different. An
intensive pronoun emphasizes its antecedent. Hence, the correct phrase is “they themselves had”. Thus,
option (a) is correct.

S16. Ans.(d)
Sol. The relative pronoun is the subject/object of the relative clause, so we do not repeat the subject/object,
Here, that is the object of built, so we don't need ‘it’.

S17. Ans.(e)
Sol. The given sentence is grammatically correct.

S18. Ans.(d)
Sol. Option (d) is the correct choice.
“Randomly”: in a way that happens, is done, or is chosen by chance rather than according to a plan.
All the other words are either grammatically or contextually incorrect.
Cautiously: in a careful and well-considered way that avoids risk
Anxiously: in a way that shows you are worried or nervous

225 adda247.com/teachers | www.sscadda.com | www.bankersadda.com | www.adda247.com


S19. Ans.(b)
Sol. “Adapted” is the correct choice.
Adapted: to change something so that you can use it in a different situation
Concise: giving a lot of information in a few words; brief
Immense: very big or great
Prohibited: to say that something is not allowed by law; to forbid

S20. Ans.(b)
Sol. Option (b) is the correct choice.
Subdued: quieter and with less energy than usual
Evolved: to develop or to make something develop gradually, from a simple to a more advanced form
Refine: to improve something by changing little details
Surge: a sudden strong movement in a particular direction by a large number of people or things

S21. Ans.(c)
Sol. Option (c) is the correct choice.
Meaning of some other words:
Sustained: to make something continue for a long period of time without becoming less
Aligned: to arrange things in a straight line or so that they are parallel to something else

S22. Ans.(b)
Sol. Option (b) is the correct choice.
Recited-- to say aloud a piece of writing, especially a poem or a list, from memory
Elaborate-- very complicated; done or made very carefully

S23. Ans.(b)
Sol. “BAC” is the correct sequence.
The U.P. ordinance not only violates guaranteed fundamental rights but is also in conflict with existing
personal laws

S24. Ans.(d)
Sol. “CBA” is the correct sequence.
The abrupt changes in the sector brought in through the three laws have aggravated the trust deficit of the
government.

S25. Ans.(d)
Sol. “BCA” is the correct sequence
The Chinese Communist Party initially embraced nationalism as a co-option strategy in the aftermath of
the Tiananmen Square massacre

S26. Ans.(e)
Sol. “CBA” is the correct sequence.
The Supreme Court has been asking States to produce quantifiable data to justify their levels of reservation.

226 adda247.com/teachers | www.sscadda.com | www.bankersadda.com | www.adda247.com


S27. Ans.(e)
Sol. Option (e)- CEDAB is the correct sequence for the given question.
“Discontent over the new farm laws is a result of sidestepping debate and discussion in Parliament”

S28. Ans.(c)
Sol. Option (c)- EDCAB is the correct sequence for the given question.
“It is crucial to define the outcomes we are expecting from population-level vaccination plans for COVID-
19 “

S29. Ans.(e)
Sol. Option (e)- ABCED is the correct sequence for the given question.
WHO must work alongside China in quickly uncovering the origins of the virus.

S30. Ans.(a)
Sol. DEABC is the correct sequence for the given question.
Only information bearing a nexus to public activity should be available to the public

S31. Ans.(a)
Sol.

S32. Ans.(b)
Sol.

S33. Ans.(e)
Sol.

227 adda247.com/teachers | www.sscadda.com | www.bankersadda.com | www.adda247.com


S34. Ans.(a)
Sol.

S35. Ans.(e)
Sol.

S36. Ans.(d)
Sol.

S37. Ans.(b)
Sol.

S38. Ans.(d)
Sol.

228 adda247.com/teachers | www.sscadda.com | www.bankersadda.com | www.adda247.com


S39. Ans.(e)
Sol.

S40. Ans.(e)
Sol.

S41. Ans.(b)
Sol. required population = 15000 × 0.9 × 1.2
= 16200

S42. Ans.(b)
Sol.

229 adda247.com/teachers | www.sscadda.com | www.bankersadda.com | www.adda247.com


S43. Ans.(b)
Sol. Let the speed of boat in still water be x km/hr and that of stream be y km/hr.

S44. Ans.(a)
Sol. Let initial quantity of liquid P and Q in container be 5x & 3x respectively

S45. Ans.(b)
Sol.

S46. Ans.(a)
Sol.

S47. Ans.(c)
Sol. Required sum = (28 + 12) + (38 + 15) = 93

S48. Ans.(c)
Sol.

S49. Ans.(b)
Sol. Let total videos shared by E = x
So, total photos shared by E = (x + 4)
ATQ –
x + x + 4 = 44
2x = 40
x = 20

230 adda247.com/teachers | www.sscadda.com | www.bankersadda.com | www.adda247.com


S50. Ans.(a)
Sol.

S51. Ans.(d)
Sol.
(? ÷5 ÷7) ×14=308
? = 22 ×35
? = 770

S52. Ans.(b)
Sol.

S53. Ans.(b)
Sol.

S54. Ans.(c)
Sol.

S55. Ans.(b)
Sol.

S56. Ans.(e)
Sol.

231 adda247.com/teachers | www.sscadda.com | www.bankersadda.com | www.adda247.com


S57. Ans.(d)
Sol.
?= 47×27+15600÷8+181
= 1269 + 1950 +181
=3400

S58. Ans.(c)
Sol.
? = 112.5×5+4560÷6-175×7
=562.5 + 760 – 1225
=1322.5 – 1225
= 97.5

S59. Ans.(c)
Sol.

S60. Ans.(a)
Sol.

S61. Ans.(b)
Sol.

S62. Ans.(d)
Sol.

232 adda247.com/teachers | www.sscadda.com | www.bankersadda.com | www.adda247.com


S63. Ans.(e)
Sol.

S64. Ans.(a)
Sol.

S65. Ans.(e)
Sol.

S66. Ans.(e)
Sol.

S67. Ans.(c)
Sol.

233 adda247.com/teachers | www.sscadda.com | www.bankersadda.com | www.adda247.com


S68. Ans.(a)
Sol.

S69. Ans.(b)
Sol.

S70. Ans.(d)
Sol.

S71. Ans.(a)
Sol.

S72. Ans.(a)
Sol.

234 adda247.com/teachers | www.sscadda.com | www.bankersadda.com | www.adda247.com


S73. Ans.(b)
Sol.

S74. Ans.(d)
Sol.

S75. Ans.(d)
Sol.

S76. Ans.(b)
Sol.

S77. Ans.(a)
Sol. I. E < F (True)
II. Q ≥ T (False)

S78. Ans.(c)
Sol. I. T < G (False)
II. C = G (False)

S79. Ans.(b)
Sol. I. K > T (False)
II. R > F (True)

S80. Ans.(e)
Sol. I. C > G (True)
II. K < D (True)

S81. Ans.(d)
Sol.

235 adda247.com/teachers | www.sscadda.com | www.bankersadda.com | www.adda247.com


S82. Ans.(c)
Sol.

S83. Ans.(b)
Sol.

S84. Ans.(a)
Sol.

S85. Ans.(e)
Sol.

236 adda247.com/teachers | www.sscadda.com | www.bankersadda.com | www.adda247.com


S86. Ans.(c)
Sol.

S87. Ans.(c)
Sol.

S88. Ans.(e)
Sol.

S89. Ans.(b)
Sol.

237 adda247.com/teachers | www.sscadda.com | www.bankersadda.com | www.adda247.com


S90. Ans.(d)
Sol.

S91. Ans.(e)
Sol.

S92. Ans.(d)
Sol.

S93. Ans.(d)
Sol.

238 adda247.com/teachers | www.sscadda.com | www.bankersadda.com | www.adda247.com


S94. Ans.(b)
Sol.

S95. Ans.(b)
Sol.

S96. Ans.(c)
Sol.

S97. Ans.(b)
Sol. “7 2”

S98. Ans.(c)
Sol. “2”

S99. Ans.(b)
Sol. “5”

S100. Ans.(b)
Sol. “1 4”

239 adda247.com/teachers | www.sscadda.com | www.bankersadda.com | www.adda247.com


Part - 10

REASONING ABILITY

Direction (1-5): Study the following information carefully 7. How many persons are sitting between P and S?
and answer the questions given below: (a) Three (b) Five (c) None
Eight persons A, B, C, D, E F, G and H are going to watch movie (d) Two (e) More than five
in different months i.e. January, April. May, June, July, August, 8. What is the position of D with respect to F?
October and December but not necessarily in the same order. (a) Fifth to the left
F was going to watch movie in the month which has 30 days. (b) Sixth to the right
There are three persons watching movie between F and C. H (c) Second to the left
watches the movie just before A, who is watching the movie (d) Fifth to the right
before B. B watches the movie just before C. H is watching the (e) None of these
movie in the month which has maximum number of days but 9. How many persons are sitting to the left of H?
not in January. D is watching the movie in the month which (a) One (b) Three (c) Two
has 30 days. G is watching the movie after D. (d) Four (e) None of these
1. If E is related to F and H is related to C then, in the same 10. If A is sitting exactly in between the P and D, then what
way D is related to which of the following ? is the position of A with respect to S?
(a) H (b) F (c) C (a) Third to the left
(d) B (e) A (b) None of these
2. Which of the following statement is true as per the given (c) Third to the right
information? (d) Second to the left
(a) D is going to watch movie after F (e) Second to the right
(b) Two persons are watching movie between G and B 11. In the word ‘CHLORINE’, how many pairs of the letters
(c) B is watching the movie in January have the same number of letters between them in the
(d) A is watching the movie in August word as in alphabet?
(e) None is true (a) Four (b) Two (c) One
3. The number of persons watching the movie between D (d) Three (e) More than four
and F is same as between B and ___? Direction (12-15): In each of the questions below are given
(a) E (b) F (c) H some statements followed by some conclusions. You have to
(d) A (e) None of these take the given statements to be true even if they seem to be at
4. Four of the following five are alike in a certain way and variance with commonly known facts. Read all the
hence they form a group. Which one of the following conclusions and then decide which of the given conclusions
does not belong to that group? logically follows from the given statements disregarding
(a) H (b) D (c) C commonly known facts.
(d) B (e) G (a) If only conclusion I follows.
5. How many persons are watching the movie between B (b) If only conclusion II follows.
and D? (c) If either conclusion I or II follows.
(a) Five (b) Three (c) Four (d) If neither conclusion I nor II follows.
(d) Two (e) None (e) If both conclusions I and II follow.
Direction (6-10): Study the following information carefully 12. Statements: Only a few home is picnic. No picnic is
and answer the questions given below: mystery. All mystery is real.
Conclusions: I. Some mystery are not picnic .
A certain number of persons are sitting in a row facing in II. All home being picnic is a possibility.
north direction. Four persons are sitting between P and Q. F
sits second to the right of Q. One person sits between F and S. 13. Statements: Only a few strong is tough. Only a few tough
S sits at eighth position from one of the extreme ends. D sits is dark
third to the right of S. H sits fifth to the left of Q. Eight persons Conclusions: I. Some strong is not dark .
are sitting to the left of Q. II. No tough is strong.
6. What is the maximum possible number of persons are 14. Statements: No network is dull. Only a few dull is fast.
sitting in a row? All fast is memory.
(a) Twenty (b) Twenty-two (c) Twenty-six Conclusions: I. Some memory are definitely not network.
(d) None of these (e) Twenty-five II. Some dull are not network.

240 adda247.com/teachers | www.sscadda.com | www.bankersadda.com | www.adda247.com


15. Statements: All tennis is football. All football is 21. How many persons are sitting between F and J when
basketball. No basketball is cricket. counting from the left of F?
Conclusions: I. No football is cricket. (a) One (b) Two (c) Three
II. All tennis is basketball. (d) Four (e) More than four
Direction (16-20): Study the following information carefully 22. Four of the following five are alike in a certain way and
and answer the questions given below: hence form a group, which of the following does not
Twelve persons are sitting in the two parallel rows containing belong to the group?
six persons in each row in such a way that there is an equal (a) S (b) F (c) J
distance between adjacent persons. In the first row, A, B, C, D, (d) K (e) N
E and F are seated and all of them are facing north. In the 23. Which of the following statement is true?
second row, P, Q, R, S, T and U are seated and all of them are (a) P sits second to the left of V
facing south. Therefore, in the given seating arrangement, (b) N is an immediate neighbor of S
each member seated in a row faces another member of the (c) Two persons sit between P and V
other row. (d) M sits opposite to N
E sits 4th to the right of B. Q faces to E. The number of persons (e) J sits opposite to K.
sit to the left of Q is same as sit to the right of S. F sits
immediate to the left of D. A sits to the left of C and to the right 24. Who among the following is sitting second to the left of
of F. P faces the one who is an immediate neighbor of A. U is M?
the only neighbor of Q. R sits to the right of T. (a) P (b) V (c) J
(d) F (e) None of these
16. Four of the following five are alike in a certain way and
hence they form a group. Which one of the following 25. Who among the following is sitting opposite to P?
does not belong to that group? (a) V (b) J (c) F
(a) R (b) P (c) U (d) M (e) None of these
(d) C (e) B Direction (26-28): Study the following information carefully
17. Who among the following faces to F? and answer the questions given below:
(a) P (b) U (c) S There are seven members in a family of three generation. A is
(d) R (e) Either (a) or (c) mother of P. P is brother of G. K is married to G. S is aunt of M.
18. What is the position of F with respect to B? K is child of L. S is sister of K.
(a) Immediate to the left 26. If L is married to J, then how J is related to G?
(b) 2nd to the left (a) Grand daughter
(c) 3rd to the right (b) Grand son
(d) 4th to the left (c) Son-in-law
(e) Immediate to the right (d) Daughter-in-law
19. Which of the following is true as per the given (e) Can’t be determined
information? 27. If A is married to R , then how R is related to M?
(a) S sits to the immediate right of P (a) Grand father (b) Brother-in-law (c) Uncle
(b) U faces to A (d) None of these (e) Can’t be determined
(c) F sits at the extreme end
(d) Both (b) and (c) true 28. How S is related to G?
(e) All are true (a) Sister (b) Sister-in-law (c) Aunt
(d) Mother-in-law (e) None of these
20. Who among the following sits 3rd to the right T?
(a) U (b) None of these (c) S 29. If it is possible to make only one meaningful word with
(d) P (e) R the 2nd , 5th, 8th and 9th letters of the word
Direction (21-25): Study the following information carefully ‘TRANSLATION’ which would be the second letter of the
and answer the questions given below: word from the left? If more than one such word can be
formed give ‘Y’ as the answer. If no such word can be
Eight persons i.e. F, J, K, M, N, P, S and V are sitting around a formed, give ‘Z’ as your answer.
square table. Four of them are sitting at the corners and (a) Y (b) S (c) T
remaining are sitting at the middle side of the table. The (d) Z (e) R
persons sitting at the corners faces away from the center and
the persons sitting at the middle sides of the table faces Directions (30-33): These questions are based on the
towards the center. All the information is not necessarily in following arrangement. Study the arrangement carefully to
the same order. answer these questions.
F sits at the middle side of the table. N sits at the immediate RQD8H9LOPTEFM6IA7J54BU2G
left of V. Two persons sit between F and J. V sits at the 30. If all the numbers are remove from the series, then
immediate right of J. Three persons sit between S and N. M is which of the following element is 10th from the left end?
an immediate neighbor of S. M is not an immediate neighbor (a) R (b) F (c) M
of J. P faces inside.
(d) B (e) U

241 adda247.com/teachers | www.sscadda.com | www.bankersadda.com | www.adda247.com


31. How many alphabets according to the English dictionary (a) Three (b) One (c) Two
are between the elements which is 7th from the left end (d) Four (e) None of these
and 9th from the right end? 34. If in the number 485794362, 1 is added to each of the
(a) Five (b) Three (c) Seven digit which is placed at even position and 1 is subtracted
(d) Ten (e) Eight from each of the digit which is placed at odd position
32. Which of the following element is 3rd from the right of then how many digits are repeating in the number thus
the element which is 11th from the left end? formed?
(a) M (b) I (c) A (a) Only 5 (b) Both 1 & 2 (c) Only 8
(d) 6 (e) R (d) Both 1 & 7 (e) None of these
33. How many such numbers are there in the above 35. Find the odd one out?
arrangement, each of which is immediately preceded by (a) BEH (b) JMP (c) TWZ
a consonant and also immediately followed by a (d) CFI (e) KOP
consonant?

QUANTITATIVE APTITUDE
Direction (36-45) : Find the value of (?) in the following 500
questions
36. √124+? +169 = 18 450

Number of Residents
(a) 34 (b) 31 (c) 33
(d) 35 (e) 32 400
37. 136 ÷ 22 ×? = 17% of 500 ÷ 10 350
(a) 0.75 (b) 1.33 (c) 0.25
(d) 1.66 (e) 0.5 300
38. 115 ÷ 5 + 12 × 6 =? +64 ÷ 4 − 35
(a) 114 (b) 118 (c) 108 250
(d) 116 (e) 111
200
39. 41% of 600 − 250 =? −77% of 900
A B C D
(a) 693 (b) 675 (c) 684
(d) 679 (e) 689 2008 2018
360
40. = 73 + 33 46. What is average of residents residing in society A in
?
(a) 3.4 (b) 4.3 (c) 3.1 2008, B in 2018, C in 2018 & D in 2008?
(d) 3.6 (e) 3.9 (a) 355 (b) 360 (c) 365
1 3 2 5 (d) 370 (e) 350
41. − 5 + 4 3 =? + 6
2
11 11 11 47. Residents residing in society B in 2008 are what percent
(a) 3 15 (b) 515 (c) 3 12 more/less than average of residents residing in society
11 11
(d) 2 (e) 4 D in 2008 & 2018?
12 15 6 6 6
42. 65 × 3 ÷ 13 + 67 −? = 81 ÷ 9 × 2 2 (a) 5 11 % (b) 3 11 % (c) 6 11 %
6 6
(a) 9 (b) 5 (c) 12 (d) 7 11 % (e) 4 11 %
(d) 8 (e) 14
48. Which society shows maximum percentage increase in
43. (2744)1/3 + (18)2 − 121 = ? −69 × 5 no. of residents from year 2008 to 2018?
(a) 658 (b) 568 (c) 666 (a) Both A & C (b) Both A & D (c) Both C & D
(d) 656 (e) 562 (d) Both A & B (e) None of these
44. 1111 ÷ 11 + 2002 ÷ 26 + 750 ÷ 25 = ?
49. What is ratio of all residents in all societies in 2008 to
(a) 204 (b) 212 (c) 208
that of in 2018?
(d) 206 (e) 210
(a) 142 : 157 (b) 157 : 142 (c) 162 : 137
5 4
45. × 121 + 1 × 288 = 141+ ? (d) 137 : 162 (e) 97 : 114
11 9
(a) 333 (b) 327 (c) 335 50. What is difference between number of residents residing
(d) 330 (e) 329 in society A & B in 2018 together and that of in society C
Directions (46 – 50): Given bar graph shows the number of & D together in 2008?
residents residing in 4 societies in years 2008, 2018. Read the (a) 30 (b) 24 (c) 20
data carefully and answer the questions carefully. (d) 28 (e) 26

242 adda247.com/teachers | www.sscadda.com | www.bankersadda.com | www.adda247.com


51. A shopkeeper sells two pens, he sold 1 pen at profit and 59. I. 9x + 3y = 15 II. 4x + 5y = 14
other pen at loss. SP of each of the two pens is Rs.300 and (a) x = y or no relation can be established
profit percentage on 1 pen is equal to loss percentage on (b) x > y
other. If overall loss of shopkeeper is 6.25%, then find (c) x ≤ y
difference between cost price of both pen. (d) x < y
(a) Rs.350 (b) Rs.100 (c) Rs.240 (e) x ≥ y
(d) Rs.160 (e) Rs.300 60. I. 2x 2 − x − 1 = 0 II. 3y 2 − 5y + 2 = 0
52. A man received Rs.3456 when he invested Rs.P at 12% (a) x ≤ y
p.a. at SI for 3 years. If he invested Rs. (P + 4400) at (b) x < y
15% p.a. at CI compounding annually for 2 years, then (c) x = y or no relation can be established
find the interest received by him. (d) x ≥ y
(a) Rs.4515 (b) Rs.4960 (c) Rs.4725 (e) x > y
(d) Rs.4185 (e) Rs.4345 Directions (61-65): Find the wrong number in the following
53. Time taken by a boat to cover 162 km each in number series.
downstream and in upstream is 14 hours and 24 61. 2, 3, 6, 15, 45, 156.5, 630
minutes. If speed of stream is 6 km/hr., then find the (a) 2 (b) 15 (c) 3
time taken by boat to cover 240 km in upstream. (d) 156.5 (e) 630
1 2 1
(a) 7 3 hours (b) 18 3 hours (c) 9 3 hours 62. 36, 20, 12, 8, 6, 5.5, 4.5
2
(d) 16 hours
1
(e) 13 hours (a) 8 (b) 36 (c) 5.5
3 3 (d) 4.5 (e) 6
54. C is 100% more efficient than B. A alone can complete a 63. 1, 3, 9, 31, 128, 651, 3913
piece of work in 9 days and B & C together can complete (a) 31 (b) 3 (c) 1
2
the same work in 2 3 days. Find what portion of work will (d) 3913 (e) 128
be completed, if A & B works together for 4 days. 64. 2, 3, 10, 40, 172, 885, 5346
13 8 5
(a) 18 (b) 9 (c) 6 (a) 40 (b) 885 (c) 172
2 17 (d) 3 (e) 10
(d) 3 (e) 18
65. 5, 8, 16, 26, 50, 98, 194
55. Ratio of age of P 2 years ago to age of R 2 years hence is (a) 5 (b) 194 (c) 8
1 : 2 and Q’s present age is 25% more than P’s present (d) 16 (e) 98
age. If average of present age of P & R is 39 years, then
66. A rectangular path of width 3m is surrounding the
find difference between P’s age 5 years hence and R’s
garden whose length is 3m more than its width. If cost of
present age.
painting the path at rate of 0.5Rs/m² is Rs 273 then find
(a) 12 years (b) 17 years (c) 21 years
the area of garden
(d) 15 years (e) 14 years (a) 1525m² (b) 1804 m² (c) 1776 m²
Directions (56–60): Solve the following quadratic equation (d) 1906 m² (e) 1664 m²
and mark the answer as per instructions. 67. In a class percentage of students who passed the exam is
56. I. x 2 − 2x − 143 = 0 II. y 2 − 169 = 0 60% and number of boys & girls who passed the exam is
(a) x > y same. If boys who failed the exam are 200% more than
(b) x < y girls who failed in exam then find the percentage of girls
(c) x ≤ y who failed out of total students
(d) x ≥ y (a) 9% (b) 13% (c) 10%
(e) x = y or no relation can be established (d) 12% (e) 15%
57. I. x 2 − 7x − 18 = 0 II. y 2 − 19y + 90 = 0 68. A man invested Rs.X at 15% p.a. at SI for 4 years and Rs.
(a) x ≤ y (1.35X) at 18% p.a. at SI for 3 years. If total interest
(b) x = y or no relation can be established received by man is Rs.15948, then find value of Rs.
(c) x > y (3.12X).
(d) x ≥ y (a) Rs.50544 (b) Rs.42764 (c) Rs.32580
(e) x < y (d) Rs.47372 (e) Rs.37440
1
58. I. 2x 2 + 5x + 3 = 0 II. y 2 + 4y − 12 = 0 69. A man covers 6 % distance via bus at 80 km/hr, 25% of
4
(a) x ≤ y the distance via car at 120 km/hr., 30% distance via
(b) x > y bicycle at 32 km/hr. and remaining distance via train at
(c) x = y or no relation can be established 62 km/hr. If total distance covered by man is 640km,
(d) x < y then find the total time taken man during the entire
(e) x ≥ y journey.

243 adda247.com/teachers | www.sscadda.com | www.bankersadda.com | www.adda247.com


(a)
65
hours (b) 13 hours (c)
44
hours is 5 : 4. If there are 72 students in the class, then find the
6 3
31 71 average weight of girls in the class.
(d) hours (e) hours
2 6 (a) 54 kg (b) 42 kg (c) 35 kg
70. Average weight of a class is 60kg and average weight of (d) 45 kg (e) 38 kg
boys in the class is 80kg. Ratio of boys to girls in the class

ENGLISH LANGUAGE
Directions (71-78): Read the following passage carefully and cannot eat all of the plants that an herbivore can. Generally,
answer the questions given below them. omnivores eat fruits and vegetables freely, but they can’t eat
Animals of all sorts live together in various ecosystems. grasses and some grains due to digestive limitations.
Within these natural communities, the animals eat specific Omnivores will also hunt both carnivores and herbivores for
diets that connect them together in a food chain. The three meat, including small mammals, reptiles, and insects. Large
diets of animals include creatures that eat only plants, those omnivores include bears and humans. Examples of medium-
that eat only meat, and animals that eat both plants and meat. sized omnivores include raccoons and pigs. Small omnivores
Animals that eat plants exclusively are herbivores, and include some fish and insects such as flies. Omnivore teeth
animals that eat only meat are carnivores. When animals eat often resemble carnivore teeth because of the need for tearing
both plants and meat, they are called omnivores. The balance meat. Omnivores also have flat molars for grinding up food.
of an ecosystem depends on the presence of every type of 71. Why the balance of the ecosystem depends on each kind
animal. If one type of animal becomes too numerous or scarce, of animal?
the entire balance of the ecosystem will change. (a) Scarcity or abundant of a single kind can cause
Carnivores will feed on herbivores, omnivores, and other disturbance in the ecosystem balance.
carnivores in an ecosystem. A natural community depends on (b) Carnivores and Omnivorous should be less in
the presence of carnivores to control the populations of other number than herbivorous.
animals. Large carnivores include wolves and mountain lions. (c) Specific diets of each kind of animals bring food
A large carnivore might hunt down large herbivores such as chain in sync.
elk and deer. Medium-sized carnivores include hawks and (d) Both (b) and (c)
snakes, and these animals typically feed on rodents, birds, (e) None of these
eggs, frogs, and insects. Examples of small carnivores include 72. How natural community depends upon Carnivores?
some smaller birds and toads. These carnivores may eat (a) They feed on grass eating, plant and meat eating
insects and worms. Carnivorous animals have strong jaws and animals.
sharp teeth to enable them to tear and rip prey. These animals (b) They feed on other carnivores.
often have long, sharp claws that they also use to tear prey. (c) They control populations of other animals.
Carnivores depend on sufficient prey in the food chain to give (d) All (a), (b), and (c)
them the food they need. If the herbivore population or the (e) None of these
population of other carnivores declines in an ecosystem,
carnivores may not survive. 73. What is the threat to survival of carnivores?
With a diet comprised of only plants, herbivores can be (a) Insufficient number of preys in the food chain.
surprisingly large animals. Examples of large herbivores (b) Decline in population of herbivores and other
include cows, elk, and buffalo. These animals eat grass, tree carnivores
bark, aquatic vegetation, and shrubby growth. Herbivores can (c) Both (a) and (b)
also be medium-sized animals such as sheep and goats, which (d) Only (b)
eat shrubby vegetation and grasses. Small herbivores include (e) None of these
rabbits, chipmunks, squirrels, and mice. These animals eat 74. Which of the following statement(s) is/are true about
grass, shrubs, seeds, and nuts. An ecosystem must provide herbivores?
_______________(1) plants to sustain herbivores, and many of (a) Their diet include only vegetation
them spend the majority of their lives eating to stay alive. If (b) Their survival depends upon abundant plants
plant availability declines, herbivores may not have enough to (c) A major part of herbivores’ life is spent on eating
eat. This could cause a decline in herbivore numbers, which (d) They have a digestive system suitable to digest
would also impact carnivores. Herbivores usually have special diverse kind of plants
biological systems to digest a variety of different plants. Their (e) All of the above
teeth also have special designs that enable them to rip off the 75. Why Omnivores are said to have an advantage?
plants and then grind them up with flat molars. (a) Diverse diet
Omnivores have an advantage in an ecosystem because their (b) Both (a) and (c)
diet is the most diverse. These animals can vary their diet (c) Diet depends upon food which is available in
depending on the food that is most plentiful, sometimes eating plentiful amount.
plants and other times eating meat. Herbivores have different (d) Only (a)
digestive systems than omnivores, so omnivores usually (e) Only (c)

244 adda247.com/teachers | www.sscadda.com | www.bankersadda.com | www.adda247.com


76. Which is the following can fill the blank (1) as given in 84. (a) Permit (b) Annihilate (c) Customary
the passage, to make the sentence grammatically and (d) Stride (e) None of these.
contextually correct?
Directions (85-89): Each question below has one blank,
(a) Largely (b) Abundant (c) Diversity
which is indicating that something has been omitted. Choose
(d) Quite (e) None of these.
the most suitable option indicating the words that can be used
77. Which of the following words is most similar in meaning to fill up the blank in the sentence to make it meaningfully
as BALANCE, highlighted in the given passage? complete.
(a) Affecting (b) Poignant (c) Equilibrium
(d) Considerate (e) None of these. 85. Maharashtra has been ________________into a new party
system over the past few years, with the BJP trying hard
78. Which of the following words is most opposite in
to replace the Congress as the dominant party in the
meaning with SUFFICIENT, highlighted in the given
State.
passage?
(i) Investigated
(a) Inadequate (b) Ample (c) Plenty
(d) Enough (e) None of these. (ii) Transitioning
(iii) Framing
Directions (79-84): In the following passage there are (a) Only (i)
blanks, each of which has been numbered. These numbers are
(b) Only (ii)
printed below the passage and against each, five options are
(c) Both (ii) and (iii)
given. Find out the appropriate word which fits the blank
(d) Both (i) and (ii)
appropriately to make a grammatically and contextually
(e) None of these.
correct sentence.
Two decades have passed since the mid-day meal became a 86. The theme song of Downton Abbey along with a
part of the daily routine in government schools nationwide. In panoramic shot of the imposing monarchic structure is
this long passage of time, procedures have ________ (79) but enough to __________ memories.
accidents continue to occur. Funds from the Centre flow (i) Evoke
smoothly though procurement of food items faces hurdles of (ii) Recall
different kinds. The latest ________ (80) in mid-meal stories (iii) Sent
concerns milk. Government norms entitle every child to (a) Only (i)
receive 150 ml of milk as part of the mid-day meal. However, (b) Only (ii)
a video revealed recently how one litre of milk was mixed in a (c) Both (ii) and (iii)
bucketful of water so that it would ________ (81) for the more (d) Both (i) and (ii)
than 80 children present that day in a school in rural Uttar (e) None of these.
Pradesh (U.P.). This was somewhat similar to the one
87. With tomato and onion prices continuing to soar, the
reported from U.P. a couple of months ago. In the earlier
Centre is considering a ___________ to increase cultivation
incident, a video showed plain chapatis being served with salt.
of these staple vegetables in the north Indian region.
The two videos made it to the national media; they also
proved useful for the officers who ________ (82) the mid-day (i) Proposal
meal scheme since they also depend on unauthorised (ii) Role
videographers to learn about the reality in schools. Each such (iii) Novel
revelation leads to the same ________ (83) official statement: (a) Only (i)
punish the guilty, locate the video-maker and deal with (b) Only (ii)
him/her. In the latest mid-meal story narrated above, (c) Both (ii) and (iii)
authorities in U.P. have reportedly done the ________ (84) (d) Both (i) and (ii)
needful, i.e. they have fired the apparent culprit who is a para- (e) None of these.
teacher. 88. The Railways will speed up trains to reduce running
79. (a) Balances (b) Stabilized (c) Persistent time and maximise _____________ of infrastructure across
(d) Stabilizing (e) None of these. its network.
80. (a) Stained (b) Retaliate (c) Accusations (i) Collecting
(d) Blunder (e) None of these. (ii) Effectiveness
81. (a) Suffice (b) Assent (c) Patriotism (iii) Utilisation
(d) Benefit (e) None of these. (a) Only (i)
(b) Only (ii)
82. (a) React (b) Negate (c) Supervise
(c) Both (ii) and (iii)
(d) Unify (e) None of these.
(d) Both (i) and (ii)
83. (a) Amalgamate (b) Reflexive (c) Applicable (e) None of these.
(d) Unlikely (e) None of these.

245 adda247.com/teachers | www.sscadda.com | www.bankersadda.com | www.adda247.com


89. There could be more interest rate _________ by the RBI as Direction (95-100): The following questions consist of a
some of the members of the monetary policy committee sentence. Four of the words of the sentences are marked in
have observed that growth is yet to bottom out bold, which may or may not be correctly spelled. Option
(i) Increment corresponding to misspelt word is your answer. If there is no
(ii) Cuts misspelled word in the sentence then choose (d), i.e., ‘All are
(iii) Growths correct’ as your answer.
(a) Only (i) 95. Education, for most of us, is a necessary public good
(b) Only (ii) central to the task of nation building and, like fresh air,
(c) Both (ii) and (iii) is neccessary to make our communities come alive.
(d) Both (i) and (ii) (a) Education (b) Building (c) Neccessary
(e) None of these. (d) Communities (e) All are correct
Directions (90-94): In each of the questions given below four 96. The Budget’s provisons for collecting more in taxes
words are given in bold. These four words may or may not be from the incomes of the super-rich were aimed at
in their correct position. The sentence is then followed by redistributing wealth to bring about more equitable
options with the correct combination of words that should development.
replace each other in order to make the sentence (a) Provisons (b) Incomes (c) Redistributing
grammatically and contextually correct. Find the correct (d) Equitable (e) All are correct
combination of the words that replace each other. If the 97. Ideas rejected more than two decades ago during the
sentence is correct as it then select option (e) as your choice. liberalisaetion phase are back in circulation.
90. Today, thermal accounts (A) capacity generation (B) (a) Rejected
(b) Decades
for about two-thirds the installed(C) generation
(c) Liberalisaetion
capacity(D) in the country.
(d) Circulation
(a) A-B (b) A-C (c) B-D
(e) All are correct
(d) C-D (e) The sentence is correct
98. The proposal to establish a National Research
91. The IMF followed(A) the World Bank in reducing(B) its
Foundation, with an “overarching goal to enable a
forecast for India’s economic(C) growth in the current
culture of research to permeate through our
financial(D) year.
universities” needs to be applauded and widely
(a) A-B (b) A-C (c) B-D
supported.
(d) C-D (e) The sentence is correct (a) Establish (b) Overarching (c) Permeate
92. In pursuit (A) of novel ways to draw caught (B) to the (d) Applauded (e) All are correct
Big Prize, the Academy(C) seems to have been 99. Policymaking has three essenteal ingredients:
attention (D) on the wrong foot again. technical elements, administrative inputs and political
(a) A-B (b) A-C (c) B-D goals and packaging.
(d) C-B (e) The sentence is correct (a) Policymaking
93. With power (A) growth, the demand(B) for economic (b) Essenteal
(C) in India is only going to increase(D) further. (c) Administrative
(a) A-B (b) A-C (c) B-D (d) Packaging
(d) C-D (e) The sentence is correct (e) All are correct
94. The Air Quality Index of Delhi worsened(A) slightly and 100. The Central Bureou of Investigation has set up an Online
expected (B) in the “poor” category and is stayed (C) to Child Sexual Abuse and Exploitation
further deteriorate(D) from the last week of the month. Prevention/Investigation Unit.
(a) A-B (b) A-C (c) B-C (a) Bureou (b) Abuse (c) Exploitation
(d) C-D (e) The sentence is correct (d) Investigation (e) All are correct

Solutions
REASONING ABILITY
Direction (1-5): June F
July H
Month Persons August A
January E October B
April D December C
May G 1. (a); 2. (d); 3. (c); 4. (b); 5. (c);

246 adda247.com/teachers | www.sscadda.com | www.bankersadda.com | www.adda247.com


Direction (6-10): Direction (16-20):

6. (a); 7. (c); 8. (d); 9. (b); 10. (e);


11. (c);
16. (b); 17. (c); 18. (e); 19. (a); 20. (e);
Direction (21-25):
Direction (12-15):
12. (a);

21. (d); 22. (b); 23. (e); 24. (a); 25. (c);
13. (d);
Direction (26-28):

14. (e);

26. (e); 27. (a); 28. (b);


29. (c); “STIR”
Direction s (30-33):
15. (e);
30. (b); 31. (d); 32. (d); 33. (c);
34. (c);

35. (e);

QUANTITATIVE APTITUDE
36. (b); 293 + ? = 324⇒ ? = 31 47. (e); average of total residents in society D in 2008 &
400 + 480
17
37. (c); 34 ×? = 100 × 500 × 10
1 2018= = 440
2
440−420 6
8.5
? = 34 = 0.25 Required % = 440
× 100 = 4 11 %
350−250
38. (a); 23 + 72 = ? + 16 – 35 ⇒ ? = 114 48. (a); society A = 250
× 100 = 40%
370−420
39. (e); 246 – 250 = ? – 693 ⇒ ? = 689 Society B = × 100 = 11.9% (decrease)
420
360 420−300
40. (d); = 73 + 27 Society C = 300
× 100 = 40%
?
480−400
? = 3.6 Society D = 400 × 100 = 20%
1 3 14 5
41. (a); − + =? + 6 Maximum increase in society A & C
2 5 3
15−18+140 5
− 6 =? 49. (d); all residents in 2008=250+420+300+400 = 1370
30
112 11 Total residents in 2018=350+370+420+480= 1620
?= =3 Required ratio = 1370 : 1620 = 137 : 162
30 15
42. (d); 15 + 67 - ? = 18 ⇒ ? = 8
2
50. (c); required difference=(350+370)–(300+400) = 20
43. (e); 14 + 324 – 121 = ? – 345 ⇒ ? = 562 100
51. (d); Cost price of both pens = (300 × 2) × 93.75 = Rs.640
44. (c); 101 + 77 + 30 = ? ⇒ ? = 208 Let profit percentage and loss percentage earned
45. (d); 55 + 416 = 141 + ? ⇒ ? = 330 on both pens be x%.
250+370+420+400 ATQ,
46. (b); required average = = 360 100 100
4 300 × 100+x + 300 × 100−x = 640

247 adda247.com/teachers | www.sscadda.com | www.bankersadda.com | www.adda247.com


200
30000 × (100)2−x2 = 640 ⇒ 10000−x2 = 9375
1 1
II. y 2 + 6y − 2y − 12 = 0
(y – 2) (y + 6) = 0
x = 25
100 100 y = 2, -6
Required difference = 300 × − 300 ×
100−25 100+25 clearly, no relation can be established
= 400 − 240 = Rs.160
59. (d); (II) × 9 – (I) × 4
52. (a); ATQ, On solving,
P×12×3
= 3456 x = 1, y = 2
100
P = Rs. 9600 clearly, x < y
Required amount 60. (c); I. 2x 2 − 2x + x − 1 = 0
2
15
= ((9600 + 4400) (1 + 100) − (9600 + 4400)) (2x + 1) (x – 1) = 0
1
x=- ,1
= 18515 − 14000 = Rs.4515 2
II. 3y 2 − 3y − 2y + 2 = 0
53. (e); Let speed of boat in still water be x km/hr. (3y – 2) (y – 1) = 0
ATQ, 2
162 162 72 y = 3, 1
+ =
x+6 x−6 5
clearly, no relation
x = 24 kmph
240 1
Required time = 24−6 = 13 3 hours 61. (d); Wrong number =156.5
Pattern of series-
54. (e); Let efficiency of B be x units/day
200
So, efficiency of C = x × 100
= 2x units/day So, there should be 157.5 in place of 156.5.
8
Now, total work = (x + 2x) × 3 = 8x units 62. (c); Wrong number= 5.5
8x
Hence, efficiency of A = 9 units/day Pattern of series-
Work completed by A & B together in 4 days =
8x 68x
( 9 + x) × 4 = 9
units
68x
17
Required portion = 9
= 18 So, there should be 5 in place of 5.5
8x
55. (b); Let present age of P be 4x years. 63. (e); Wrong number =128
125
So, present age of Q = 100 × 4x Pattern of series –
= 5x years
Now, present age of R = (4x − 2) × 2 − 2
= (8x − 6)years So, there should be 129 in place of 128
ATQ, 64. (a); Wrong number = 40
4x+8x−6
2
= 39 ⇒ x = 7 Pattern of series –
Required difference = (8 × 7 − 6) − (4 × 7 + 5)
= 50 − 33 = 17 years
56. (e); I. x 2 − 13x + 11x − 143 = 0 So, there should be 39 in place of 40.
(x – 13)(x+11) = 0 65. (d); Wrong number = 16
x = -11, 13
Pattern of series –
II. y 2 = 169
y = ± 13
clearly, no relation can be established
57. (a); I. x 2 − 9x + 2x − 18 = 0 So, there should be 14 in place of 16.
(x – 9) (x + 2) = 0 66. (b); Let width of garden = x m
x = -2, 9 So length of garden = (x+3)m
II. y 2 − 10y − 9y + 90 = 0 According to question
(y – 10) (y – 9) = 0 273
(x + 6) (x + 9) – x(x + 3) = 0.5
y = 9, 10
clearly, x ≤ y x² + 15x + 54 – x² – 3x = 546
12x + 54 = 546
58. (c); I. 2x 2 + 3x + 2x + 3 = 0
x = 41
(x + 1) (2x + 3) = 0
3 Area of garden = 41 × 44 = 1804 m²
x = -1, -2

248 adda247.com/teachers | www.sscadda.com | www.bankersadda.com | www.adda247.com


67. (c); Let total students be 100x 69. (e); Distance covered by bus =
25
× 640 = 40km
400
Then passed students be 60x 25
Passed boys & girls are 30x each. Distance covered by car = × 640
100
Let failed girls = y = 160 km
Now Distance covered by bicycle
30
y + 3y = 40x = 100 × 640 = 192 km
4y = 40x ⇒ y = 10x And distance covered by train
Required percentage = 10% = (640 − (40 + 160 + 192)) = 248 km
68. (e); ATQ, 40 160 192 248 71
Required time = 80 + 120 + 32 + 62 = 6 hours
X×15×4 1.35X×18×3
+ = 15948 5
100 100
0.60X + 0.729X = 15948 70. (c); Number of boys in the class = 72 × 9 = 40
4
X = 12000 Number of girls in the class = 72 × = 32
9
Required value = 3.12 × 12000 (60×72)−(40×80) 4320−3200
Required average = = = 35 kg
= Rs.37440 32 32

ENGLISH LANGUAGE
71. (a); Refer to the first paragraph. The hint can be drawn 75. (b); Refer to the fourth paragraph. The hint can be
from the lines, ‘The balance of an ecosystem drawn from the lines:
depends on the presence of every type of animal. If Option (a): Omnivores have an advantage in an
one type of animal becomes too numerous or ecosystem because their diet is the most diverse.
scarce, the entire balance of the ecosystem will Option (c): These animals can vary their diet
change.’ depending on the food that is most plentiful,
In this way, option (a) is justified with the given sometimes eating plants and other times eating
lines. Hence, it is the correct answer choice. meat.
72. (d); Refer to the second paragraph. The hint can be Hence, option (b), [Both (a) and (c)] is the right
drawn from the lines, ‘Carnivores will feed on answer choice.
herbivores, omnivores, and other carnivores in an 76. (b); Abundant can fill in the blank (1) to make the
ecosystem. A natural community depends on the sentence grammatically and contextually correct.
presence of carnivores to control the populations Hence, option (b) is the right answer choice.
of other animals.’ 77. (c); Equilibrium is most similar in meaning with the
Option (a), (b), and (c) are justified with the above given word. Hence, option (c) is the right answer
lines. Hence option (d)[All (a), (b), and (c)] is the choice.
right answer choice.
78. (a); Inadequate is most opposite in meaning with the
73. (c); Refer to the last part of second paragraph. The hint given word. Hence, option (a) is the right answer
can be drawn from the lines, ‘Carnivores depend on choice.
sufficient prey in the food chain to give them the
food they need. If the herbivore population or the 79. (b); Stabilized can fit the blank (79) to make it
population of other carnivores declines in an grammatically and contextually correct sentence.
ecosystem, carnivores may not survive.’ Hence, option (b) is the right answer choice.
From these lines, both option (a) and (b) are 80. (d); Blunder can fit the blank (80) to make it
justifies. Hence, option (c)[Both (a) and (b)] is the grammatically and contextually correct sentence.
right answer choice. Hence, option (d) is the right answer choice.
74. (e); Refer to the third paragraph. The hint can be drawn 81. (a); Suffice can fit the blank (81) to make it
from the lines; grammatically and contextually correct sentence.
Option (a): With a diet comprised of only plants, Hence, option (a) is the right answer choice.
herbivores can be surprisingly large animals. 82. (c); Supervise can fit the blank (82) to make it
Option (b): An ecosystem must provide abundant grammatically and contextually correct sentence.
plants to sustain herbivores Hence, option (c) is the right answer choice.
Option (c): many of them spend the majority of
83. (b); Reflexive can fit the blank (83) to make it
their lives eating to stay alive
grammatically and contextually correct sentence.
Option (d): Herbivores usually have special
Hence, option (b) is the right answer choice.
biological systems to digest a variety of different
plants. 84. (c); Customary can fit the blank (84) to make it
Hence, option (e), [All of the above] is the right grammatically and contextually correct sentence.
answer choice. Hence, option (c) is the right answer choice.

249 adda247.com/teachers | www.sscadda.com | www.bankersadda.com | www.adda247.com


85. (b); From the given options, (i) will make the given 93. (b); In the given question, ‘Economic’ at its current
sentence grammatically incorrect whereas option position fails to impart reasonable meaning to the
(iii) is out of context. So, only ‘Transitioning’ will fit sentence, also, it is not in accordance with the
the given blank to make the given sentence both sentence structure. But, interchanging it with
grammatically and contextually correct. Hence, the ‘Power’ will make the given sentence both
correct answer choice would be option (b) grammatically and contextually correct. Hence, the
86. (d); Evoke means bring or recall. correct answer choice would be option (b)
So, from the given options, ‘Evoke’ and ‘Recall’ fits 94. (c); In the given statement, ‘stayed’ and ‘expected’ at
the given blank to make the given sentence both their current position fails to add a reasonable
grammatically and contextually correct. Hence, the
sense in the sentence but interchanging them will
correct answer choice would be option (d)
make the given sentence both grammatically and
87. (a); From the given options, option (ii) and (iii) will contextually correct. Hence, the correct answer
make the given sentence contextually meaningless. choice would be option (c)
So, only ‘proposal’ fits the given blank to make the
given sentence both grammatically and 95. (c); From the given words, spelling of ‘Neccessary' is
contextually correct. Hence, the correct answer incorrect, instead it should be ‘Necessary’ which
choice would be option (a) means essential. Hence, the correct answer choice
would be option (c)
88. (c); As the given sentence is talking about increasing
the speed of trains to reduce running time and 96. (a); From the given words, spelling of ‘Provisons’ is
about the infrastructure across the railway incorrect instead it should be ‘Provisions’ which
network, the appropriate filler for the given blank means the action of providing or supplying
would be ‘Effectiveness’ and ‘utilisation’. Hence, something for use. Hence, the correct answer
the correct answer choice would be option (c) choice would be option (a)
89. (b); Taking hint from the last part of the sentence which 97. (c); From the given options, spelling of ‘Liberalisaetion’
is talking about the low growth, the appropriate is incorrect instead it should be ‘Liberalisation’
filler for the given blank would be ‘cuts’. Hence, the which means the removal or loosening of
correct answer choice would be option (b) restrictions on something. Hence, the correct
90. (a); In the given sentence, ‘Accounts’ and ‘Generation’ answer choice would be option (c)
at their current position fails to impart valid 98. (e); There is no spelling error in the highlighted words.
meaning to the sentence. But interchanging them
Hence, the correct answer choice would be option
will make the given sentence both grammatically
(e)
and contextually correct. Hence, the correct answer
choice would be option (a) 99. (b); From the given options, spelling of ‘essenteal’ is
91. (e); All of the given highlighted words are correct at incorrect instead it should be ‘essential’ which
their respective position. Hence, the correct means absolutely necessary; extremely important.
answer choice would be option (e) Hence, the correct answer choice would be option
(b)
92. (c); In the given sentence, ‘caught’ and ‘attention’ at
their current position are incorrect as they fails to 100. (a); From the given words, spelling of ‘Bureou’ is
impart valid meaning to the sentence but incorrect instead it should be ‘Bureau’ which
interchanging them will make the sentence both means an office or department for transacting
grammatically and contextually correct. Hence, the particular business. Hence, the correct answer
correct answer choice would be option (c) choice would be option (a)

250 adda247.com/teachers | www.sscadda.com | www.bankersadda.com | www.adda247.com

You might also like